Win up to 100% Scholarship

Register Now

Environment and Ecology

 

2022

Question 1

Which of the following are nitrogen-fixing plants?

  1. Alfalfa

  2. Amaranth

  3. Chickpea

  4. Clover

  5. Purslane (Kulfa)

  6. Spinach

Select the correct answer using the code given below:

(a) 1, 3 and 4 only

(b) 1, 3, 5 and 6 only

(c) 2, 4, 5 and 6 only

(d) 1, 2, 4, 5 and 6

Ans: a

Sub-Theme: Nitrogen fixation

Option (a) is the correct answer: Alfalfa, Chickpea and Clover are nitrogen-fixing plants.

Nitrogen-fixation:

  • Nitrogen fixation is the process by which nitrogen is taken from its molecular form (N2) in the atmosphere and converted into nitrogen compounds useful for other biochemical processes. Fixation can occur through atmospheric (lightning), industrial, or biological processes.
  • Nitrogen-Fixing Plants: A N-fixing crop is a natural way to provide plant-adjusted N without any industrial harm to nature.
  • Nitrogen fixing plants are:
    • Clovers, vetches, and peas are nitrogen- fixing plants used by farmers worldwide and in the Southern Great Plains of US in particular.
    • Peas or beans can be used as a summer nitrogen-fixing cover crop or harvested for food. Both ways, they enrich the soil with plant-suitable N. Southern peas prefer warm seasons and are sown when the soil temperature reaches 60F.
    • Beans: fava (aka faba, broad), alfalfa, green (aka French), runner, field, sweet, peanuts (aka groundnuts), soybeans, cream, black-eyed, or purple-hulled beans, lupins, lentils, cowpeas

 

Question 2

Among the following crops, which one is the most important anthropogenic source of both methane and nitrous oxide?

(a) Cotton

(b) Rice

(c) Sugarcane

(d) Wheat

Ans: b

Sub-Theme: Agriculture and Greenhouse gases

Option (b) is the correct answer: Rice is one of the most important anthropogenic sources of both methane and nitrous oxide.

Agriculture and Greenhouse gases:

  • Methane (CH4) and nitrous oxide (N2O) are the two most important GHGs after carbon dioxide (CO2)
  • The most rapid increase in CH4 emission was found in natural wetlands and rice fields due to increased rice cultivation area and climate warming.
  • N2O emission increased substantially in all the biome types and the largest increase occurred in upland crops due to increasing air temperature and nitrogen fertilizer use.
  • Rice paddies are considered one of the most important sources of CH4 and N2O emissions, which have attracted considerable attention due to their contribution to global warming.

NOTE: Climate Smart Agriculture and GHG Emissions was very much in news, hence the question. Therefore, it is very important to holistically cover both the static and current affairs for UPSC Pre and Mains.

 

Question 3

“System of rice Intensification” of cultivation, in which alternate wetting and drying of rice fields is practised, results in:

  1. Reduced seed requirement

  2. Reduced methane production

  3. Reduced electricity consumption

Select the correct answer using the code given below

(a) 1 and 2 only

(b) 2 and 3 only

(c) 1 and 3 only

(d) 1, 2 and 3

Ans: d

Sub-Theme: System of Rice Intensification

Option (d) is the correct answer: All of the given results come from the System of Rice Intensification.

System of Rice Intensification:

  • The System of Rice Intensification involves cultivating rice with as much organic manure as possible, starting with young seedlings planted singly at wider spacing in a square pattern; and with intermittent irrigation that keeps the soil moist but not inundated, and frequent inter cultivation with weed that actively aerates the soil.
  • SRI is not a standardised, fixed technological method.
  • It is rather a set of ideas, a methodology for comprehensively managing and conserving resources by changing the way that land, seeds, water, nutrients, and human labour are used to increase productivity from a small but well-tended number of seed.
Advantages of SRI In situ Conservation
  • Higher yields – Both grain and straw
  • Reduced duration (by 10 days)
  • Reduced seed requirement
  • Lesser chemical inputs
  • Less water requirement
  • Reduced electricity consumption
  • Less chaffy grain %
  • Grain weight increased without change in grain size
  • Higher head rice recovery
  • Withstand cyclonic gales
  • Cold tolerance
  • Soil health improves through biological activity
  • Reduced methane production

 

 

  • Higher labour costs in the initial years
  • Difficulties in acquiring the necessary skills
  • Not suitable when no irrigation source available

 

 

 

 

 

 

 

 

Question 4

In the context of WHO Air Quality Guidelines, consider the following statements:

1. The 24-hour mean of PM2.5 should not exceed 15 ug/m3 and annual mean of 5 should not exceed 5 ug/m3.

2. In a year, the highest levels of ozone pollution occur during the period of inclement weather.

3. PM10 can penetrate the lung barrier and enter the bloodstream.

4. Excessive ozone in the air can trigger asthma.

Which of the statements given above are correct?

(a) 1, 3 and 4

(b) 1 and 4 only

(c) 2, 3 and 4

(d) 1 and 2 only

Ans: b       

Sub-Theme: Air quality

Statement 1 is correct: The 24-hour ceiling mean of PM 2.5 used to be 25 micrograms but has now dropped to 15.

Statement 2 is incorrect: Ozone at ground level is formed by the reaction with sunlight (photochemical reaction) of pollutants such as nitrogen oxides (NOx) from vehicle and industry emissions and volatile organic compounds (VOCs) emitted by vehicles, solvents and industry. As a result, the highest levels of ozone pollution occur during periods of sunny weather and not during inclement weather.

Statement 3 is incorrect: PM is a common proxy indicator for air pollution. It affects more people than any other pollutant. While particles with a diameter of 10 microns or less, (≤ PM10) can penetrate and lodge deep inside the lungs, the even more health-damaging particles are those with a diameter of 2.5 microns or less, (≤ PM2.5). PM2.5 can penetrate the lung barrier and enter the blood system.

Statement 4 is correct: Excessive ozone in the air can have a marked effect on human health. It causes breathing problems, triggers asthma, reduces lung function and causes lung diseases. Ozone triggers asthma because it is very irritating to the lungs and airways.

 

Question 5

Consider the following:

  1. Carbon monoxide

  2. Nitrogen oxide

  3. Ozone

  4. Sulphur dioxide

Excess of which of the above in the environment is/ are cause(s) of acid rain?

(a) 1, 2 and 3

(b) 2 and 4 only

(c) 4 only

(d) 1, 3 and 4

Ans: b

Sub-Theme: Air Pollution

Option (b) is the correct answer: Nitrous oxide and Sulphur dioxide cause Acid rain.

  • Normal rain has a pH of about 5.6 when pH falls below this; it is called Acid rain (pH 4.2 to 4.4).
  • Acid rain results when Sulfur Dioxide (SO2) & Nitrogen Oxides (NOx) react with water, oxygen & other chemicals to form Sulfuric & Nitric acids.
  • NOx + SO2 + Moisture + other chemicals

→ Acid Rain (HNO3 + H2SO4)

  • Effects: Harms microorganisms in the soil; inhibit the activity of nitrogen fixation bacteria; soil acidification; ocean acidification; affects the growth of plants; food chain; kill aquatic animals; corrosion of metals & weathering of stone buildings & statues.

Common Air Pollutants and Their Effects

Pollutant Sources Effects
 

 

Particulate matter (PM)

Vehicles, power plants, construction activities, oil

refinery, railway yard, industries, etc.

Cardiovascular diseases, reduces visibility (haze)
Nitrogen dioxide (NO2) Emissions from combustion processes Can aggravate respiratory diseases, acid rain, Hazy weather
 

Sulfur dioxide (SO2)

Burning of fossil fuels, power plants, metals processing

and smelting facilities, vehicles

Affects respiratory system & functions of the lungs. causes irritation of the eyes, chronic bronchitis, Acid rain
 

 

Ozone (O3)

Results from photochemical reactions b/w NOx & VOCs in presence of sunlight. Affect the lungs, the respiratory tract, and the eyes, lung cancer

 

NOTE: This particular straightforward question is repeated verbatim from 2013.

 

Question 6

Which one of the following has been constituted under the Environment (Protection) Act, 1986?

(a) Central Water Commission

(b) Central Ground Water Board

(c) Central Ground Water Authority

(d) National Water Development Agency

Ans: c

Sub-Theme: Environment (Protection) Act, 1986/ Central Ground Water Authority Central Ground Water Authority:

    • Central Ground Water Authority has been constituted under Section 3 (3) of the Environment (Protection) Act, 1986 to regulate and control development and management of groundwater resources in the country.
    • Central Ground Water Authority takes care of regulation and control, management and development of groundwater in the country and issues necessary regulatory directions for the purpose.

Environment (Protection) Act, 1986:

  • Enacted aftermath of Bhopal Gas Tragedy (1984) for protecting, improving environment & reducing pollution.
  • Authorises the central government to control & reduce pollution from all sources.
  • Provides procedures for setting standards of emission or discharge of environmental pollutants.

 

Question 7

With reference to Indian laws about wildlife protection, consider the following statements:

  1. Wild animals are the sole property of the government.

  2. When a wild animal is declared protected, such animal is entitled for equal protection whether it is found in protected areas or outside.

  3. Apprehension of a protected wild animal becoming a danger to human life is sufficient ground for its capture or killing.

Which of the statements given above is/are correct?

(a) 1 and 2

(b) 2 only

(c) 1 and 3

(d) 3 only

Ans: b

Sub-Theme: Wildlife Protection Important Provisions of Wildlife (Protection) Act, 1972:

Statement 1 is incorrect: As per the Wild Life (Protection) Act, 1972, every Wild Animal shall be the property of the State Government, and, where such animal is hunted in a sanctuary or National Park declared by the Central Government, such animal or any animal article, trophy, uncured trophy or meat [derived from such animal, or any vehicle, vessel, weapon, trap or tool used in such hunting] shall be the property of the Central Government.

Hunted animals are the sole property of the Government.

Statement 2 is correct: The law governing the subject of wildlife, the Wildlife (Protection) Act, 1972, provides for equal protection for wild animals irrespective of where they are found. It does not discriminate between animals found in protected areas and outside.

Statement 3 is incorrect: Mere apprehension or fear that a wild animal could endanger human life is not a ground for capture or killing. As per Wildlife (Protection) Act, 1972, only if the wild animal becomes a danger to human life or is deceased or disabled beyond recovery can it be allowed to be captured or killed by the competent authority, the Chief Wildlife Warden of the State.

 

Question 8

The “Miyawaki method” is well known for the:

(a) Promotion of commercial farming in arid and semi-arid areas

(b) Development of gardens using genetically modified flora

(c) Creation of mini forests in urban areas

(d) Harvesting wind energy on coastal areas and on sea surfaces

Ans: c

Sub-Theme: Miyawaki Method of Urban Afforestation

Option (c) is the correct answer: Miyawaki method is a method of urban afforestation by turning backyards into mini-forests. It includes planting trees as close as possible in the same area which not only saves space, but the planted saplings also support each other in growth and block sunlight from reaching the ground, thereby preventing the growth of weeds. Thus the saplings become maintenance-free (self- sustainable) after the first three years. It helps to create a forest in just 20 to 30 years while through conventional methods it takes anywhere between 200 to 300 years.

Miyawaki Process

  • The native trees of the region are identified and divided into four layers — shrub, sub- tree, tree, and canopy.
  • The quality of soil is analysed and biomass which would help enhance the perforation capacity, water retention capacity, and nutrients in it, is mixed with it.
  • A mound is built with the soil and the seeds are planted at a very high density — three to five sapling per square meter.
  • The ground is covered with a thick layer of mulch.


 

Question 9

Consider the following pairs:

Wetland/Lake – Location

  1. Hokera Wetland – Punjab

  2. Renuka Wetland – Himachal Pradesh

  3. Rudrasagar Lake – Tripura

  4. Sasthamkotta Lake – Tamil Nadu

How many pairs given above are correctly matched?

(a) Only one pair

(b) Only two pairs

(c) Only three pairs

(d) All four pairs

Ans: b

Sub-Theme: Wetlands

Option (b) is correct:

Wetlands Location and Significance
Hokera Wetland Jammu and kashmir.

A perennial natural wetland that is part of the Jhelum basin.

 Renuka Wetland
  • Himachal Pradesh
  • A naturally occurring wetland that receives freshwater springs and underground karst formations from the lower Himalayas and the Giri river.
Lake Rudijala/ Rudrasagar Lake
  • Melaghar, Tripura
  • A lowland sedimentation reservoir in the northeastern highlands that receives water from three enduring streams that empty into the River Gomti.
 Sasthamkotta
  • Kollam district, Kerala
  • Being Kerala’s largest freshwater lake, it has long been a popular destination for travellers.

 

Question 10.

“If rainforests and tropical forests are lungs of the Earth, then surely wetlands function as its kidneys.” Which one of the following functions of wetlands best reflects the above statements?

(a) The water cycle in wetlands involves surface runoff, subsoil percolation and evaporation.

(b) Algae form the nutrient base upon which fish, crustaceans, molluscs, birds, reptiles and mammals thrive.

(c) Wetlands play a vital role in maintaining sedimentation balance and soil stabilisation.

(d) Aquatic plants absorb heavy metals and excess nutrients.

Ans: d

Sub-Theme: Wetland Conservation

Option (d) is the correct answer: Natural wetlands have often been referred to as “earth’s kidneys” because of their high and long-term capacity to filter pollutants from the water that flows through them. Hence, Aquatic plants absorb heavy metals and excess nutrients.

Functions of Kidney and Wetland

  • Kidneys in our body remove wastes and extra fluid from our body. Our kidneys also remove acid that is produced by the cells of your body and maintain a healthy balance of water, salts, and minerals—such as sodium, calcium, phosphorus, and potassium—in your blood.
  • In the same manner as sediment, excess nutrients and chemicals flow off of the land, wetlands filter the runoff before it reaches open water. Nutrients are stored and absorbed by plants or Sediment settles at the bottom after reaching an area with slow water flow.

 

Question 11.

With reference to “Gucchi” sometimes mentioned in the news, consider the following statements:

  1. It is a fungus.

  2. It grows in some Himalayan Forest areas.

  3. It is commercially cultivated in the Himalayan foothills of north-eastern India.

Which of the statements given above is/are correct?

(a) 1 only

(b) 3 only

(c) 1 and 2

(d) 2 and 3

Ans: c          

Sub-Theme: Species and their habitat

Option (c) is the correct answer: Guchhi Mushroom is a Species of Fungus cultivated in Himalayan Forests.

Statement 1 is correct: Guchhi mushroom is a species of fungus in the family Morchellaceae of the Ascomycota.

Statement 2 is correct: It is grown in the foothills of Himalayas mostly in the temperate forests of Himachal Pradesh, Uttaranchal, and Jammu and Kashmir.

Statement 3 is incorrect: The mushrooms cannot be cultivated commercially and grow in conifer forests across temperate regions.

 

Question 12

Which of the following is not a bird?

(a) Golden Mahseer

(b) Indian Nightjar

(c) Spoonbill

(d) White Ibis

Ans: a      

Sub-Theme: Freshwater Species

Golden Mahseer (freshwater fish)

Option (a) is the correct answer: Golden Mahseer is also known as the tiger of Indian rivers. It is a species of the genus Tor. The Golden Mahseer, the longest-living freshwater fish, is native to mountain and sub-mountain regions found in the Himalayan Rivers (Indus, Ganga and Brahmaputra). The Pong Dam reservoir, around 250 km from state capital Shimla and 190 km from Chandigarh, supports an ample population of the golden Mahseer. Conservation Status: IUCN: Endangered.

 

2021

 

Question 1

In nature, which of the following is/are most likely to be found surviving on a surface without soil?

  1. Fern

  2. Lichen

  3. Moss

  4. Mushroom

Select the correct answer using the code given below.

(a) 1 and 4 only

(b) 2 only

(c) 2 and 3

(d) 1, 3 and 4

Ans: c

Sub-Theme: Biodiversity/Ecosystem

Option (c) is the correct answer: Lichen and Moss can survive on a surface without soil.

Bryophytes:

  • The plant body is differentiated into a small stem and simple leaves, but true roots are
  • They usually grow in moist E.g. Liverworts, mosses
  • Mosses constitute the major component of Indian bryoflora followed by liverworts and hornworts
  • Mosses are non-vascular plants. They do not need soil to survive, instead they have a rhizoid multi-cell anchoring structure that is used to climb and grip over rocky surfaces.
  • Moss thrives in moist and shady

Statement 1 is incorrect: There are four particular types of habitats that ferns are found in: moist, shady forests; crevices in rock faces, especially when sheltered from the full sun; acid wetlands including bogs and swamps; and tropical trees, where many species are epiphytes.

Statement 2 is correct: Lichens are generally found on rock, tree bark, soil, houses, tombstones, cars, old farm equipment, etc. Thus, lichen can be found surviving on surfaces without soil.

Statement 3 is correct: Mosses are nonvascular plants. They do not need soil to survive, instead they have a rhizoid multi-cell anchoring structure that is used to climb and grip over rocky surfaces. Moss thrives in moist and shady areas.

Statement 4 is incorrect: A mushroom is the fleshy, spore-bearing fruiting body of a fungus, typically grows above ground, on soil, etc.

 

Question 2

Consider the following kinds of organisms:

  1. Copepods

  2. Cyanobacteria

  3. Diatoms

  4. Foraminifera

Which of the above are primary producers in the food chains of oceans?

(a) 1 and 2

(b) 2 and 3

(c) 3 and 4

(d) 1 and 4

Ans: b

Sub-Theme: Food chain

Option (b) is the correct answer: Cyanobacteria and Diatoms are primary producers in the food chains of oceans.

Copepods:

  • Any member of the widely dispersed crustacean subclass Copepoda is referred to as one of them.
  • Free-living copepods form a crucial link in the food chain and are often assigned the role of “primary consumers”.
  • Copepods play a significant ecological role by feeding numerous fish species.

Cyanobacteria:

  • Cyanobacteria, also called blue-green algae, are microscopic organisms found naturally in all types of water.
  • These single-celled organisms live in fresh, brackish (combined salt and fresh water), and marine water.
  • These organisms use sunlight to make their own food.

Diatoms:

  • Diatoms are photosynthesizing algae, they have a siliceous skeleton (frustule) and are found in almost every aquatic environment including fresh and marine waters, soils, in fact almost anywhere moist.
  • They are not mobile or can only move a small distance along a substrate by secreting mucilaginous material along a raphe, which is a groove or canal that resembles a slit.
  • Since they are autotrophic, they can only exist in the photic zone (water depths down to about 200m depending on clarity).

Foraminifera:

  • These are any unicellular organism of the rhizopodan order Foraminiferida (formerly Foraminifera), characterized by long, fine pseudopodia that extend from a uninucleated or multinucleated cytoplasmic body encased within a test, or shell.
  • All maritime environments contain foraminifera, which can be either planktic or benthic in nature.
  • In a variety of maritime habitats, foraminifera have been observed to feed mostly on bacteria, tiny diatoms, and nanoplankton.

Statement 1 is incorrect: Free-living copepods form a crucial link in the food chain and are often assigned the role of “primary consumers”.

Statement 2 is correct: Cyanobacteria, also called blue-green algae, are microscopic organisms found naturally in all types of water. These organisms use sunlight to make their own food.

Statement 3 is correct: Diatoms are photosynthesising algae. They are found in almost every aquatic environment including fresh and marine waters. They are primary producers in oceanic food chain.

Statement 4 is incorrect: Foraminifera are found in all marine environments, they may be planktic or benthic in their mode of life. Foraminifera are recorded as feeding chiefly upon bacteria, small diatoms, and nanoplankton in a wide variety of marine environments.

 

Question 3

Which of the following are detritivores?

  1. Earthworms

  2. Millipedes

  3. Woodlice

  4. Jellyfish

  5. Seahorses

Select the correct answer using the code given below

(a) 1, 2 and 4 only

(b) 2, 3, 4 and 5 only

(c) 1, 3 and 5 only

(d) 1, 2, 3, 4 and 5

Ans: c

Sub-Theme: Food Chain

Option (c) is the correct answer: Earthworms, Millipedes and Woodlice are Detritivores.

  • Detritivores also known as detritivores, detritophages, detritus feeders, or detritus eaters are heterotrophs that obtain nutrients by consuming detritus (decomposing plant and animal parts as well as faeces).
  • By doing so, all these detritivores contribute to decomposition and the nutrient cycles.
  • Detritivores are an important aspect of many They can live on any type of soil with an organic component, including marine ecosystems, where they are termed interchangeably with bottom feeders.
  • Typical detritivorous animals include earthworm, millipedes, springtails, woodlice, dung flies, slugs, many terrestrial worms, sea stars, sea cucumbers, fiddler crabs, and some sedentary polychaetes such as worms of the family Terebellidae.

 

Question 4

In which one of the following biogeochemical cycles, the weathering of rocks is the main source of release of nutrients to enter the cycle?

(a) Carbon cycle

(b) Nitrogen cycle

(c) Phosphorus cycle

(d) Sulphur cycle

Ans: c

Sub-Theme: Biogeochemical Cycles

Option (c) is the correct answer: Phosphate ions and other minerals gradually leak out of rocks as a result of weathering and rain.

Phosphorus Cycle:

  • The phosphorus cycle (unlike the carbon and nitrogen cycles) lacks an atmospheric component. Phosphorus is one of the most important nutrients in biological systems since it is a fundamental ingredient of nucleic acids, phospholipids, and several phosphorylated compounds.

56 1

  • Phosphate ions and other minerals gradually leak out of rocks as a result of weathering and rain. This inorganic phosphate is subsequently distributed in soils and water.
  • Plants take up inorganic phosphate from the soil and then these plants are consumed by animals. Phosphate is incorporated into organic molecules such as DNA in plants and animals. When the plants and animals die, it decays, and the organic phosphate is returned to the soil.
  • In the soil, organic forms of phosphate can be made available to plants by bacteria that break down organic matter into inorganic forms of This process is known as mineralisation.
  • Phosphorus from the soil can enter streams and eventually the seas. It can then be absorbed into sediments over time.

 

Question 5

Which of the following have species that can establish a symbiotic relationship with other organisms?

  1. Cnidarians

  2. Fungi

  3. Protozoa

Select the correct answer using the code given below

(a) 1 and 2 only

(b) 2 and 3 only

(c) 1 and 3 only

(d) 1, 2 and 3

Ans: d

Sub-Theme: Species Interaction

Option (d) is the correct answer: All species can establish a symbiotic relationship with other organisms.

Symbiotic Relationship:

  • A relationship between two (or more) species that may or may not be advantageous to one or both is referred to as symbiotic. The organisms could belong to the same species or to separate ones.
  • For eg., Coral has a symbiotic relationship (each gives something to the other and gets something back in return) with ‘zooxanthellae’ microscopic algae which live on coral.

Option 1 is correct: The relationship between cnidarians and dinoflagellate algae is termed “symbiotic” because both the animal host and the algae are benefiting from the association. It is a mutualistic interaction.

Option 2 is correct: Two common mutualistic relationships involving fungi are mycorrhiza (fungi and plant roots) and lichen (fungi and either cyanobacteria or green algae).

Option 3 is correct: Protozoa mostly represent a close mutualistic association between a protozoan and unicellular symbiont such as bacteria, cyanobacteria or/and unicellular algae or protozoans and a multicellular organisms such as ruminants, lower termites, wood-eating cockroaches, plants.

 

Question 6

With reference to the water on the planet Earth, consider the following statements:

1. The amount of water in the rivers and lakes is more than the amount of groundwater.

2. The amount of water in polar ice caps and glaciers is more than the amount of groundwater.

Which of the statements given above is/are correct?

(a) 1 Only

(b) 2 Only

(c) Both 1 and 2

(d) Neither 1 nor 2

Ans: b

Sub-Theme: Water Conservation

Water Resources:

  • 71 % of earth surface constitutes water & only 29 % constitutes a land area of earth
  • 97 % of water area constitutes Oceans & Seas & 2.5 % as freshwater
  • 75 % of freshwater in glaciers & ice caps
  • 7 % of freshwater is groundwater
  • Fresh Water → Glaciers > Groundwater> Ice & Snow > Lakes

Picture7

NOTE: This question is repeated from 2013. Also, it is a direct question from Class 7 NCERT (Geography).

 

Question 7

Which one of the following is used in preparing a natural mosquito repellent?

(a) Congress grass

(b) Elephant grass

(c) Lemongrass

(d) Nut grass

Ans: c

Sub-Theme: Biodiversity/Everyday Science

Option (c) is the correct answer: Lemongrass is used in preparing a natural mosquito repellent.

  • In tropical and subtropical regions of Asia, Africa, and Australia, it is a tall herb that is widely distributed. Lemongrass is most frequently produced in China and India.
  • The lemongrass herb is well known for its effectiveness at warding off mosquitoes, it works as a mosquito repellent. It contains the chemical citronella, which aids in disguising the smells that mosquitoes use to target victims.
  • A 2011 study found that the strong odour of lemongrass oil effectively killed or fully repelled 95% of the mosquitoes from a particular range of species.

 

Question 8

The vegetation of savannah consists of grassland with scattered small trees, but extensive areas have no trees. The forest development in such areas is generally kept in check by one or more or a combination of some conditions. Which of the following are such conditions?

  1. Burrowing animals and termites

  2. Fire

  3. Grazing herbivores

  4. Seasonal rainfall

  5. Soil properties

Select the correct answer using the code given below.

(a) 1 and 2

(b) 4 and 5

(c) 2, 3 and 4

(d) 1, 3 and 5

Ans: c

Sub-Theme: Biomes

Savannah Biome:

  • It is a particular kind of grassland biome made up of vast expanses of open grassland with few trees.
  • There are two kinds of savannahs:
    1. Tropical savannahs
    2. Semi-tropical

Geographical Location:

  • All continents excluding Antarctica have
  • The largest savannahs are found close to the equator in Africa.
  • Serengeti National Park in Tanzania, which is well-known for its enormous wildebeest and zebra populations, is one of the most well-known African savannahs.
  • The park is also home to lions, leopards, elephants, hippos, and gazelles.

Statement 1 is incorrect: The savannah biome relies on their herd numbers and speed for survival, as the vast open areas provide little means of escape from quick predators.

Statement 2 is correct: The combination of high temperatures and little precipitation makes savannahs perfect areas for grass and brush fires during their dry seasons.

Statement 3 is correct: Many of the savannah biome animals are grazing herbivores that migrate through the region.

Statement 4 is correct: Savannah regions have two distinct seasons – a wet season and a dry season. There is very little rain in the dry season. In the wet season vegetation grows, including lush green grasses and wooded areas.

Statement 5 is incorrect: Soil properties influence the type of savannah and its vegetation but don’t contribute to checking it once formed.

 

Question 9

How is permaculture farming different from conventional chemical farming?

1. Permaculture farming discourages monocultural practices but in conventional chemical farming, monoculture practices are predominant.

2. Conventional chemical farming can cause an increase in soil salinity but the occurrence of such a phenomenon is not observed in permaculture

3. Conventional chemical farming is easily possible in semi-arid regions but permaculture farming is not so easily possible in such regions.

4. Practice of mulching is very important in permaculture farming but not necessarily so in conventional chemical farming.

Select the correct answer using the code given below:

(a) 1 and 3

(b) 1, 2 and 4

(c) 4 only

(d) 2 and 3

Ans: b

Sub-Theme: Sustainable Agriculture

Statement 1 is correct: Permaculture is a totally integrated design system that’s modelled on nature. Permaculture farming promotes multi-cropping and integrated farming systems. While chemical farming is more suitable for monoculture cropping due to the use of crop- specific inputs like irrigation, chemical fertiliser and harvesting methods.

Statement 2 is correct: Conventional farming usually alters the natural environment, increases soil salinity, and eliminates biodiversity. Such problems are not seen in permaculture farming as it relies on organic fertilizers.

Statement 3 is incorrect: Chemical farming is based on the intense use of inputs like chemical fertilizers and irrigation that make the soil less fertile over time, thus restricting its practice in semi-arid regions. Thus, conventional farming is not suitable for semi-arid regions. Permaculture involves well-designed systems that don’t produce waste and permaculture tries to imitate well-designed systems. Thus, it can work in arid climates as well.

Statement 4 is correct: Mulching is an important component of Permaculture systems for promoting maximum efficiency.

 

Question 10

In the context of India’s preparation for Climate-Smart Agriculture, consider the following statements:

1. The ‘Climate-Smart Village’ approach in India is a part of a project led by the Climate Change, Agriculture and Food Security (CCAFS), an international research programme.

2. The project of CCAFS is carried out under the Consultative Group on International Agricultural Research (CGIAR) headquartered in France.

3. The International Crops Research Institute for the Semi-Arid Tropics (ICRISAT) in India is one of the CGIAR’s research centres.

Which of the statements given above are correct?

(a) 1 and 2 only

(b) 2 and 3 only

(c) 1 and 3 only

(d) 1, 2 and 3

Ans: d

Sub-Theme: Agriculture and Environment

Climate-Smart Agriculture (CSA):

  • It is a strategy to assist those in charge of managing agricultural systems in efficiently adapting to climate change.
  • The CSA approach pursues the triple objectives of:
    1. Sustainably increasing productivity and incomes
    2. Adapting to climate change
    3. Reducing greenhouse gas emissions where possible
  • The Sustainable Development Goals (SDGs) 1 & 2 envision a world with zero

Statement 1 is correct: The Climate-Smart Village project in India is a CGIAR Research Program on Climate Change, Agriculture and Food Security (CCAFS).

Statement 2 is correct: Climate Change, Agriculture and Food Security (CCFAS) is carried out under CGIAR (formerly the Consultative Group for International Agricultural Research).

Statement 3 is correct: ICRISAT is a CGIAR Research Center. It is a non-profit, non-political public international research organization that conducts agricultural research for development in Asia and sub-Saharan Africa with a wide array of partners throughout the world.


 

Question 11

Why is there a concern about copper smelting plants?

1. They may release lethal quantities of carbon monoxide into the environment.

2. The copper slag can cause the leaching of some heavy metals into the environment.

3. They may release sulphur dioxide as a pollutant.

Select the correct answer using the code given below.

(a) 1 and 2 only

(b) 2 and 3 only

(c) 1 and 3 only

(d) 1, 2 and 3

Ans: b

Sub-Theme: Industrial Pollution

Copper Smelting:

  • It is a process of applying heat to ore in order to extract a base metal.
  • Smelting uses heat and a chemical reducing agent to decompose the ore, driving off other elements as gases or slag and leaving the metal base behind.
  • It is the method of extracting pure elemental copper (Cu) from copper concentrate (CuFeS2).
  • Some facilities that carry out metal and smelting processes are known to emit high quantities of air pollutants such as hydrogen fluoride, sulfur dioxide, oxides of nitrogen, offensive and noxious smoke fumes, vapours, gases, and other toxins.
  • A variety of heavy metals: Lead, arsenic, chromium, cadmium, nickel, copper, and zinc are also released by the facilities.
    • In addition, some smelting processes can also produce large quantities of solid waste, known as slag, which usually contains significant amounts of contaminants. This slag may leach heavy metals (arsenic, cadmium, lead or mercury depending on the composition of the ore) into groundwater reservoirs.

Statement 1 is incorrect: Copper smelting does not release lethal quantities of carbon monoxide into the environment.

Statement 2 is correct: Some smelting processes can also produce large quantities of solid waste, known as slag. This slag may leach heavy metals (arsenic, cadmium, lead or mercury depending on the composition of the ore) into groundwater reservoirs.

Statement 3 is correct: Smelting processes are known to emit high quantities of air pollutants such as hydrogen fluoride, sulphur dioxide, oxides of nitrogen, offensive and noxious smoke fumes, vapours, gases, and other toxins.

 

Question 12

With reference to furnace oil, consider the following statements:

  1. It is a product of oil refineries.

  2. Some industries use it to generate power.

  3. Its use causes sulphur emissions into the environment.

Which of the statements given above are correct?

(a) 1 and 2 only

(b) 2 and 3 only

(c) 1 and 3 only

(d) 1, 2 and 3

Ans: d

Sub-Theme: Air Pollution/Emissions

Furnace oil:

  • Furnace oil or fuel oil is a dark viscous residual product of crude-oil It is used as a fuel in different types of combustion equipment.
  • The emissions of oxides of sulphur are a direct result of the sulphur content of the fuel oil.

Applications of Furnace oil:

  • Marine engines and slow speed engines for power generation;
  • Drying tea leaves;
  • Thermic fluid heaters and hot air generators

Statement 1 is correct: Furnace oil or fuel oil is a dark viscous residual product of crude-oil distillation. It is used as a fuel in different types of combustion equipment.

Statement 2 is correct: Marine engines and slow speed engines for power generation use Furnace oil.

Statement 3 is correct: The emissions of oxides of sulphur are a direct result of the sulphur content of the fuel oil.

 

Question 13

Magnetite par ticles, suspected to cause neurodegenerative problems, are generated as environmental pollutants from which of the following?

  1. Brakes of motor vehicles

  2. Engines of motor vehicles

  3. Microwave stoves within homes

  4. Power plants

  5. Telephone lines

Select the correct answer using the code given below.

(a) 1, 2, 3 and 5 only

(b) 1, 2 and 4 only

(c) 3, 4 and 5 only

(d) 1, 2, 3, 4 and 5

Ans: b

Sub-Theme: Industrial Pollution

Option (b) is the correct answer: Brakes of motor vehicles, engines of motor vehicles, and power plants are sources of Magnetic Particles.

Magnetite particles:

  • Magnetite particles are a highly magnetic mineral form of iron and are known to be present in air pollution.
  • Exposure to airborne particulate matter (PM) is associated with pulmonary, cardiovascular and neurological

Option 1 is correct: Vehicle brake systems are the major source of airborne magnetite at the roadside.

Options 2 and 4 are correct: Petrol and diesel- engine exhaust and power plants are also a source of airborne magnetite.

Options 3 and 5 are incorrect: There is no adequate evidence supporting the generation of magnetite particles from sources like microwave Stoves within homes and telephone lines.

 

Question 14

The ‘Common Carbon Metric’, supported by UNEP, has been developed for

(a) Assessing the carbon footprint of building operations around the world

(b) Enabling commercial farming entities around the world to enter carbon emission trading

(c) Enabling governments to assess the overall carbon footprint caused by their countries

(d) Assessing the overall carbon foot-print caused by the use of fossil fuels by the world in a unit time

Ans: a

Sub-Theme: Air Pollution/Carbon Emission Option (a) is the correct answer: The carbon footprint of building operations is assessed through ‘Common Carbon Metric’.

  • The Common Carbon Metric is a protocol developed by United Nations Environment Program’s Sustainable Buildings & Climate Initiative (UNEP-SBCI) for measuring energy use & reporting GHG emissions from Building Operations.
  • Its purpose is to support greenhouse gas (GHG) emissions reductions through accurate measurement of energy efficiency improvements in building operations.

 

Question 15

What is blue carbon?

(a) Carbon captured by oceans and coastal ecosystems

(b) Carbon sequestered in forest biomass and agricultural soils

(c) Carbon contained in petroleum and natural

(d) Carbon present in atmosphere

Ans: a

Sub-Theme: Carbon Sink

Blue Carbon:

  • Carbon captured by the world’s ocean and coastal ecosystems.
  • Three types of coastal ecosystems — mangroves, seagrasses and tidal marshes — store half the “blue” carbon buried beneath the ocean floor.
  • These areas absorb and store carbon at a much faster rate than other areas, such as forests, and can continue to do so for millions of years.
  • The carbon found in coastal soil is often thousands of years old.

Blue Carbon Initiative:

  • Conservation International (CI), the International Union for Conservation of Nature (IUCN) and the Intergovernmental Oceanic Commission (IOC) of UNESCO has collaborated with governments and organisations across the world to develop mechanisms for ensuring coastal Blue Carbon ecosystems.

It also comprises:

  • Engagement of  local, national and international organisations
  • Comprehensive methods  for carbon accounting
  • Incentive mechanism
  • Scientific research for climate mitigation

 

Question 16

Consider the following animals:

  1. Hedgehog

  2. Marmot

  3. Pangolin

To reduce the chance of being captured by predators, which of the above organisms rolls up/roll up and protects/protects its/their vulnerable parts?

(a) 1 and 2

(b) 2 only

(c) 3 only

(d) 1 and 3

Ans: d

Sub-Theme: Food Chain/Species

Statement 1 is correct: Hedgehogs are small mammals with short limbs and a body low to the ground. Their most distinctive characteristic is the thousands of stiff, sharp spines – harder and sharper than those of a porcupine – that cover the animal’s back and sides, like a pincushion filled with needles. If attacked they will curl into a prickly and unappetizing ball that deters most predators. IUCN Red List Status: Least concern

Statement 2 is incorrect: Marmot are any of 14 species of giant ground squirrels found primarily in North America and Eurasia. Marmots are well suited for life in cold environments and have small fur-covered ears, short, stocky legs, and strong claws for digging. Due to the absence of scales or spines, they do not roll up and protect their vulnerable parts. IUCN Red Listed as: Least Concern.

Statement 3 is correct: Pangolin, also called scaly anteaters because of their preferred diet, pangolins are the most trafficked mammal in the world. If touched or grabbed it will roll up completely into a ball, while the sharp scales on the tail can be used to lash out.


 

Question 17

Which one of the following is a filter feeder?

(a) Catfish

(b) Octopus

(c) Oyster

(d) Pelican

Ans: (c)

Sub-Theme: Species

Option (c) is the correct answer: Oyster is a Filter Feeder.

Filter feeders are a subgroup of suspension feeding animals that feed by straining suspended matter and food particles from water, typically bypassing the water over a specialised filtering structure. Clams, krill, sponges, oysters, baleen whales, and numerous fish are among the creatures that rely on this way of feeding (including some sharks). Filter feeders include some birds like flamingos and several varieties of duck. Filter feeders are regarded as ecosystem engineers since they might be crucial in the clarification of water. They serve as indicator organisms and are crucial in bioaccumulation.

 

Question 18

With reference to ‘palm oil’, consider the following statements:

1. The palm oil tree is native to Southeast Asia.

2. Palm oil is a raw material for some industries producing lipstick and perfumes.

3. The palm oil can be used to produce biodiesel.

Which of the statements given above are correct?

(a) 1 and 2 only

(b) 2 and 3 only

(c) 1 and 3 only

(d) 1, 2 and 3

Ans: b

Sub-Theme: Species

Palm Oil (Native to Africa)

Statement 1 is incorrect: Oil palm trees are native to Africa but were brought to South-East Asia over 100 years ago as an ornamental tree crop.

Statement 2 is correct: Non-food applications of Palm oil are Cosmetics, toiletries, soaps and detergents. Oleochemical industry, as a base material for laundry detergents, household cleaners and cosmetics.

Statement 3 is correct: Palm oil can be used to produce biodiesel, which can be used in compression ignition engines, i.e., diesel engines without any modifications.

Applications of Palm Oil

Food-based applications Cooking oil, substitute for butter, vanaspati/ vegetable ghee, margarine, confectionery and bakery fats, ice cream, coffee creamers,              emulsifiers, vitamin E supplements among others.
Non-food applications Cosmetics, toiletries, soaps and detergents. Oleochemical industry, as a base material for laundry detergents,    household cleaners and cosmetics. Also, Palm oil can be used to produce biodiesel.

 

NOTE: Recently, Prime Minister Narendra Modi has announced this new national initiative on palm oil and in August, 2021, Environmental activists and politicians have expressed concerns over the centre’s proposal to promote palm oil cultivation in the Northeastern states and in the Andaman and Nicobar islands. Hence, the question!!

 

Question 19

Consider the following statements:

  1. Moringa (drumstick tree) is a leguminous evergreen tree.

  2. The Tamarind tree is endemic to South Asia.

  3. In India, most of the tamarind is collected as minor forest produce.

  4. India exports tamarind and seeds of moringa.

  5. Seeds of moringa and tamarind can be used in the production of biofuels.

Which of the statements given above are correct?

(a) 1, 2, 4 and 5

(b) 3, 4 and 5

(c) 1, 3 and 4

(d) 1, 2, 3 and 5

Ans: b

Sub-Theme: Species

Statement 1 is incorrect: Moringa is a fast- growing, drought-resistant tree native to the Indian subcontinent. Found in the wild in the sub- Himalayan regions of Northern India and now grown worldwide in the tropics and sub-tropics.

Statement 2 is incorrect: Tamarind is a leguminous tree bearing edible fruit that is indigenous to tropical Africa.

Statements 3 and 4 are correct: In India most of the Tamarind is collected as Minor Forest Produce with Minimum Support Price. Tamarind is also actively exported from India.

Statement 5 is correct: Biodiesel is derived from the tamarind seed through the transesterification process as potential alternative feedstock for the diesel engine.

NOTE:  Recently,  the  Agricultural and Processed Food Products Export Development Authority (APEDA) has been supporting private entities in creating the necessary infrastructure to promote Moringa products exports from India. Hence, the question!

 

2020

 

Question 1

What are the advantages of fertigation in agriculture?

1. Controlling the alkalinity of irrigation water is possible.

2. Efficient application of Rock Phosphate and all other phosphatic fertilizers is possible.

3. Increased availability of nutrients to plants is possible.

4. Reduction in the leaching of chemical nutrients is possible.

Select the correct answer using the code given below:

(a) 1, 2 and 3 only

(b) 1, 2 and 4 only

(c)1, 3 and 4 only

(d) 2, 3 and 4 only

Ans: c

Sub-Theme: Agriculture and Environment

Fertigation:

  • It is a technique for applying fertiliser where the drip irrigation system incorporates the fertiliser into the irrigation water. In this technology, irrigation is used to evenly disperse fertiliser solutions.
  • Fertigation is practised extensively in commercial agriculture and horticulture.
  • Fertigation is used to supplement nutrients that are already present or to make up for nutrient shortages found during plant tissue
  • It is usually practised on high-value crops such as vegetables, turf, fruit trees, and ornamentals.

Statement 1 is correct: Drip irrigation also avoids water spillage on the field which could have promoted weed growth or increased soil alkalinity due to water logging. So, fertigation also helps control weed growth and cut down alkalinity.

Statement 2 is incorrect: Rock phosphate is not soluble and thus not suitable for fertigation.

Statement 3 is correct: Accurate placement of nutrient, where the water goes the nutrient goes as well. Thus, increased nutrient absorption by plants.

Statement 4 is correct: Reduction of fertiliser, chemicals, and water. It leads to reduction in the leaching of chemicals into the water supply.

 

Question 2

What is/are the advantage/ advantages of zero tillage in agriculture?

  1. Sowing of wheat is possible without burning the residue of the previous crop.

  2. Without the need for a nursery of rice saplings, direct planting of paddy seeds in the wet soil is possible.

  3. Carbon sequestration in the soil is possible.

Select the correct answer using the code given below:

(a) 1 and 2 only

(b) 2 and 3 only

(c) 3 only

(d) 1, 2 and 3

Ans: d

Sub-Theme: Sustainable Agriculture Benefits/Advantages of Zero tillage or No-till farming:

    • In some cases it may reduce labour, fuel, irrigation and machinery costs.
    • No-till farming decreases the amount of soil erosion tillage causes in certain soils, especially in sandy and dry soils on sloping
    • No-till can increase yield because of higher water infiltration and storage capacity, and less erosion.
    • Another possible benefit is that because of the higher water content, instead of leaving a field fallow it can make economic sense to plant another crop instead.
    • No-till farming has been claimed to increase soil organic matter, and thus increase carbon sequestration.

Statement 1 is correct: Zero tillage is the process where the crop seed will be sown through drillers without prior land preparation and disturbing the soil where previous crop stubbles are present.

Statement 2 is correct: Direct Seeded Rice (DSR) is a viable option to reduce the unproductive water flows, with increasing shortage of water, dry-DSR with minimum or zero tillage further enhances the benefits of this technology by saving labour.

Statement 3 is correct: No-till farming has been claimed to increase soil organic matter, and thus increase carbon sequestration.

 

Question 3

In the context of India, which of the following is/ are considered to be practice(s) of eco-friendly agriculture?

  1. Crop diversification

  2. Legume intensification

  3. Tensiometer use

  4. Vertical farming

Select the correct answer using the code given below:

(a) 1, 2 and 3 only

(b) 3 only

(c) 4 only

(d) 1, 2, 3 and 4

Ans: d

Sub-Theme:Climate Smart Agriculture

Statement 1 is correct: The Climate-Smart Village project in India is a program of CGIAR Research Program on Climate Change, Agriculture and Food Security (CCAFS). The CCAFS started piloting the Climate-Smart Village in 2012 in Africa (Burkina Faso, Ghana, Mali, Niger, Senegal, Kenya, Ethiopia, Tanzania, and Uganda) and South Asia (Bangladesh, India, and Nepal).

Statement 2 is correct: Climate Change, Agriculture and Food Security (CCFAS) is carried out under CGIAR (formerly the Consultative Group for International Agricultural Research). Headquarter of CGIAR is in Montpellier, France. CGIAR is a global partnership that unites international organizations engaged in research about food security.

Statement 3 is correct: International Crops Research Institute for the Semi-Arid Tropics (ICRISAT) is a CGIAR Research Center. ICRISAT is a non-profit, non-political public international research organization that conducts agricultural research for development in Asia and sub-Saharan Africa with a wide array of partners throughout the world.

 

Question 4

What is the use of biochar in farming?

1. Biochar can be used as a part of the growing medium in vertical farming.

2. When biochar is a part of the growing medium, it promotes the growth of nitrogen-fixing microorganisms.

3. When biochar is a part of the growing medium, it enables the growing medium to retain water for longer time.

Which of the statements given above is/are correct?

(a) 1 and 2 only

(b) 2 only

(c) 1 and 3 only

(d) 1, 2 and 3

Ans: d

Sub-Theme: Environment and Agriculture

Biochar:

  • Biochar is a high-carbon, fine-grained residue that is currently produced through modern pyrolysis processes (direct thermal decomposition of biomass in the absence of oxygen and preventing combustion).
  • A mixture of solid (the biochar proper), liquid (bio-oil), and gas (syngas) products are created.
  • Increased agricultural output, protection from several foliar and soil-borne diseases, and increased soil fertility in acidic soils (low pH soils) are all potential benefits of
  • The use of biochar in vertical farming can enhance yield significantly.

Benefits:

  • Carbon Sink: The burning and natural decomposition of biomass release large amounts of carbon dioxide and methane into the Earth’s atmosphere.
  • Soil Improvement: The extremely porous nature of biochar is found to be effective at retaining both water and water-soluble nutrients. Its presence on the earth can improve water quality, increase soil fertility, raise agricultural productivity, and reduce pressure on old-growth forests. Using biochar in the soil promotes the growth of nitrogen-fixing microorganisms.
  • Water retention: Biochar is hygroscopic. Thus, it is a desirable soil material in many locations due to its ability to attract and retain water.

Statement 1 is correct: The use of biochar in vertical farming can increase output significantly.

Statement 2 is correct: Using biochar in the soil promotes the growth of nitrogen-fixing microorganisms.

Statement 3 is correct: Biochar is hygroscopic. Thus, it is a desirable soil material in many locations due to its ability to attract and retain water.

 

Question 5

Which of the following are the reasons/factors for exposure to benzene pollution?

  1. Automobile exhaust

  2. Tobacco smoke

  3. Wood burning

  4. Using varnished wooden furniture

  5. Using products made of polyurethane

Select the correct answer using the code given below:

(a) 1, 2 and 3 only

(b) 2 and 4 only

(c) 1, 3 and 4 only

(d) 1, 2, 3, 4 and 5

Ans: a

Sub-Theme: Air pollution

Option (a) is the correct answer: Automobile exhaust, tobacco smoke, and wood burning are the major factors for exposure to benzene pollution.

Benzene

  • At room temperature, the chemical benzene is a colourless or light-yellow liquid. It has a sweet odour and is highly flammable.
  • It evaporates into the air very quickly.
  • Its vapour is heavier than air and may sink into low-lying areas.
  • It dissolves only slightly in water and will float on top of the water.

Sources and Uses:

  • Volcanoes and forest fires are two examples of natural sources of benzene. Additionally, gasoline, cigarette smoke, and crude oil all naturally contain benzene.
  • Some industries use benzene to make other chemicals that are used to make plastics, resins, nylon, and synthetic fibres.
  • It is also used to make some types of lubricants, rubbers, dyes, detergents, drugs, and pesticides.

 

Question 6

According to India’s National Policy on Biofuels, which of the following can be used as raw materials for the production of biofuels?

  1. Cassava

  2. Damaged wheat grains

  3. Groundnut seeds

  4. Horse gram

  5. Rotten potatoes

  6. Sugar beet

Select the correct answer using the code given below:

(a) 1, 2, 5 and 6 only

(b) 1, 3, 4 and 6 only

(c) 2, 3, 4 and 5 only

(d) 1, 2, 3, 4, 5 and 6

Ans: a

Sub-Theme: Eco-Engineering

National Policy on Biofuels 2018:

  • The Policy categorizes biofuels as “Basic Biofuels” First Generation (1G) bioethanol & biodiesel and “Advanced Biofuels” – Second Generation (2G) ethanol, Municipal Solid Waste (MSW) to drop-in fuels, Third Generation (3G) biofuels, bio-CNG etc. to enable the extension of appropriate financial and fiscal incentives under each category.
  • The Policy expands the scope of raw material for ethanol production by allowing the use of Sugarcane Juice, Sugar containing materials like Sugar Beet, Sweet Sorghum, Starch containing materials like Corn, Cassava, Damaged food grains like wheat, broken rice, Rotten Potatoes

NOTE 1: Though all six of the given materials can be used for preparing biofuel, but the question is specific and asked about ‘India’s National Policy on Biofuels’.

NOTE 2: Earlier, UPSC has asked questions on ‘Eco-Engineering’ and specifically on ‘Biofuel’ in 2017. Also, we need to achieve the SDG goal and INDC by 2030. Therefore, it underlines the importance of the theme. Please refer to PYQ for a detailed explanation of Biofuel.

 

Question 7

Which one of the following statements best describes the term ‘Social Cost of Carbon’?

It is a measure, in monetary value, of the:

(a) Long-term damage done by a tonne of CO2 emissions in a given year.

(b) Requirement of fossil fuels for a country to provide goods and services to its citizens, based on the burning of those fuels.

(c) Efforts put in by a climate refugee to adapt to live in a new place.

(d) Contribution of an individual person to the carbon footprint on the planet Earth.

Ans: a

Sub-Theme: Air Pollution/Carbon Pricing/ Carbon Emission

Option (a) is the correct answer: Social cost of Carbon is best described as the measure of monetary value of long-term damage done by a tonne of CO2 emissions in a given year.

Social Cost Of Carbon:

  • The social cost of carbon (SCC) is used to estimate in dollars all economic damage that would result from emitting one ton of carbon dioxide into the atmosphere.
  • It indicates how much it is worth to us today to avoid the damage that is projected for the
  • The SCC puts the effects of climate change into economic terms to help policymakers and other decision-makers understand the economic impacts of decisions that would increase or decrease emissions.
  • India’s country-level social cost of carbon emission was estimated to be the highest at $86 per tonne of CO2. It means the Indian economy will lose $86 by emitting each additional tonne of CO2. India is followed by the US ($48) and Saudi Arabia ($47).

Social Cost:

  • It refers to the total cost borne by society due to the production of a commodity.
  • Social Cost is the cost that is not borne by the firm but incurred by others in the society.
  • Examples include: Large business firms cause air pollution, water pollution and other damages in a particular area which involve costs to society.
  • These costs are treated as social costs. It is also called External Cost.

 

Question 8

Consider the following statements:

1. Coal ash contains arsenic, lead and mercury.

2. Coal-fired power plants release sulphur dioxide and oxides of nitrogen into the environment.

3. High ash content is observed in Indian coal.

Which of the statements given above is/are correct?

(a) 1 only

(b) 2 and 3 only

(c) 3 only

(d) 1, 2 and 3

Ans: d

Sub-Theme: Air Pollution/Coal Ash

Statement 1 is correct: Coal ash contains contaminants like mercury, cadmium and arsenic. Without proper management, these contaminants can pollute waterways, groundwater, drinking water, and the air.

Statement 2 is correct: This statement is direct from NCERT Class 11 Chemistry Chapter 14 (Environmental Chemistry), Pg. 410. Burning of fossil fuels (which contain sulphur and nitrogenous matter) such as coal and oil in power stations and furnaces or petrol and diesel in motor engines produce sulphur dioxide and nitrogen oxides, which leads to ‘acid rain’.

Statement 3 is correct: Indian Coal has comparatively higher ash content than imported coal due to the drift theory of the formation of coal deposits in India. Coal seams formed due to drift theory contain higher ash as compared to the in-situ theory of formation


 

Question 9

If a particular plant species is placed under Schedule VI of The Wildlife Protection Act, 1972, what is the implication?

(a) A licence is required to cultivate that plant.

(b) Such a plant cannot be cultivated under any circumstances.

(c) It is a Genetically Modified crop plant.

(d) Such a plant is invasive and harmful to the ecosystem.

Ans: a

Sub-Theme: Environmental Laws

Option (a) is the correct answer: The Wildlife Protection Act, 1972 has six schedules. Schedule VI has following provisions:

  • No person is allowed to cultivate a specified plant mentioned in schedule 6, except with a licence granted by the Chief WildLife
  • The plant mentioned in the sixth schedule:
    • Beddomes Cycad
    • Blue Vanda
    • Kuth
    • Ladies slipper orchids
    • Pitcher plant
    • Red Vanda
NOTE: In 2017 a similar question came from the same topic, therefore, please refer to the solution of that question for a detailed explanation of Schedules under the Wildlife Protection Act, 1972.

 

Question 10

Steel slag can be the material for which of the following?

  1. Construction of base road

  2. Improvement of agricultural soil

  3. Production of cement

Select the correct answer using the code given below:

(a) 1 and 2 only

(b) 2 and 3 only

(c) 1 and 3 only

(d) 1, 2 and 3

Ans: d

Sub-Theme: Government Policies

Steel Slag

  • Steel slag, a by-product of steel making, is produced during the separation of the molten steel from impurities in steel-making furnaces.
  • The slag occurs as a molten liquid melt and is a complex solution of silicates and oxides that solidifies upon cooling.

Statement 1 and 3 are correct: Steel slag is used as a base course material, the material under the surface layer of an asphalt road, track or surface. Also produce portland slag cement.

Statement 2 is correct: Steel slag can be used in the agricultural sector due to its ability to correct soil acidity. In fact, developed countries like Japan, the USA have taken the lead in making fertilisers using steel-making slag.

NOTE: In Nov, 2019 the Ministry of Steel issued a new Steel Scrap Recycling Policy, hence, the question featured and underlines the importance of reading the daily newspaper.

 

Question 11

In rural road construction, the use of which of the following is preferred for ensuring environmental sustainability or to reduce carbon footprint?

  1. Copper slag

  2. Cold mix asphalt technology

  3. Geotextiles

  4. Hot mix asphalt technology

  5. Portland cement

Select the correct answer using the code given below:

(a) 1, 2 and 3 only

(b) 2, 3 and 4 only

(c) 4 and 5 only

(d) 1 and 5 only

Ans: a

Sub-Theme: Green Technologies

Statement 1 is correct: Copper slag is a by- product obtained during smelting and refining of copper. The use of copper slag in cement and concrete provides potential environmental as well as economic benefits.

Statement 2 is correct: Cold mix asphalt is produced by mixing unheated mineral aggregate with either emulsified bitumen or foamed bitumen. It is suitable for light to medium trafficked roads when used in base and surface courses.

Statement 3 is correct: Government has allowed the use of coir based geotextiles for construction of rural roads under the Pradhan Mantri Gram Sadak Yojana in 2020.

Statement 4 is incorrect: Hot Mix Asphalt (HMA) is a combination of approximately 95% stone, sand, or gravel bound together by asphalt cement, a product of crude oil. The wide use of hot mix technology leads to environmental pollution as these plants emit a huge amount of greenhouse gases.

Statement 5 is incorrect: Portland cement is a major CO2 emitter.

NOTE: There was news on PMGSY new technology guidelines, where the government encourages the use of “Green Technologies” and non-conventional materials for constructing rural roads under Pradhan Mantri Gram Sadak Yojana (PMGSY). Therefore, thorough newspaper reading is very crucial both in terms of UPSC Pre and Mains.

 

Question 12

Consider the following statements:

1. 36% of India’s districts are classified as “overexploited” or “critical” by the Central Ground Water Authority (CGWA).

2. CGWA was formed under the Environment (Protection) Act.

3. India has the largest area under groundwater irrigation in the world.

Which of the statements given above is/are correct?

(a) 1 only

(b) 2 and 3 only

(c) 2 only

(d) 1 and 3 only

Ans: b

Sub-Theme: Major Environmental Organization/ Groundwater

Central Ground Water Authority (CGWA):

  • It was established in accordance with Section 3(3) of the Environment (Protection) Act of 1986 to oversee the management and development of the nation’s groundwater

Statement 1 is incorrect: 256 of our approximately 700 districts have groundwater levels which are “critical” or “over-exploited” as per the latest data from the Central Ground Water Board (2017).

Statement 2 is correct: Central Ground Water Authority (CGWA) constituted under Section 3(3) of ‘The Environment (Protection) Act, (1986)’ regulates the extraction of groundwater through guidelines which are updated regularly.

Statement 3 is correct: FAO research paper shows the countries with the largest extent of areas equipped for irrigation with groundwater, in absolute terms, are India (39 million ha), China (19 million ha) and the USA (17 million ha)

 

Question 13. With reference to India’s Desert National Park, which of the following statements are correct?

1. It is spread over two districts.

2. There is no human habitation inside the park.

3. It is one of the natural habitats of the Great Indian Bustard.

Select the correct answer using the code given below:

(a) 1 and 2 only

(b) 2 and 3 only

(c) 1 and 3 only

(d) 1, 2 and 3

Ans: c          

Sub-Theme: National Parks/Biosphere Reserve

Desert National Park, Rajasthan(Jaisalmer and Barmer, Rajasthan)

  • With a 3162 km2 area, this national park is one of the biggest.
  • The environment of the Thar Desert is well- illustrated by the Desert National Park.
  • The main landform is made up of permanent dunes, jagged boulders, and compact salt lake bottoms.
  • Flora: ronj, palm trees, ber, dhok.
  • Mammals: desert fox, Bengal fox, desert cat, wolf, hedgehog, chinkara.
  • Reptiles: spiny-tailed lizard, monitor lizard, saw-scaled viper, Russell’s viper, common krait.
  • Avifauna: Sandgrouse, Indian bustard, partridges, bee-eaters, larks, and shrikes are year-round inhabitants of the area, while demoiselle crane and houbara bustard migrate there during the winter. Raptors include falcons, long-legged and honey buzzards, and tawny and steppe eagles.
  • Human Activity: Normally, human activity is prohibited in national parks, but several villages were located inside the Desert National Park and lacked access to even the most basic services, including as water, power, and roads.

Statement 1 is correct: Desert National Park is situated in the west Indian state of Rajasthan and spreads over the districts of Jaisalmer and Barmer.

Statement 2 is incorrect: Normally human activities are not allowed in National Park but many villagers lived inside Desert National park, and they did not have access to even basic facilities like road, electricity and water.

Statement 3 is correct: The Thar desert harbours a wide array of flora and faunal species. It is the only place where Rajasthan State Bird, Great Indian Bustard are found naturally.

 

 

Question 14

Among the following Tiger Reserves, which one has the largest area under “Critical Tiger Habitat”?

(a) Corbett

(b) Ranthambore

(c) Nagarjunsagar-Srisailam

(d) Sunderbans

Ans: c

Sub-Theme: National Parks/Biosphere Reserve

Option (c) is the correct answer: Nagarjunsagar- Srisailam is the largest tiger reserve in India having a Core/ Critical Tiger Habitat of 3721 Sq. Km. The tiger reserve spans five districts in Telangana and Andhra Pradesh. The Nallamala Hills make up the majority of the region. The reserve is home to the multifunctional reservoirs Srisailam and Nagarjunasagar. This reserve’s basin is bisected by the Krishna River.

Nagarjunsagar-Srisailam

  • Background information: It was alerted in 1978 and placed under Project Tiger’s protection in 1983.
  • The name was changed to “Rajiv Gandhi Wildlife Sanctuary” in 1992.
  • Extent: The tiger reserve spans five districts in Telangana and Andhra Pradesh.The Nallamala Hills make up the majority of the region.
  • The reserve is home to the multifunctional reservoirs Srisailam and Nagarjunasagar.
  • River: This reserve’s basin is bisected by the Krishna River.
  • Fauna: In addition to the Bengal Tiger, which serves as the area’s top predator, other other species are also present here, including the leopard, rusty-spotted cat, pangolin, mugger crocodiles, Indian rock python, and countless different types of birds.
  • Flora: A variety of woodlands, from southern mixed deciduous tropical forest to southern thorn forest. Multiple types of shrubs and bamboo thickets also grow in this area.NOTE: According to the fourth cycle of the All India Tiger Estimation (AITE) the number of tigers in the Nagarjunasagar Srisailam Tiger Reserve (NSTR) in Andhra Pradesh has gone up, reversing the steady decline in their number over a period


 

Question 15

Which one of the following protected areas is well- known for the conservation of a sub-species of the Indian swamp deer (Barasingha) that thrives well on hard ground and is exclusively graminivorous?

(a) Kanha National Park

(b) Manas National Park

(c) Mudumalai Wildlife Sanctuary

(d) Tal Chhapar Wildlife Sanctuary

Ans: a

Sub-Theme: National Parks/Biosphere Reserve

Option (a) is the correct answer: Kanha National Park is located in Madhya Pradesh’s Mandla and Balaghat districts, covering an area of 940 square kilometres. There were two sanctuaries in the modern-day Kanha region: Hallon and Banjar. Kanha National Park was established in 1955, and the Kanha Tiger Reserve was established in 1973. The largest national park in Central India is Kanha National Park. It is the first tiger reserve in India to officially introduce a mascot, “Bhoorsingh the Barasingha”.

Kanha National Park (Mandla and Balaghat districts, Madhya Pradesh)

  • Fauna: The Hard Ground Barasingha, also known as the Swamp Deer or Rucervus duvaucelii, is the state animal of Madhya Pradesh and is only found in the Kanha Tiger Reserve. Tiger, Leopard, Dhole, Bear, Gaur, and Indian Python are some of the other species discovered.
  • Flora: Its evergreen Sal forests are its most famous feature (Shorea Robusta).

 

Question 16

Which of the following are the most likely places to find the musk deer in its natural habitat?

  1. Askot Wildlife Sanctuary

  2. Gangotri National Park

  3. Kishanpur Wildlife Sanctuary

  4. Manas National Park

Select the correct answer using the code given below:

(a) 1 and 2 only

(b) 2 and 3 only

(c) 3 and 4 only

(d) 1 and 4 only

Ans: a

Sub-Theme: National Parks/Biosphere Reserve

Option (a) is the correct answer: To safeguard the critically endangered Musk Deer and its environment, the Askot Musk Deer Sanctuary was created in 1986. The refuge is sometimes referred to as “Green Paradise on Earth.” It is situated in Askot, a tiny village in the Pithoragarh district of Uttarakhand. Fauna:Snow leopard, Himalayan black bear, Himalayan tahr, blue sheep, musk deer, loong, monal, kalij pheasant, and cheer pheasant are some examples of the local fauna.

Gangotri National Park was founded in 1989 and is located in Uttarkashi, Uttrakhand, in the Bhagirathi River’s upper watershed. The Ganga River’s source, Gaumukh in Gangotri Glacier, is situated inside the park. The park area effectively connects Kedarnath Wildlife Sanctuary and Govind National Park. The park is completely surrounded by temperate coniferous forests.The typical vegetation includes chirpine, deodar, fir, spruce, oak, and rhododendron. The park is home to a number of rare and endangered animals, including the snow leopard, musk deer, Himalayan monal, Himalayan snowcock, and blue sheep (also known as bharal).

 

Question 17

With reference to Indian elephants, consider the following statements:

1. The leader of an elephant group is a female.

2. The maximum gestation period can be 22 months.

3. An elephant can normally go on calving till the age of 40 years only.

4. Among the States in India, the highest elephant population is in Kerala.

Which of the statements given above is/are correct?

(a) 1 and 2 only

(b) 2 and 4 only

(c) 3 only

(d) 1, 3 and 4 only

Ans: a

Sub-Theme: Species

Asian Elephants:

Asian elephants come in three subspecies: Indian, Sumatran, and Sri Lankan.

The majority of the continent’s remaining elephants belong to the Indian subspecies, which has the largest range.

Statement 1 is correct: The leader of a group is the oldest female. She guides the herd in their search for food and water sources. These matriarchal units can occasionally divide into smaller, temporary groups.

Statement 2 is correct: The gestation period lasts for 22 months, yielding a single baby.

Statement 3 is incorrect: Although sexually mature in their early teens, elephants generally only start to mate at about 20 years and stop bearing calves at about 50.

Statement 4 is incorrect: Recent census showed Kerala as having 5,706 elephants, while Karnataka with 6049 elephants.


 

Question 18

With reference to India’s biodiversity, Ceylon frogmouth, Coppersmith barbet, Gray-chinned minivet and White-throated redstart are

(a) Birds

(b) Primates

(c) Reptiles

(d) Amphibians

Ans: a

Sub-Theme: Species Diversity

Option (a) is the correct answer: Ceylon frogmouth, Coppersmith barbet, Gray- chinned minivet and White-throated redstart are Birds.

  • Sri Lankan/Ceylon Frogmouth bird lives in a particular location in Sri Lanka, and it is also thought to inhabit the Thattekad bird sanctuary (Kerala). Additionally, it is present in Maharashtra, Goa, and Karnataka. It is usually found on the western side of the Western Ghats. IUCN Status: Least Concern.
  • Coppersmith barbet is renowned for its metronomic call, which resembles the hammering of metal by a coppersmith. It is a permanent inhabitant of Southeast Asia and the Indian subcontinent. IUCN Red List: Least Concern
  • Gray-chinned minivet is a species of bird in the family Campephagidae. It is found from the Himalayas to China, Taiwan and Southeast Asia. Its natural habitat is forests about 1,000–2,000 m (3,300–6,600 ft) in elevation. IUCN Red List: Least-concern.

White-throated redstart is a species of bird in the family Muscicapidae. It can be found in northeastern India, Nepal, Bhutan, central China, and the far northern reaches of Myanmar. From Pangolakha Wildlife Sanctuary, East Sikkim, India in alpine snowy meadows. IUCN Red List: Least Concern

 

2019

 

Question 1

In India, the use of carbofuran, methyl parathion, phorate and triazophos is viewed with These chemicals are used as

(a) pesticides in agriculture

(b) preservatives in processed foods

(c) fruit-ripening agents

(d) moisturizing agents in cosmetics

Ans: a

Sub-Theme: Pesticides in Agriculture

Option (a) is the correct answer: Carbofuran, Methyl Parathion, Phorate and Triazophos are used as Pesticides.

Pesticides

  • In contrast to insecticides, which are used to specifically target and kill insects, pesticides are chemicals that can be used to kill fungus, bacteria, insects, plant diseases, etc.
  • Other categories of chemicals used:
  • Molluscicides control snails, slugs and similar molluscs.
  • By preventing fungal spores from penetrating plant tissue, fungicides manage hazardous fungi for plants.
  • Herbicides control
  • In India, pesticides must be registered with the Ministry of Agriculture & Farmers Welfare’s affiliated Directorate of Plant Protection, Quarantine & Storage.

 

Question 2

In the context of which one of the following are the terms ‘pyrolysis and plasma gasification’ mentioned?

(a) Extraction of rare earth elements

(b) Natural gas extraction technologies

(c) Hydrogen fuel-based automobiles

(d) Waste-to-energy technologies

Ans: d

Sub-Theme: Waste Management

Option (d) is the correct answer: Pyrolysis and Plasma Gasification are associated with Waste-to-energy technologies.

  • Pyrolysis is a process of chemical decomposition of organic matter brought about by heat.
  • In this process, the organic material is heated in the absence of air until the molecules thermally break down to become a gas comprising smaller molecules (known collectively as syngas).
  • Gasification can also take place as a result of partial combustion of organic matter in the presence of a restricted quantity of oxygen or air. The gas, so produced, is known as producer gas.
  • The gases produced by pyrolysis mainly comprise carbon monoxide (25%), hydrogen and hydrocarbons (15%), and carbon dioxide and nitrogen (60%).
  • It is used commercially as a waste to an energy system that converts municipal solid waste into synthesis gas (syngas) containing hydrogen and carbon monoxide that can be used to generate power.

 

Question 3

Why is there a great concern about the ‘microbeads’ that are released into the environment?

(a) They are considered harmful to marine ecosystem.

(b) They are considered to cause skin cancer in children.

(c) They are small enough to be absorbed by crop plants in irrigated fields.

(d) They are often found to be used as food adulterants.

Ans: a

Sub-Theme: Water Pollution

Option (a) is the correct answer: Microbeads (Harmful to Marine Ecosystem)

  • Microbeads are manufactured solid plastic particles of less than one millimetre in their largest dimension.
  • They are most frequently made of polyethene but can be of other petrochemical plastics such as polypropylene and polystyrene.
  • They are used in exfoliating personal care products, toothpaste and in biomedical and health-science research.
  • Microbeads can cause plastic particle water pollution and pose an environmental hazard for aquatic animals in freshwater and ocean
  • The Bureau of Indian Standards (BIS) has classified microbeads as “unsafe” for use in cosmetic products.
  • NGT had earlier directed the Centre to test cosmetic products containing microbeads after a plea sought a ban on their use on the ground they are extremely dangerous for aquatic life and the environment.

 

Question 4

Which of the following statements are correct about the deposits of ‘methane hydrate’?

  1. Global warming might trigger the release of methane gas from these deposits.

  2. Large deposits of ‘methane hydrate’ are found in Arctic Tundra and under the seafloor.

  3. Methane in the atmosphere oxidised to carbon dioxide after a decade or two.

Select the correct answer using the code given below.

(a) 1 and 2 only

(b) 2 and 3 only

(c) 1 and 3 only

(d) 1, 2 and 3

Ans: d

Sub-Theme: Methane Emission

Methane Hydrate:

  • A methane molecule is enclosed in a cage of interlocking water molecules to form the crystalline solid known as methane hydrate.
  • Methane is frozen into a form of water ice known as methane hydrate and is trapped in the sediments under the seafloor in large but relatively unknown quantities.
  • Methane hydrate is an “ice” that only naturally forms in underground deposits under ideal temperature and pressure circumstances.

Statement 1 is correct: Owing to the melting of ice, global warming might trigger the release of methane gas from these deposits hence having a significant impact in climate change.

Statement 2 is correct: Sediment and sedimentary rock units below Arctic permafrost have the temperature and pressure conditions suitable for the formation and stability of methane hydrate.

Statement 3 is correct: Methane is relatively short-lived in the atmosphere; a molecule of methane is oxidised to water and carbon dioxide within a decade or so, mainly by reaction with other trace gases.

 

Question 5

Consider the following:

  1. Carbon monoxide

  2. Methane

  3. Ozone

  4. Sulphur dioxide

Which of the above are released into the atmosphere due to the burning of crop/biomass residue?

(a) 1 and 2 only

(b) 2, 3 and 4 only

(c) 1 and 4 only

(d) 1, 2, 3 and 4

Ans: d

Sub-Theme: Air Pollution

Option (d) is the correct answer: All the given gases are result of Stubble Burning:

  • Burning stubble (parali) is the process of removing crop residue from the field in order to prepare it for the next crop.
  • Farmers in Haryana and Punjab must move quickly in order to plant the upcoming winter crop (Rabi crop), and if they are late due to short winters, they risk suffering significant losses. The cheapest and quickest method to remove the stubble is by burning it.

Effects of Stubble Burning:

  • Pollution: Open stubble burning releases a lot of harmful pollutants into the air, including smoke, particles, greenhouse gases (CO2, N2O, O3 CH4), air pollutants (CO, NH3, NOx, SO2, NMHC), and volatile organic compounds. These pollutants are dangerous to human health.
  • Soil Fertility: Husk burning reduces the fertility of the soil by destroying its
  • Heat Penetration: When stubble burns, heat is produced that permeates the soil, causing the loss of moisture and beneficial
  • Thus, all the above-mentioned gases are emitted due to the burning of crop/biomass

 

Question 6

In the context of which of the following do some scientists suggest the use of cirrus cloud thinning technique and the injection of sulphate aerosol into the stratosphere?

(a) Creating the artificial rains in some regions

(b) Reducing the frequency and intensity of tropical cyclones

(c) Reducing the adverse effects of solar wind on the Earth

(d) Reducing the global warming

Ans: d

Sub-Theme: Geoengineering

Option (d) is the correct answer: Geoengineering is used for Reducing global warming.

  • There is a renewed interest in two dominant methods of geoengineering such as stratospheric sulphate aerosol increase and cirrus cloud thinning aims to reduce global warming and precipitation rates to pre-industrial levels.
  • Geoengineering, often known as climate intervention, is the deliberate and extensive modification of the Earth’s climatic system with the intention of causing adverse global
  • The stratosphere can be injected with sulphate aerosol particles to block some of the sun’s rays and possibly cool the planet.
  • Sulphur droplets are good at scattering the sun’s light back out into space and it does not heat the stratosphere as much as other
  • Cirrus Clouds: The narrow bands of detached clouds or mostly white patches with frail white These could have a silky gloss or a fibrous (hair-like) appearance.
  • Cirrus clouds are always composed of ice crystals, and their transparency depends on how far the crystals are spaced apart.
  • Generally, these clouds barely affect the brightness of the sun when they pass over its They might block its light and erase its contour if they are particularly dense.

 

Question 7

Consider the following statements:

  1. Agricultural soils release nitrogen oxides into the environment.

  2. Cattle release ammonia into the environment.

  3. Poultry industry releases reactive nitrogen compounds into the environment.

Which of the statements given above is/are correct?

(a) 1 and 3 only

(b) 2 and 3 only

(c) 2 only

(d) 1, 2 and 3

Ans: d

Sub-Theme: Climate Change and Agriculture Option (d) is the correct answer: All statements about Climate Change and Agriculture are correct.

  • Total GHG Emissions: share of agriculture in total emissions has gradually declined from 28% in 1994 to 14% in 2016.
  • Emission Classification:
    • Agricultural emissions in India are primarily from the livestock sector (54.6%) and the use of nitrogenous fertilizers (19%).
    • Rice cultivation in anaerobic conditions accounts for a major portion of agricultural emissions (17.5%).
    • The single largest source of nitrous oxide (N2O) emissions is agricultural soils.
  • Air Pollution: As per the World Air Quality Report, 2020, 22 of the 30 most polluted cities in the world are in India and Delhi is the world’s most polluted capital. Delhi suffers severely from air pollution during the winter months due to stubble burning in adjoining states.
  • The Sixth Assessment Report by the IPCC has issued a “code red” to humanity declaring 1.5°C warming of the planet as inevitable.

Statement 1 is correct: Agricultural soils are the largest single source of nitrous oxide (N2O) emissions.

Statement 2 is correct: Cattle account for 80% of the ammonia production.

Statement 3 is correct: Poultry industry recorded an excretion of reactive nitrogen compounds of 0.415 tonnes in 2016.

 

Question 8

In the context of proposals to the use of hydrogen- enriched CNG (H-CNG) as fuel for buses in public transport, consider the following statements: 

1. The main advantage of the use of H-CNG is the elimination of carbon monoxide emissions.

2. H-CNG as fuel reduces carbon dioxide and hydrocarbon emissions.

3. Hydrogen up to one-fifth by volume can be blended with CNG as fuel for buses.

4. H-CNG makes the fuel less expensive than CNG.

Which of the statements given above is/are correct?

(a) 1 only

(b) 2 and 3 only

(c) 4 only

(d) 1, 2, 3 and 4

Ans: b

Sub-Theme: Mitigating Vehicular Emissions Option (b) is the correct answer: Advantages of use of hydrogen-enriched CNG (H-CNG) as fuel.

Hydrogen-enriched CNG (H-CNG)

  • A blended gas known as HCNG is produced by mixing hydrogen with CNG.
  • Since it burns cleaner than propane (C3H8) or LPG, it can be used in place of gasoline, diesel fuel, and those fuels.

Advantages

  • HCNG reduces emissions of CO up to 70%.
  • Enables up to 5 % savings in fuel.
  • First step towards the future Hydrogen economy.
  • Engines can be calibrated to release lower amounts of NO.
  • Engines can run on HCNG with minimal
  • Ideal fuel for heavy-duty vehicles and applications with high loads..
  • Better performance due to higher Octane rating of H2.

Statement 1 is incorrect: HCNG doesn’t eliminate but reduces emissions of CO up to 70%. Statement 2 is correct: It can be used in place of gasoline, diesel fuel and propane (C3H8) / LPG and its combustion produces fewer undesirable gases.

Statement 3 is correct: The blending of hydrogen with CNG provides a blended gas termed as HCNG. Statement 4 is incorrect: Current cost of Hydrogen is more than the cost of Natural Gas. So, HCNG’s cost is more than CNG.

 

Question 9

In India, ‘extended producer responsibility’ was introduced as an important feature in which of the following?

(a) The Bio-medical Waste (Management and Handling) Rules, 1998

(b) The Recycled Plastic (Manufacturing and Usage) Rules, 1999

(c) The e-Waste (Management and Handling) Rules, 2011

(d) The Food Safety and Standard Regulations, 2011

Ans: c

Sub-Theme: E-Waste Management

Option (c) is the correct answer: Laws to manage e-waste have been in place in India since 2011 under The e-Waste (Management and Handling) Rules, 2011, which was amended in 2016 as E-waste (Management) Rules, 2016 recognises producers’ liability for recycling and reducing e-waste in the country.

Extended Producer Responsibility (EPR)

•   It is a policy approach under which producers are given a significant responsibility – financial and/or physical– for the treatment or disposal of post- consumer products.

•   The Central Pollution Control Board (CPCB) has been given the Extended Producer Responsibility (EPR) authorisation under the new e-waste rules.

 

Question 10

Consider the following statements:

1. As per law, the Compensatory Afforestation Fund Management and Planning Authority exists at both National and State levels.

2. People’s participation is mandatory in the compensatory afforestation programmes carried out under the Compensatory Afforestation Fund Act, 2016.

Which of the statements given above is/are correct?

(a) 1 only

(b) 2 only

(c) Both 1 and 2

(d) Neither 1 nor 2

Ans: a

Sub-Theme: Environmental Policies and Initiatives

Compensatory Afforestation Fund Act (Campa Act), 2016

  • Objective: To provide an appropriate institutional mechanism to utilize afforestation funds.
  • Sets up the Central and State Compensatory Afforestation Fund Management & Planning Authorities (CAMPA).
  • It creates a State Compensatory Afforestation Fund under the Public Account of every state, as well as a National Compensatory Afforestation Fund under the Public Account of India.
  • National Fund receives 10% & State Fund gets 90% of funds

Statement 1 is correct: As per the Act, the Compensatory Afforestation Fund Management and Planning Authority (CAMPA) is set up at both central and state levels for expeditious and transparent utilisation of funds released for forest land diverted for non-forest purposes.

Statement 2 is incorrect: There is no such provision in the act, also the term “participation” does not occur in the said Act.

 

Question 11

Consider the following statements:

The Environment Protection Act, 1986 empowers the Government of India to:

1. State the requirement of public participation in the process of environmental protection, and the procedure and manner in which it is sought

2. Lay down the standards for emission or discharge of environmental pollutants from various sources

Which of the statements given above is/are correct?

(a) 1 only

(b) 2 only

(c) Both 1 and 2

(d) Neither 1 nor 2

Ans: b

Sub-Theme: Major Environmental Laws

ENVIRONMENT (PROTECTION) ACT, 1986

  • Objective: To     protect,      improve     the environment & reduce pollution.
  • Enacted aftermath of Bhopal Gas Tragedy (1984).
  • The Act, empowers the Government of India for:
    • Coordination of action by the state,
    • Planning and execution of nationwide programmes,
    • Authorises the central government to control & reduce pollution from all sources.
    • Laying down  environmental quality standards, especially those governing emission or discharge of environmental pollutants,
    • Placing restrictions on the location of industries and so on.
    • Authority to issue direct orders included orders to close, prohibit or regulate any industry.
    • Power of entry for examination, testing of equipment and other purposes and power to analyse the sample of air, water, soil or any other substance from any place.

Statement 1 is incorrect: Environment (Protection) Act, 1986 doesn’t contain any provision related to public participation.

Statement 2 is correct: Clearly, the Central Government lays down the standards for emission or discharge of environmental pollutants from various sources

 

Question 12

As per the Solid Waste Management Rules, 2016 in India, which one of the following statements is correct?

(a) Waste generators have to segregate waste into five categories.

(b) The Rules are applicable to notified urban local bodies, notified towns and all industrial townships only.

(c) The Rules provide for exact and elaborate criteria for the identification of sites for landfills and waste processing facilities.

(d) It is mandatory on the part of the waste generator that the waste generated in one district cannot be moved to another district.

Ans: c

Sub-Theme: Solid Waste Management

Solid Waste Management Rules, 2016

  • Solid Waste Management Rules, 2016 promote the use of compost, conversion of waste into energy, and revision of parameters for landfills location and capacity.
  • It replaced the Municipal Solid Wastes (Management and Handling) Rules, 2000, are now applicable beyond municipal areas and have included urban agglomerations, census towns, notified industrial townships
  • Emphasis should be placed on waste segregation at the source, manufacturer responsibility for packaging and sanitary waste disposal, and user fees for collection, processing, and disposal from the bulk
  • It has also been advised that the bio- degradable waste should be processed, treated and disposed of through composting or bio-methanation within the premises as far as possible and the residual waste shall be given to the waste collectors or agency as directed by the local authority.

 

Question 13

Which one of the following National Parks lies completely in the temperate alpine:

(a) Manas National Park

(b) Namdapha National Park

(c) Neora Valley National Park

(d) Valley of Flowers National Park

Ans: d

Sub-Theme: National Parks/Biosphere Reserve

Option (d) is the correct answer: Valley of Flowers National Park was established in 1982.

Located in Chamoli, in the state of Uttarakhand, between 3352 and 3658 metres above sea level, it is renowned for its meadows of indigenous alpine flowers and the variety of vegetation. The park is entirely inside the temperate alpine region. It is located close to Joshimath in the Garhwal area, tucked in the upper reaches of the Bhyundar Ganga river’s Pushpawati river valley. The Asiatic black bear, snow leopard, musk deer, brown bear, red fox, and blue sheep are among the rare and endangered species that call this area home. Himalayan monal pheasants and other high-altitude birds can also be seen in the park.

 

Question 14

Which of the following are in Agasthyamala Biosphere Reserve?

(a) Neyyar, Peppara and Shendurney Wildlife Sanctuaries; and Kalakad Mundanthurai Tiger Reserve

(b) Mudumalai, Sathyamangalam and Wayanad Wildlife Sanctuaries; and Silent Valley National Park

(c) Kaundinya, Gundla Brahmeswaram and Papikonda Wildlife Sanctuaries; and Mukurthi National Park

(d) Kawal and Sri Venkateswara Wildlife Sanctuaries; and Nagarjunasagar-Srisailam Tiger Reserve

Ans: a          

Sub-Theme: National Parks/Biosphere Reserve

Option (a) is the correct answer: Agasthyamala Biosphere Reserve was established in 2001. It is located near the southernmost tip of the Western Ghats, straddling the boundary between the districts of Pathanamthitta, Kollam, and Thiruvananthapuram in Kerala and Tirunelveli and Kanyakumari in Tamil Nadu, South India. It includes the following wildlife sanctuaries: Kalakkad Mundanthurai Tiger Reserve, Shendurney Wildlife Sanctuary, Peppara Wildlife Sanctuary, and Neyyar Wildlife Sanctuary. It comprises the tropical wet evergreen forests of India, the moist deciduous forests of the South Western Ghats, the montane rain forests of the South Western Ghats, and Shola. The Kanikaran tribe, one of the oldest extant ancient tribes in the world, is also found in Agasthyamalai. In March 2016, it was added to the World Network of Biosphere Reserves of UNESCO.

 

 

Question 15

Consider the following statements:

1. Asiatic lions are naturally found in India only.

2. Double-humped camels are naturally found in India only.

3. One-horned rhinoceros is naturally found in India only.

Which of the statements given above is/are correct?

(a) 1 only

(b) 2 only

(c) 1 and 3 only

(d) 1, 2 and 3

Ans: a         

Sub-Theme: Species

Statement 1 is correct: Asiatic lions were once distributed to the state of West Bengal in the east and Rewa in Madhya Pradesh, in central India. At present Gir National Park and Wildlife Sanctuary is the only abode of the Asiatic lion. IUCN Red List: Endangered

Statement 2 is incorrect: Double-humped Camel or Bacterian Camel is native to the steppes of Central Asia, restricted to the Gobi and Gashun Gobi deserts of northwest China and Mongolia, and Ladakh in India.

Statement 3 is incorrect: Only the Great One- Horned Rhino is found in India, while the other four species spread across Africa, Java and Sumatra. IUCN Red List: Vulnerable. Indian Rhino Vision 2020 (IRV 2020) was launched in 2005 by the Forest Department, Government of Assam, in partnership with WWF India, the International Rhino Foundation, and several other organisations. It aimed to raise the rhino population in Assam to 3,000. Four Protected Areas in Assam have rhinos: Pabitora Wildlife Reserve, Rajiv Gandhi Orang National Park, Kaziranga National Park, and Manas National Park.

 

Question 16

Consider the following statements:

  1. Some species of turtles are herbivores.

  2. Some species of fish are herbivores.

  3. Some species of marine mammals are herbivores.

  4. Some species of snakes are viviparous.

Which of the statements given above are correct?

(a) 1 and 3 only

(b) 2, 3 and 4 only

(c) 2 and 4 only

(d) 1, 2, 3 and 4

Ans: d         

Sub-Theme: Species Diversity

Statement 1 is correct: Unlike other members of its family, such as the hawksbill sea turtle, the green sea turtle is mostly herbivorous.

Statement 2 is correct: Surgeonfish and Parrotfish are algae eaters, other herbivores include the Japanese angelfish, yellow blotch- rabbitfish, and tilapia.

Statement 3 is correct: Manatees and dugongs are the only herbivores among marine mammals.

Statement 4 is correct: Boa constrictors and green anacondas are two examples of viviparous snakes, meaning they give birth to live young with no eggs involved at any stage of development.

 

Question 17

Consider the following pairs:

Wildlife Naturally              found in

  1. Blue-finned Mahseer   : Cauvery River

  2. Irrawaddy Dolphin       : Chambal River

  3. Rusty-spotted Cat       : Eastern Ghats

Which of the pairs given above are correctly matched?

(a) 1 and 2 only

(b) 2 and 3 only

(c) 1 and 3 only

(d) 1, 2 and 3

Ans: c         

Sub-Theme: Species Habitat

Pair 1 is correctly matched: Blue Finned Mahseer is found in the Mula-Mutha River close to the Indian city of Pune, a part of the Krishna River basin. It is also found in other rivers of the Deccan Plateau and we know the deccan plateau has three principal rivers: the Godavari, the Krishna, and the Cauvery. It is a freshwater fish. It is also known as Deccan Mahseer or Tor Khudree. IUCN Status: Least Concern (LC).

Pair 2 is incorrectly matched: Irrawaddy Dolphin’s habitat extends from the Bay of Bengal to New Guinea and the Philippines. Also found in three rivers namely The Irrawaddy (Myanmar), the Mahakam (Indonesian Borneo) and the Mekong. The highest single lagoon population of dolphins is considered to be found in Chilika Lake. IUCN: Endangered.

Pair 3 is correctly matched: Rusty Spotted Cat was observed in eastern Gujarat’s Gir National Park, in Maharashtra’s Tadoba-Andhari Tiger Reserve and along India’s Eastern Ghats.


 

2018

 

Question 1

Which of the following leaf modifications occur(s) in the desert areas to inhibit water loss?

  1. Hard and waxy leaves

  2. Tiny leaves

  3. Thorns instead of leaves

Select the correct answer using the code given below:

(a) 2 and 3 only

(b) 2 only

(c) 3 only

(d) 1, 2 and 3

Ans: (d)

Sub-Theme: World Biome/Adaptation

Option (d) is the correct answer: Hard and waxy leaves, Tiny leaves and Thorns instead of leaves are the modifications that occur in the desert areas to inhibit water loss.

Desert Biome:

  • Both warm and mid-latitude deserts have a predominance of xerophytic or drought- resistant plants.
  • This comprises cacti, thorny bushes, wiry grasses with extensive roots, and scatted dwarf acacias.
  • In order to gather moisture and search for ground water, the majority of desert shrubs have lengthy roots that are widely
  • Plants with few or no leaves typically have needle-shaped, waxy, leathery, or hairy foliage to reduce water loss through
  • Many kinds of grasses and herbs have tough, thick skins on their seeds that serve as protection when the plants are dormant.

 

Question 2

Which of the following statements best describes “carbon fertilization”?

(a) Increased plant growth due to increased concentration of carbon dioxide in the atmosphere

(b) Increased temperature of Earth due to increased concentration of carbon dioxide in the atmosphere

(c) Increased acidity of oceans as a result of increased concentration of carbon dioxide in the atmosphere

(d) Adaptation of all living beings on Earth to the climate change brought about by the increased concentration of carbon dioxide in the atmosphere

Ans: (a)

Sub-Theme: Agriculture and Environment

Carbon fertilisation

  • It is the artificial enrichment of the atmosphere of greenhouses with carbon dioxide, an essential nutrient for plants and vegetables. It also goes by the name of “Carbon Dioxide Fertilization.”
  • It is used to raise production levels in order to do so both qualitatively and quantitatively.
  • It can be used to grow nearly any form of vegetable (asparagus, celery, lettuce, tomatoes, aubergines, etc.), greenhouse fruit (strawberries), and ornamental plants, and is especially suited for cold climates.

Impact on agriculture:

  • Increase in the photosynthetic rate: It results in an increase in the instantaneous rate of photosynthesis in C3 plants; and increases the transpiration efficiency of the leaf (C3 and C4 plants).
  • Increases the water use efficiency: During high concentration of carbon dioxide, plants maintain narrow openings of leaf surface which protects them from water loss.
  • Faster plant growth enhances food production and allows for the sequestration of more carbon dioxide.
  • Plants distribute a greater proportion of photosynthate to roots under high concentrations of atmospheric carbon
  • The reproductive biomass growth, as well as vegetative biomass growth, is usually increased by elevated carbon dioxide.

 

Question 3

With reference to agricultural soils, consider the following statements:

1. A high content of organic matter in soil drastically reduces its water holding capacity.

2. Soil does not play any role in the sulphur cycle.

3. Irrigation over a period of time can contribute to the salinization of some agricultural lands.

Which of the statements given above is/are correct?

(a) 1 and 2 only

(b) 3 only

(c) 1 and 3 only

(d) 1, 2 and 3

Ans: b

Sub-Theme: Agriculture and Environment

Statement 1 is incorrect: Soil water holding capacity is the amount of water that a given soil can hold for crop use. Soil texture and organic matter are the key components that determine soil water holding capacity. Organic matter influences the physical conditions of a soil in several ways.

Statement 2 is incorrect: Sulphur is one of three nutrients that are cycled between the soil, plant matter and the atmosphere. The sulphur cycle describes the movement of sulphur through the atmosphere, mineral and organic forms, and through living things.

Statement 3 is correct: Salinization is a major problem associated with irrigation, because deposits of salts build up in the soil and can reach levels that are harmful to crops.

NOTE: Again, same story!! Carefully observe the extreme keyword ‘drastically’ and ‘any’ in statements 1 and 2 respectively. Not all but most of the time these extreme statements happen to be wrong.

 

Question 4

With reference to the circumstances in Indian agriculture, the concept of “Conservation Agriculture” assumes significance. Which of the following fall under the Conservation Agriculture?

  1. Avoiding the monoculture practices

  2. Adopting minimum tillage

  3. Avoiding the cultivation of plantation crops

  4. Using crop residues to cover soil surface

  5. Adopting spatial and temporal crop sequencing/ crop rotations

Select the correct answer using the code given below: (a) 1, 3 and 4

(b) 2, 3, 4 and 5

(c) 2, 4 and 5

(d) 1, 2, 3 and 5

Ans: c

Sub-Theme: Agriculture and Environment

Conservation Agriculture (CA):

  • It is a farming system that promotes minimum soil disturbance (i.e. no tillage), maintenance of a permanent soil cover, and diversification of plant species.
  • It enhances biodiversity and natural biological processes above and below the ground surface, which contribute to increased water and nutrient use efficiency and to improved and sustained crop

According to FAO’s website there are three principles of Conservation of Agriculture:

  1. Minimum mechanical soil disturbance: It refers to low disturbance no-tillage and direct seeding. The disturbed area must be less than 15 cm wide or less than 25% of the cropped area (whichever is lower).
  2. Permanent soil organic cover: Three categories are distinguished: 30-60%, >60- 90% and >90% ground cover, measured immediately after the direct seeding operation. Area with less than 30% cover is not considered as CA.
  3. Species diversification: Rotation/ association should involve at least 3 different crop species. The rotation of crops is not only necessary to offer a diverse “diet” to the soil micro organisms, but as they root at different soil depths, they are capable of exploring different soil layers for

 

Question 5

Which of the following is/are the possible consequence/s of heavy sand mining in river beds?

  1. Decreased salinity in the river

  2. Pollution of groundwater

  3. Lowering of the water-table

Select the correct answer using the code given below:

(a) 1 only

(b) 2 and 3 only

(c) 1 and 3 only

(d) 1, 2 and 3

Ans: b

Sub-Theme: Sand Mining (Environmental Issues)

Pollution of groundwater and Lowering of the water-table are consequences of heavy sand mining in river beds.

Statement 1 is incorrect: It may also lead to saline-water intrusion from the nearby sea.

Statement 2 is correct: The amount of suspended particles in the water at the excavation site and downstream increases due to increased riverbed and bank erosion. Aquatic ecosystems and water users may be negatively impacted by suspended particles.

Statement 3 is correct: The groundwater table drops leaving the drinking water wells on the embankments of these rivers dry.

Sand mining

  • Sand is often extracted using an open pit, however it is occasionally mined from inland dunes, oceans, riverbeds, and beaches.
  • Sand mining is the extraction of sand, mainly through an open pit (or sandpit) but sometimes mined from beaches and inland dunes or dredged from ocean and river beds.

Impact of Sand Mining on Rivers

  • Alteration of Rivers
  • Damage River Biodiversity
  • Alter Sediment Budget
  • Lead to poor water quality
  • Worsen water scarcity
  • Threat to Coastal Communities
  • Damage to Public and Private Property

 

Question 6

How is the National Green Tribunal (NGT) different from the Central Pollution Control Board (CPCB)?

1. The NGT has been established by an Act whereas the CPCB has been created by an executive order of the Government.

2. The NGT provides environmental justice and helps reduce the burden of litigation in the higher courts whereas the CPCB promotes cleanliness of streams and wells, and aims to improve the quality of air in the country.

Which of the statements given above is/are correct?

(a) 1 only

(b) 2 only

(c) Both 1 and 2

(d) Neither 1 nor 2

Ans: b

Sub-Theme: Environmental Laws

NATIONAL GREEN TRIBUNAL (NGT) ACT, 2010

  • Objective: For effective & expeditious disposal (within 6 months of appeal) of the environmental cases and to help reduce the burden of litigation in the higher courts.
  • Establishes NGT, which has jurisdiction over all civil disputes addressing important environmental issues.
  • Provides for Enforcement of legal environmental rights, relief & compensation for damages caused.
  • The tribunal’s decision is enforceable as a civil court judgement and is governed by natural justice principles.

CENTRAL POLLUTION CONTROL BOARD (CPCB):

  • The Water (Prevention and Control of Pollution) Act of 1974 authorised the establishment of the CPCB as a statutory
  • It is also entrusted with the powers & functions under the Air (prevention and control of pollution) Act, 1981.
  • It works under the Ministry of Environment, Forest & Climate Change (MoEF&CC).

Statement 1 is incorrect: Both the NGT and CPCB were established by an Act – NGT Act, 2010 and Water (Prevention and Control of Pollution) Act, 1974. Respectively.

Statement 2 is correct: The objective of NGT is to provide effective & expeditious disposal (within 6 months of appeal) of the environmental cases and to help reduce the burden of litigation in the higher courts.

 

Question 7

With reference to the ‘Global Alliance for Climate- Smart Agriculture (GACSA)’, which of the following statements is/are correct?

  1. GACSA is an outcome of the Climate Summit held in Paris in 2015.

  2. Membership of GACSA does not create any binding

  3. India was instrumental in the creation of GACSA.

Select the correct answer using the code given below:

(a) 1 and 3 only

(b) 2 only

(c) 2 and 3 only

(d) 1, 2 and 3

Ans: b

Sub-Theme: Sustainable Agriculture

Global Alliance for Climate-Smart Agriculture (GACSA)

  • GACSA is an inclusive, voluntary and action-oriented multi-stakeholder platform on Climate-Smart Agriculture (CSA).
  • The vision is to improve food security, nutrition and resilience in the face of climate change.
  • GACSA aims to catalyze and help create transformational partnerships to encourage actions that reflect an integrated approach to the three pillars of CSA.

Statement 1 is incorrect: the concept of Climate- Smart Agriculture (CSA) was originally developed by FAO and officially presented and at the Hague Conference on Agriculture, Food Security and Climate Change in 2010, through the paper “Climate-Smart Agriculture: Policies, Practices and Financing for Food Security, Adaptation and Mitigation”. In 2014 an alliance was set up with this issue as its focal point: the GASCA (Global Alliance for Climate-Smart Agriculture).

Statement 2 is correct: Membership in the Alliance does not create any binding obligations and each member individually determines the nature of its participation.

Statement 3 is incorrect: India is just a signatory and had no instrumental role in the creation of GACSA.

 

Question 8

The Partnership for Action on Green Economy (PAGE), a UN mechanism to assist countries transition towards greener and more inclusive economies, emerged at

(a) The Earth Summit on Sustainable Development 2002, Johannesburg

(b) The United Nations Conference on Sustainable Development 2012, Rio de Janeiro

(c) The United Nations Framework Convention on Climate change 2015, Paris

(d) The World Sustainable Development Summit 2016, New Delhi

Ans: b

Sub-Theme: Major Environmental Organizations and their mechanism

Option (b) is correct: Partnership for Action on Green Economy (PAGE) is a direct response to the Rio+20 Declaration, The Future We Want i.e. The United Nations Conference on Sustainable Development 2012, Rio de Janeiro. PAGE serves as a framework to coordinate UN efforts in the green economy and to help nations meet and track the upcoming Sustainable Development Goals, particularly SDG 8: Promote sustained, inclusive and sustainable economic growth, full and productive employment, and decent work for everyone.

PAGE brings together the experts from five UN agencies:

  • United Nations Environment Programme (UNEP or UN Environment),
  • United Nations Development Programme (UNDP),
  • International Labour Organization (ILO),
  • United Nations Industrial Development Organization (UNIDO), and
  • United Nations Institute for Training and Research (UNITAR).

NOTE: UPSC has repeatedly asked such questions directly from the website of various Environmental Organization’s ‘About Us’ pages. This highlights the importance of basic knowledge of various environmental organisations, at least the major ones.

 

Question 9

“Momentum for Change: Climate Neutral Now” is an initiative launched by

(a) The Intergovernmental Panel on Climate Change

(b) The UNEP Secretariat

(c) The UNFCCC Secretariat

(d) The World Meteorological Organization

Ans: c

Sub-Theme: Major Environmental Organizations

Option (c) is correct: The Climate Neutral Now programme, created by the UNFCCC secretariat in 2015, aims to encourage and support a climate-neutral world by the middle of the century, as outlined in the Paris Agreement approved the same year. In order to achieve climate neutrality, the effort calls on businesses, organisations, governments, and individuals to reduce their climate footprint using a straightforward three-step process:

  1. Measure their greenhouse gas emissions;
  2. Reduce them as much as possible;
  3. Compensate those which cannot be avoided by using UN certified emission reductions (CERs).

 

Question 10

In which one of the following States is Pakhui Wildlife Sanctuary located?

(a) Arunachal Pradesh

(b) Manipur

(c) Meghalaya

(d) Nagaland

Ans: a        

Sub-Theme: National Parks/Biosphere Reserve

Option (a) is the correct answer: Another name of Pakke Tiger Reserve is “Pakhui Tiger Reserve”, In the northeastern Indian state of Arunachal Pradesh’s Pakke Kessang district, there is a Project Tiger reserve. The Eastern Himalayas Biodiversity Hotspot includes it. The Pakke Wildlife Sanctuary is located in Arunachal Pradesh in the undulating, hilly Eastern Himalayan foothills. In the west, north, and east, it is bordered by the Bhareli or Kameng River, and by the Pakke River. It is renowned for its incredible encounters with four local hornbill species. For its Hornbill Nest Adoption Program, this Tiger Reserve received the 2016 India Biodiversity Award in the category of “Conservation of vulnerable species.”

 

Question 11

Why is a plant called Prosopis juliflora often mentioned in the news?

(a) Its extract is widely used in cosmetics.

(b) It tends to reduce the biodiversity in the area in which it grows.

(c) Its extract is used in the synthesis of pesticides.

(d) None of these

Ans: b        

Sub-Theme: Invasive Species

Option (b) is the correct answer: Prosopis Juliflora is a shrub or small tree in the family Fabaceae. It is native to Mexico, South America and the Caribbean. It was initially introduced in India during colonial times, since then it has become an invasive species. It’s an aggressive coloniser, a common weed of wastelands, scrublands and degraded forests.

 

2017

 

Question 1

The term ‘M-STrIPES’ is sometimes seen in the news in the context of

(a) Captive breeding of Wild Fauna

(b) Maintenance of Tiger Reserves

(c) Indigenous Satellite Navigation System

(d) Security of National Highways

Ans: b

Sub-Theme: Technology in wildlife protection Option (b) is the correct answer: M-STrIPES stands for Monitoring system for Tigers –Intensive Protection and Ecological Status.

M-STrIPES:

  • Launched in 2010 by NTCA and WII across Indian tiger reserves.
  • It is a software-based monitoring system, designed to assist wildlife protection.
  • The system consists of two components: Field based methods for ecological monitoring, law enforcement, recording wildlife offences, and patrolling.
  • A specialised GIS application for reporting, analysis, and data storage.
  • Forest guards are required to use a GPS to track their movements as part of M-STrIPES, in addition to entering their observations on site- specific data sheets.

 

Question 2

Due to some reasons, if there is a huge fall in the population of species of butterflies, what could be its likely consequence/consequences?

  1. Pollination of some plants could be adversely

  2. There could be a drastic increase in the fungal infections of some cultivated plants.

  3. It could lead to a fall in the population of some species of wasps, spiders and birds.

Select the correct using the code given below:

(a) 1 only

(b) 2 and 3 only

(c) 1 and 3 only

(d) 1, 2 and 3

Ans: c

Sub-Theme: Biotic Interaction

Pollination:

  • Transfer of pollen grains (shed from the anther) to the stigma of a pistil is termed pollination. Flowering plants have evolved an amazing array of adaptations to achieve
  • Kinds of Pollination: Depending on the source of pollen, pollination can be divided into three types –
    1. Autogamy
    2. Geitonogamy
    3. Xenogamy

Statement 1 is correct: This is basic knowledge that Bees, butterflies and other biotic agents play a vital role in the pollination of plants and the production of crops by transporting pollen grains from one place to another.

Statement 2 is incorrect: There is no credible source or research behind this and moreover the extreme keyword ‘drastic’ in the statement makes it more absurd and irrelevant.

Statement 3 is correct: These are some of the common predators of butterflies: wasps, ants, parasitic flies, birds, snakes, toads, rats etc. The decline in the butterfly population would therefore adversely affect the food chain

NOTE: if we observe statement 2 carefully, the examiner used the word “drastic increase” which is extreme and absolute. It also connotes all of sudden change in the present situation. So, elimination of statement 2 left us with option (a) and option (c). Now it is common understanding that decline in butterfly population would adversely affect the food chain, so the reduction in other dependent species such as wasps, spiders and birds. So, statement 3 is correct.

 

Question 3

Which of the following statements can help in water conservation in agriculture?

  1. Reduced or zero tillage of the land

  2. Applying gypsum before irrigating the field

  3. Allowing crop residue to remain in the field

Select the correct answer using the code given below:

(a) 1 and 2 only

(b) 3 only

(c) 1 and 3 only

(d) 1, 2 and 3

Ans: c

Sub-Theme: Agriculture and Environment

  • Statement 1 is correct: Conservation Tillage includes no-till farming or zero tillage is a way of growing crops or pasture from year to year without disturbing the soil through tillage. Zero tillage helps in soil moisture conservation.
  • Statement 2 is incorrect: Though, Gypsum helps in dissolution of the salt clay pan found in soils that hinder the percolation of water and helps in water conservation. But the official answer key published by UPSC shows option (c) as the correct answer. Thus, going by the UPSC key statement 2 is wrong.
  • Statement 3 is correct: Crop residues act as mulches and reduce evaporation losses and protect the soil from direct impact of raindrops, thus controlling crusting and sealing processes.

 

Question 4

Biological Oxygen Demand (BOD) is a standard criterion for:

(a) Measuring oxygen levels in blood

(b) Computing oxygen levels in forest ecosystems

(c) Pollution assay in aquatic ecosystems

(d) Assessing oxygen levels in high altitude regions

Ans: c

Sub-Theme: Water Pollution

Option (c) is the correct answer: Pollution assay in aquatic ecosystems is based on Biological Oxygen Demand (BOD) criterion.

  • Biochemical Oxygen Demand (BOD) is a measure of the amount of organic waste that pollutes water.
  • BOD is the amount of dissolved oxygen needed by bacteria in decomposing the organic wastes present in water. It is measured in milligrammes per litre of water of oxygen.
  • The higher value of BOD indicates low DO content of water.
  • BOD is not a reliable method for assessing water pollution because it only includes biodegradable elements.

NOTE: This is an easy and straight question from NCERT Class 11 Chemistry Chapter 14 (Environmental Chemistry), though the non- science graduates may not find interest in reading class 11 and 12 science NCERTs this particular chapter is very important for the environment and ecology.

 

Question 5

In the context of solving pollution problems, what is/are the advantage/advantages of bioremediation technique?

1. It is a technique for cleaning up pollution by enhancing the same biodegradation process that occurs in nature.

2.  Any contaminant with heavy metals such as cadmium and lead can be readily and completely treated by bioremediation using

3. Genetic engineering can be used to create microorganisms specifically designed for

Select the correct answer using the code given below:

(a) 1 only

(b) 2 and 3 only

(c) 1 and 3 only

(d) 1, 2 and 3

Ans: (c)

Sub-Theme: Eco-Engineering

Benefits of Bioremediation:

  • It minimises damage to ecosystems by just using natural methods.
  • In order to remove toxins from soil and groundwater, bioremediation frequently takes place underground, where pumped-in nutrients and bacteria can be used.
  • As a result, compared to other cleanup techniques, bioremediation causes less disruption to the neighbourhood
  • A minor change or addition designed to the environment, such as molasses, vegetable oil, or simple air optimises conditions for microbes to flourish, thereby accelerating the completion of the bioremediation
  • It produces comparatively few harmful byproducts (mainly due to the fact that contaminants and pollutants are converted into water and harmless gases like carbon dioxide).
  • Since it doesn’t require a lot of labour or expensive equipment, it is less expensive than most cleanup techniques.

Statement 1 is correct: Bioremediation uses naturally occurring organisms to break down hazardous substances into less toxic or non-toxic substances.

Statement 2 is incorrect: Not all contaminants are easily treated by bioremediation using microorganisms. For eg. Heavy metals such as cadmium and lead are not readily absorbed or captured by microorganisms.

Statement 3 is correct: The use of genetic engineering to create organisms specifically designed for bioremediation has great potential. For e.g. Alcanivorax, a bacteria with oil-eating abilities, can be used to create species that are much more capable of cleaning oil spills.

NOTE: UPSC asks standard conceptual questions on Eco-engineering every year. Bioremediation, biofuels, and genetic engineering are some such standard concepts. Hence, conceptual clarity of this particular topic is very important. Also, always be cautious of extreme statements and words such as ‘readily’ (quickly) and ‘completely’.

 

Question 6

It is possible to produce algae-based biofuels, but what is/are the likely limitation(s) of developing countries in promoting this industry?

1. Production of algae-based biofuels is possible in seas only and not on continents.

2. Setting up and engineering the algae-based biofuels production requires high level of expertise/technology until the construction is

3. Economically viable production necessitates the setting up of large-scale facilities which may raise ecological and social concerns.

Select the correct answer using the code given below:

(a) 1 and 2 only

(b) 2 and 3 only

(c) 3 only

(d) 1, 2 and 3

Ans: b

Sub-Theme: Eco-Engineering

Biofuels

  • A biofuel is any hydrocarbon fuel that is created in a short amount of time—days, weeks, or even months—from organic matter (living or once-living material).
  • Biofuels may be solid, liquid or gaseous in nature:
    • Solid: Wood, dried plant material, and manure
    • Liquid: Bioethanol and Biodiesel
    • Gaseous: Biogas

Statement 1 is incorrect: Algaculture (farming algae) can be initiated on land unsuitable for agriculture or saline water or wastewater.

Statement 2 is correct: Producing algae for biofuels (excluding simple seaweed production) requires significant capital investments, which may be a significant barrier in developing countries that have a weak investment climate.

Statement 3 is correct: Using the land allotted to food crops to algal biofuel reduces the amount of food available for humans, resulting in an increased cost for both the food and the fuel produced. This is an ecological as well as economic concern.

 

Question 7

In the context of mitigating the impending global warming due to anthropogenic emissions of carbon dioxide, which of the following can be potential sites for carbon sequestration?

  1. Abandoned and uneconomic coal seams

  2. Depleted oil and gas reservoirs

  3. Subterranean deep saline formations

Select the correct answer using the code given below:

(a) 1 and 2 only

(b) 3 only

(c) 1 and 3 only

(d) 1, 2 and 3

Ans: d

Sub-Theme: Global warming mitigation/Carbon sequestration

Option (d) is the correct answer: All are potential sites for carbon sequestration.

Carbon Sequestration:

  • Carbon sequestration is a process to capture and store CO2 to curb global It is captured from the air, industries or power stations and stored permanently underground.

Types of CO2 Sequestration:

  • Terrestrial Carbon Sequestration: It is the process through which CO2 from the atmosphere is absorbed by trees and plants through photosynthesis and stored as carbon in soils and biomass (tree trunks, branches, foliage, and roots)
  • Geologic Carbon Sequestration: CO2 can be stored, including oil reservoirs, gas reservoirs, unmineable coal seams, saline formations and shale formations with high organic content.
  • Ocean Carbon Sequestration: Oceans absorb, release and store large amounts of CO2 from the atmosphere.

Statement 1 is correct: Uneconomic coal seams can be used to store CO2 because the CO2 molecules attach to the surface of coal.

Statement 2 is correct: CO2 is sometimes injected into declining oil fields to increase oil recovery.

Statement 3 is correct: Other potential sites are gas reservoirs, saline formations and shale formations with high organic content.

NOTE: UPSC has asked about “potential sites”. If we see options, all the given options could be potential sites for carbon sequestration. It is always advisable to stay focused while reading questions and try to identify such keywords from the given questions.

 

Question 8

In India, if a species of tortoise is declared protected under Schedule I of the Wildlife (Protection) Act, 1972, what does it imply?

(a) It enjoys the same level of protection as the tiger.

(b) It no longer exists in the wild, a few individuals are under captive protection; and not it is impossible to prevent its extinction.

(c) It is endemic to a particular region of India.

(d) Both (b) and (c) stated above are correct in this context.

Ans: a

Sub-Theme: Environmental Laws

Schedule I covers endangered species that need rigorous protection. The species are granted protection from poaching, killing, trading etc. It enjoys the same level of protection as the Bengal tiger.

Wildlife (Protection) Act, 1972:

  • The Act was passed in order to safeguard various plant and animal species.
  • Except for the State of Jammu & Kashmir, it covers the entirety of India.
  • Prior to this legislation, India had only five designated national parks.
  • At present, there are 101 National Parks in

 

Question 9

According to the Wildlife (Protection) Act, 1972, which of the following animals cannot be hunted by any person except under some provisions provided by law?

  1. Gharial

  2. Indian wild ass

  3. Wild buffalo

Select the correct answer using the code given below:

(a) 1 only

(b) 2 and 3 only

(c) 1 and 3 only

(d) 1, 2 and 3

Ans: d

Sub-Theme: Environmental Laws

Option (d) is the correct answer: All are Scheduled I species.

“Schedules and Species under the Act”

Schedules Parameters Species
 

 

 

 

Schedule I

•   It covers endangered species that need rigorous protection. The species are granted protection from poaching, killing, trading etc.

•   A person is liable to the harshest penalties for violation of the law under this Schedule.

•   Species under this Schedule are prohibited to be hunted throughout India, except under threat to human life or in case of a disease that is beyond recovery.

The Black Buck; Bengal Tiger; Clouded Leopard; Snow Leopard; Swamp Deer; Himalayan Bear; Asiatic Cheetah; Kashmiri Stag; Fishing Cat; Lion-tailed Macaque; Musk Deer; Rhinoceros; Brow Antlered Deer; Wild Buffalo; Chinkara (Indian Gazelle); Capped Langur; Golden Langur; Hoolock Gibbon; Gharials; Dugong; Great Indian Bustard; Indian Wild Ass; etc.
 

 

 

Schedule II

•   Animals under this list are also accorded high protection with the prohibition on their trade.

•   They cannot be hunted except under threat to human life or if they are suffering from a disease/ disorder that goes beyond recovery.

Assamese Macaque, Pig Tailed Macaque, Stump Tailed Macaque, Bengal Hanuman langur, Himalayan Black Bear, Himalayan Newt/ Salamander, Jackal, Flying Squirrel, Giant Squirrel, Sperm Whale, Indian Cobra, King Cobra
 

 

 

 

Schedule III & IV

•   Species that are not endangered are included under Schedule III and IV.

•   This includes protected species with hunting prohibited but the penalty for any violation is less compared to the first two schedule.

•   These animals can be hunted.

 

 

Schedule III

Chital (spotted deer), Bharal (blue

sheep), Hyena, Nilgai, Sambhar (deer), Sponges

 

Schedule IV

Flamingo, Hares, Falcons, Kingfishers, Magpie, Horseshoes Crabs
 

Schedule V

•   This schedule contains animals that are considered as vermin (small wild animals that carry disease and destroy plants and food). Common Crows, Fruit Bats, Rats, Mice
 

 

 

Schedule VI

•   It provides for regulation in cultivation of a specified plant and restricts its possession, sale and transportation.

•   Both cultivation and trade of specified plants can only be carried out with prior permission of competent authority.

Beddomes’ cycad (Native to India), Blue Vanda (Blue Orchid), Red Vanda (Red Orchid), Kuth (Saussurea lappa), Slipper orchids (Paphiopedilum spp.), Pitcher plant (Nepenthes khasiana)

 

Question 10

With reference to ‘Global Climate Change Alliance’, which of the following statements is/are correct?

1. It is an initiative of the European union.

2. It provides technical and financial support to targeted developing countries to integrate climate change into their development policies and budgets.

3. It is coordinated by World Resources Institute (WRI) and World Business Council for Sustainable Development (WBCSD).

Select the correct answer using the code given below:

(a) 1 and 2 only

(b) 3 only

(c) 2 and 3 only

(d) 1, 2 and 3

Ans: a

Sub-Theme: Global Climate Finance Architecture

Global Climate Change Alliance (GCCA)

  • The GCCA operates through the existing national and international channels for political cooperation and dialogue set up by the European Commission.
  • By becoming the main project of the Global Climate Change Alliance Plus (GCCA+), GCCA entered a new phase in 2015.
  • This new phase is marked by new features and a strategic focus on promoting the 2015 Paris Agreement on Climate Change, the 2030 Agenda for Sustainable Development, and its Sustainable Development Goals.

Statement 1 is correct: The Global Climate Change Alliance (GCCA) is an initiative of the European Union.

Statement 2 is correct: It was launched to strengthen dialogue and cooperation on climate change between the European Union (EU) and developing countries most vulnerable to climate change, in particular, Least Developed Countries (LDCs) and Small Island Developing States (SIDS), which are hardest hit by the adverse effects of climate change.

Statement 3 is incorrect: There is no mention of WRI and WBCSD on the official page of GCCA, even though a lot of other institutions are mentioned like FAO, UNDP etc.

 

Question 11

Consider the following statements:

1. Climate and Clean Air Coalition (CCAC) to Reduce Short Lived Climate Pollutants is a unique initiative of G2O group of countries.

2. The CCAC focuses on methane, black carbon and hydrofluorocarbons.

Which of the statements given above is/are correct?

(a) 1 only

(b) 2 only

(c) Both 1 and 2

(d) Neither 1 nor 2

Ans: b

Sub-Theme: Global Climate Finance Architecture

Climate and Clean Air Coalition (CCAC):

  • The Climate and Clean Air Coalition is a voluntary alliance of governments, intergovernmental organisations, corporations, academic institutions, and organisations from civil society dedicated to enhancing air quality and defending the environment by taking steps to reduce short-lived climate pollutants.

Statement 1 is incorrect: The United Nations Environment Programme (UNEP), the governments of Bangladesh, Canada, Ghana, Mexico, Sweden, and the United States joined forces in 2012 to launch initiatives to handle short-lived climate pollutants as an urgent and collective concern.

Statement 2 is correct: Short-Lived Climate Pollutants include black carbon, methane, tropospheric ozone, and hydro-fluorocarbons.

NOTE: UPSC usually puts such “about us” page questions not with the primary desire of getting correct answers from the aspirants, but rather to eliminate candidates. Now with a calm and composed mind we can make some intelligent guesses to reach the correct answer. Though it’s a bit risky but still if we consider ‘unique’ as an extreme word with G20 then the first statement becomes wrong. And the second statement looks like a factual one with no extreme word like ‘only’, not all but in most cases such type of question ought to be right. Please use due diligence while attempting this method.

 

Question 12

Consider the following statements in respect of Trade Related Analysis of Fauna and Flora in Commerce (TRAFFIC):

1. TRAFFIC is a bureau under the United Nations Environment Programme (UNEP).

2. The mission of TRAFFIC is to ensure that trade in wild plants and animals is not a threat to the conservation of nature.

Which of the above statements is/are correct?

(a) 1 only

(b) 2 only

(c) Both 1 and 2

(d) Neither 1 nor 2

Ans: b

Sub-Theme: Conservation Agencies/Schemes

/Initiatives

Statement 1 is incorrect: The TRAFFIC, the Wildlife Trade Monitoring Network, is a non- governmental organisation which works to monitor the wildlife trade in the context of both biodiversity conservation and sustainable development. Headquarters: Cambridge, United Kingdom. The World Wildlife Fund (WWF) and the International Union for Conservation of Nature collaborate on this programme (IUCN).

Statement 2 is correct: It is working globally on the Wildlife trade monitoring network. It specialises in investigating and analysing wildlife trade trends, patterns, impacts and drivers to provide the leading knowledge base on trade in wild animals and plants.

TRAFFIC and India:

  • Since 1991, TRAFFIC has functioned as a Programme Division of WWF-India, with headquarters in New Delhi.
  • Since then, it has collaborated closely with the federal, state, and local governments as well as a number of organisations to research, track, and influence action to stop the illicit wildlife trafficking.

NOTE: Conservation agencies/schemes/ initiatives esp. WWF is the favourite topic of UPSC, one example being Earth hour asked in PYQ. Hence, try to cover WWF, TRAFFIC and related websites comprehensively to answer such questions. Reading Shankar will help you to solve this question without much trouble.

 

Question 13

From the ecological point of view, which one of the following assumes importance in being a good link between the Eastern Ghats and the Western Ghats?

(a) Sathyamangalam Tiger Reserve

(b) Nallamala Forest

(c) Nagarhole National Park

(d) Seshachalam Biosphere Reserve

Ans: a

Sub-Theme: National Parks/Biosphere Reserve

Option (a) is the correct answer: Sathyaman- galam Tiger Reserve (STR) is situated where the Western and Eastern Ghats converge. This region is home to a sizable tiger population and is adjacent to other landscapes dedicated to protecting tigers, including BRT, Bandipur, Mudumalai, and Nagarahole. The Nilgiri-Eastern Ghats Elephant Reserve includes this area as well. Due to its position, it contains a wide variety of vegetation and animals.

NOTE: UPSC has the habit of picking up options  from  questions  in  its  previous year’s papers and asking new questions after improvising them. Nallamala Forest, Sathyamangalam  Tiger  Reserve,  etc.  are some examples. Hence, it underlines the importance of solving and analysing PYQs.

 

Question 14

If you want to see gharials in their natural habitat, which one of the following is the best place to visit?

(a) Bhitarkanika Mangroves

(b) Chambal River

(c) Pulicat Lake

(d) Deepor Beel

Ans: b          

Sub-Theme: Species (Fauna)

Option (a) is incorrect: Gharials inhabit deep freshwater habitats, not both freshwater and saltwater habitats as inhabited by crocodile species. It is the home of saltwater crocodiles.

Option (b) is correct:   The most important surviving populations of Gharials are within four tributaries of the Ganges River – Girwa, Son, and Chambal Rivers in India and the Rapti- Narayani River in Nepal. The most significant breeding population is within the Chambal River, which spans the Indian states of Uttar Pradesh, Rajasthan, and Madhya Pradesh.

Option (c) is incorrect: Pulicat Lake is the second largest brackish water lake or lagoon in India after Chilika Lake. It is located on the border of Andhra Pradesh and Tamil Nadu with over 96% of it in Andhra Pradesh and 4% in Tamil Nadu situated on the Coromandel Coast in South India. The lake encompasses Pulicat Lake Bird Sanctuary. The Buckingham Canal, a navigation channel, is part of the lagoon on its western side.

Option (d) is incorrect: Deepor Beel is one of the largest freshwater lakes in Assam and the State’s only Ramsar site besides being an Important Bird Area by Birdlife International. Till date no such credible information has been found in terms of Gharial population in Deepor beel.

 

2016

 

Question 1

‘Gadgil Committee Report’ and ‘Kasturirangan Committee Report’, sometimes seen in the news, are related to

(a) constitutional reforms

(b) Ganga Action Plan

(c) linking of rivers

(d) protection of Western Ghats

Ans: d

Sub-Theme: Disaster Management

Option (d) is the correct answer: ‘Gadgil Committee Report’ and ‘Kasturirangan Committee Report’, are related to protection of Western Ghats.

GADGIL COMMITTEE REPORT, 2011:

  • Eco-sensitive areas: The Western Ghats Ecology Expert Panel (WGEEP), chaired by Prof. Madhav Gadgil, was established by the Ministry of Environment & Forests in
  • Objective: It was constituted to primarily demarcate ecologically sensitive areas in the Western Ghats and recommended measures for the management of these ecologically sensitive areas.

KASTURIRANGAN COMMITTEE REPORT, 2013:

  • The Kasturirangan committee was constituted to examine the WGEEP report.

 

Question 2

Which of the following best describe/ describes the aim of the ‘Green India Mission’ of the Government of India?

1. Incorporating environmental benefits and costs into the Union and State Budgets thereby implementing the ‘green accounting’

2. Launching the second green revolution to enhance agricultural output so as to ensure food security to one and all in the future

3. Restoring and enhancing forest cover and responding to climate change by a combination of adaptation and mitigation measures

Select the correct answer using the code given below.

(a) 1 only

(b) 2 and 3 only

(c) 3 only

(d) 1, 2 and 3

Ans: c

Sub-Theme: Conservation/Environmental pollution

Green India Mission:

  • The National Action Plan on Climate Change (NAPCC) launched eight missions, and GIM is one of them.
  • Aiming to maintain, restore, and improve India’s dwindling forest cover and combating climate change through a combination of adaptation and mitigation strategies, GIM was introduced in February 2014.

Objectives of the Mission:

  • To protect, restore and enhance India’s falling forest cover.
  • To respond to climate change through a combination of adaptation as well as mitigation measures.
  • To increase forest-based livelihood
  • To enhance annual Carbon sequestration by 50 to 60 million tonnes in the year

Statement 1 and 2 are incorrect: Both the statements are irrelevant to Green India Mission.

Statement 3 is correct: The mission intends to increase forest/tree cover; improve/enhance ecosystem services and increase the forest-based livelihood income of about 3 million households.

 

Question 3

In the cities of our country, which among the following atmospheric gases are normally considered in calculating the value of the Air Quality Index?

  1. Carbon dioxide

  2. Carbon monoxide

  3. Nitrogen dioxide

  4. Sulfur dioxide

  5. Methane

Select the correct answer using the code given below.

(a) 1, 2 and 3 only

(b) 2, 3 and 4 only

(c) 1, 4 and 5 only

(d) 1, 2, 3, 4 and 5

Ans: b

Sub-Theme: Air Pollution

Option (b) is the correct answer: CO2 , CO and SO2 are normally considered in calculating the value of the Air Quality Index.

  • The air quality index, or AQI, is a statistic that is used to inform the public of how filthy the air is now or is expected to become.
  • A growing proportion of the population is projected to endure worsening health impacts as AQI rises.
  • According to different national air quality standards, various countries have their own air quality indices.
  • The AQI is most frequently used to describe ozone levels at ground level.
  • The five contaminants that endanger human health can be represented by the AQI,
    • Ground-level Ozone or O3
    • Particulate Matter (soot and dust) or PM
    • Carbon Monoxide or CO
    • Sulphur Dioxide or SO2 and
    • Nitrogen Dioxide or NO2
    • All except Carbon dioxide are taken into consideration while calculating the AQI.

 

Question 4

The term ‘Intended Nationally Determined Contributions’ is sometimes seen in the news in the context of

(a) Pledges made by the European countries to rehabilitate refugees from the war-affected Middle East

(b) Plan of action outlined by the countries of the world to combat climate change

(c) Capital contributed by the member countries in the establishment of Asian Infrastructure Investment Bank

(d) Plan of action outlined by the countries of the world regarding Sustainable Development Goals

Ans: b

Sub-Theme: Climate Change/Global Warming

Intended Nationally Determined Contributions: Plan of action outlined by the countries of the world to Combat Climate Change

  • Intended Nationally Determined Contributions (INDC) are (intended) reductions in greenhouse gas emissions under the United Nations Framework Convention on Climate Change (UNFCCC).
  • NDCs are at the heart of the Paris Agreement and embody efforts by each country to reduce national emissions and adapt to the impacts of climate change.
  • The UNFCCC, in its Nationally Determined Contributions (NDC) Synthesis Report, has called for more ambitious climate action plans by the countries in order to achieve the Paris Agreement target of containing global temperature rise to 2°C by the end of the century (ideally it is 1.5°C).
  • The NDC Synthesis Report covers submissions up to 31st December 2020 and includes new or updated NDCs by 75 Parties, which represent approximately 30% of global greenhouse gas emissions.
  • The report was sought ahead of the 26th session of the Conference of the Parties (COP 26) to the UNFCCC which is scheduled to take place from 1st- 12th November 2021, in Glasgow, UK.
  • Each NDC reflects the country’s ambition, taking into account its domestic circumstances and capabilities.

 

Question 5

What is ‘Greenhouse Gas Protocol’?

(a) It is an international accounting tool for government and business leaders to understand, quantify and manage greenhouse gas emissions.

(b) It is an initiative of the United Nations to offer financial incentives to developing countries to reduce greenhouse gas emissions and to adopt eco-friendly technologies

(c) It is an inter-governmental agreement ratified by all the member countries of the United Nations to reduce greenhouse gas emissions to specified levels by the year 2022

(d) It is one of the multilateral REDD+ initiatives hosted by the World Bank

Ans: a

Sub-Theme: Global Warming

Option (a) is the correct answer: ‘Greenhouse Gas Protocol’ is an International Accounting Tool for Government and Business Leaders

  • GHG Protocol is developing standards, tools and online training that helps countries to measure and manage greenhouse gas (GHG) emissions from private and public sector operations, value chains and mitigation

GHG Protocol:

  • To assist nations, cities, and businesses in monitoring their progress toward their climate goals, GHG Protocol is creating standards, tools, and online training.
  • GHG Protocol establishes frameworks to measure and manage greenhouse gas (GHG) emissions from private and public sector operations, value chains and mitigation
  • The GHG Protocol arose when the World Resources Institute (WRI) and World Business Council for Sustainable Development (WBCSD) recognized the need for an international standard for corporate GHG accounting and reporting in the late

 

Question 6

With reference to the Agreement at the UNFCCC Meeting in Paris in 2015, which of the following statements is/are correct?

1. The Agreement was signed by all the member countries of the UN and it will go into effect in 2017.

2. The Agreement aims to limit greenhouse gas emissions so that the rise in average global temperature by the end of this century does not exceed 2 degrees C or even 1.5 degrees C above pre-industrial levels.

3. Developed countries acknowledged their historical responsibility in global warming and committed to donate $1000 billion a year from 2020 to help developing countries to cope with climate change.

Select the correct answer using the code given below.

(a) 1 and 3 only

(b) 2 only

(c) 2 and 3 only

(d) 1, 2 and 3

Ans: b

Sub-Theme: Summits and Conventions

Statement 1 is incorrect: It was only “agreed” by 195 nations in Paris at the conference. The agreement will enter into force only after 55 countries that account for at least 55% of global emissions have deposited their instruments of ratification.

Statement 2 is correct: Paris Summit COP21 aims to limit global warming to well below 2, preferably to 1.5 degrees Celsius, compared to pre-industrial levels. It requires all parties to put forward their Nationally Determined Contributions (NDCs) which are voluntary in nature.

Statement 3 is incorrect: Developed countries will work to define a clear roadmap on ratcheting up climate finance to USD 100 billion (not USD 1000 billion) by 2020.

Paris Summit COP21:

  • Legally binding International Treaty on Climate Change.
  • It was adopted by 196 Parties at COP 21 in Paris, on 12 December 2015 & entered into force on 4 November 2016.
  • Aim: to limit global warming to well below 2, preferably to 1.5 degrees Celsius, compared to pre-industrial levels.
  • It requires all parties to put forward their Nationally Determined Contributions (NDCs) which are voluntary in nature.

NOTE: Following basic information about COP21 from any newspaper/monthly magazine was enough to solve this rather difficult question. Observe statement 3 carefully, the examiner has just made his efforts to mention “USD100Bn as USD1000Bn” to make the statement incorrect and confuse aspirants. If we eliminate option 3, we get the correct answer i.e. only 2.

  • Itn 2017, UPSC asked question about the spread of the Himalayas, Western Ghats and Pulicat lake with the similar trick of changing/interchanging
  • In 2016, UPSC asked question on “Astrosat” with the similar trick of changing/interchanging numbers.

The key takeaway from the above question is to read numbers very carefully and cautiously.

 

Question 7

Consider the following statements:

  1. The Sustainable Development Goals were first proposed in 1972 by a global think tank called the ‘Club of Rome’.

  2. The Sustainable Development Goals have to be achieved by 2030.

Which of the statements given above is/are correct?

(a) 1 only

(b) 2 only

(c) Both 1 and 2

(d) Neither 1 nor 2

Ans: b         

Sub-Theme: Sustainable Development Goals

The Sustainable Development Goals (SDGs) are a collection of 17 global goals designed to be a “blueprint to achieve a better and more sustainable future for all”

The SDGs are a part of UN Resolution 70/1, the 2030 Agenda, which was adopted by the United Nations General Assembly in 2015.

They are known as “Transforming our world: the 2030 Agenda for Sustainable Development” in official documents.

Statement 1 is incorrect: In 1972, the Club of Rome’s first major Report, ‘The Limits to Growth’ was published.

Statement 2 is correct: In 2015, the United Nations General Assembly approved 17 goals with 169 associated targets. They became effective from 1/1/2016 and were to be achieved by 2030.

 

Question 8

What is/are the importance/ importances of the ‘United Convention to Combat Desertification’?

1. It aims to promote effective action through innovative national programmes and supportive international partnerships

2. It has a special/particular focus on South Asia and North Africa regions, and its Secretariat facilitates the allocation of major portions of financial resources to these regions.

3. It is committed to a bottom-up approach, encouraging the participation of local people in combating the desertification.

Select the correct answer using the code given below.

(a) 1 only

(b) 2 and 3 only

(c) 1 and 3 only

(d) 1, 2 and 3

Ans: c        

Sub-Theme: Major Conventions

UNITED NATIONS CONVENTION TO COMBAT

DESERTIFICATION (UNCCD):

  • Established in 1994.
  • It is the sole legally binding international agreement   linking  environment and development to sustainable land management.
  • Aim: Its 197 Parties aim, through partnerships, to implement the Convention and achieve the Sustainable Development Goals. The end goal is to protect land from over-use and drought, so it can continue to provide food, water and energy.

Statement 1 is correct: Its 197 Parties aim, through partnerships, to implement the Convention and achieve the Sustainable Development Goals. The end goal is to protect the land from overuse and drought, so it can continue to provide food, water and energy.

Statement 2 is incorrect: The Convention addresses specifically the arid, semi-arid and dry sub-humid areas, known as the drylands, where some of the most vulnerable ecosystems and peoples can be found.

Statement 3 is correct: The UNCCD is particularly committed to a bottom-up approach, encouraging the participation of local people in combating desertification and land degradation.

 

Question 9

Consider the following pairs:

Terms sometimes seen in the news : Their origin

1. Annex-I Countries                        : Cartagena Protocol

2. Certified Emissions Reductions    : Nagoya Protocol

3. Clean Development Mechanisms : Kyoto Protocol

Which of the pairs given above is/are correctly matched?

(a) 1 and 2 only

(b) 2 and 3 only

(c) 3 only

(d) 1, 2 and 3

Ans: c

Sub-Theme: Major Conventions

Classification of Parties and their commitments under Kyoto Protocol:

 

Annex I

Developed countries [US, UK, Russia etc.] + Economies in transition (EIT) [Ukraine, Turkey, some eastern European countries etc.]
 

 

 

Annex II

Developed countries (Annex II is a subset of Annex I).

Required to provide financial and technical support to the EITs and developing countries to assist them in reducing their greenhouse gas emissions.

 

 

 

 

Annex B

Annex I Parties with first or second- round Kyoto greenhouse gas emissions targets.

The first-round targets apply over the years 2008–2012 and the second- round Kyoto targets, apply from 2013 to 2020.

Compulsory binding targets to reduce GHG emissions.

 

 

Non-Annex I

Parties to the UNFCCC are not listed in Annex I of the Convention (mostly low-income developing countries).

No binding targets to reduce GHG emissions.

 

LDCs

Least-developed countries

No binding targets to reduce GHG emissions.

Options 1 and 2 are incorrect: Terms in these options are associated with the Kyoto Protocol.

Option 3 is correct: Clean Development Mechanism is associated with the Kyoto Protocol.

 

Question 10

With reference to an initiative called ‘The Economics of Ecosystems and Biodiversity (TEEB)’, which of the following statements is/are correct?

1. It is an initiative hosted by UNEP, IMF and World Economic Forum.

2. It is a global initiative that focuses on drawing attention to the economic benefits of  biodiversity.

3. It presents an approach that can help decision- makers recognize, demonstrate and capture the value of ecosystems and biodiversity.

Select the correct answer using the code given below.

(a) 1 and 2 only

(b) 3 only

(c) 2 and 3 only

(d) 1, 2 and 3

Ans: c

Sub-Theme: Major Environmental Conventions/ Environmental Policies and Initiatives

The Economics of Ecosystems and Biodiversity (TEEB)

  • Launched by Germany and the European Commission in 2007
  • Study led by Pavan Sukhdev
  • It is a global effort to raise awareness of the economic advantages of biodiversity.
  • Objective: To raise attention to the rising costs associated with biodiversity loss and ecosystem degradation and to bring together experts in science, economics, and policy to facilitate actionable solutions.

Statement 1 is incorrect: It is an international initiative, launched by Germany and the European Commission.

Statement 2 is correct: It is an international initiative to draw attention to the global economic benefits of biodiversity.

Statement 3 is correct: Its objective is to highlight the growing cost of biodiversity loss and ecosystem degradation and to draw together expertise from the fields of science, economics and policy to enable practical actions.

 

 

Question 11

Which of the following statements is/are correct?

Proper design and effective implementation of UN- REDD+ Programme can significantly contribute to:

  1. protection of biodiversity

  2. resilience of forest ecosystems

  3. poverty reduction

Select the correct answer using the code given below.

(a) 1 and 2 only

(b) 3 only

(c) 2 and 3 only

(d) 1, 2 and 3

Ans: d

Sub-Theme: Environmental Conventions/ Environmental Policies and Initiatives

The UN-REDD Programme has identified the following three topics for supporting national REDD+ governance structures:

  • Country-led multi-stakeholder governance assessments that contribute to the development and implementation of national REDD+ strategies (for tackling forest degradation and biodiversity loss) and nationally owned monitoring frameworks
  • Social standards, including poverty and
  • Land and/or carbon tenure and the role of REDD+ within national land use strategies

UN REDD+

  • Reducing Emissions from Deforestation and Forest Degradation (REDD+) is a mechanism developed by Parties of the
  • In order to provide incentives for developing countries to cut emissions from forested lands and invest in low-carbon paths, it sets a financial value for the carbon contained in forests.
  • For activities based on results, the developing countries will be compensated.The REDD+ goes beyond simply deforestation and forest degradation by including the role of conservation, sustainable management of forests and enhancement of forest carbon stocks.

 

Question 12

With reference to ‘Agenda 21’, sometimes seen in the news, consider the following statements:

  1. It is a global action plan for sustainable development.

  2. It originated in the World Summit on Sustainable Development held in Johannesburg in 2002.

Which of the statements given above is/are correct?

(a) 1 only

(b) 2 only

(c) Both 1 and 2

(d) Neither 1 nor 2

Ans: a

Sub-Theme: Sustainable Development Goals

Rio Declaration on Environment and Development:

  • The Rio  Declaration  consisted  of 27 principles intended to guide countries in future sustainable development. It was signed by over 175 countries.

Agenda 21:

  • It is a non-binding action plan sourced from Rio Earth Summit, 1992.
  • The number “21” refers to an agenda for the 21st century.
  • Objective: to achieve Global sustainable
  • Since 2015, Sustainable Development Goals are included in the Agenda 2030.

 

Question 13

The FAO accords the status of ‘Globally Important Agricultural Heritage System (GIAHS)’ to traditional agricultural systems.

What is the overall goal of this initiative?

1. To provide modern technology, training in modern farming methods and financial support to local communities of identified GIAHS so as to greatly enhance their agricultural productivity.

2. To identify and safeguard eco-friendly traditional farm practices and their associated landscapes, agricultural biodiversity and knowledge systems of the local communities.

3. To provide Geographical Indication status to all the varieties of agricultural produce in such identifies GIAHS.

Select the correct answer using the code given below.

(a) 1 and 3 only

(b) 2 only

(c) 2 and 3 only

(d) 1, 2 and 3

Ans: b

Sub-Theme: Major Environmental Organizations

    • Globally Important Agricultural Heritage System (GIAHS)
    • Outstanding landscapes of aesthetic beauty constitute “Globally Important Agricultural Heritage Systems” that mix resilient ecosystems, agricultural biodiversity, and priceless cultural legacy.
    • The FAO established the Globally Important Agricultural Heritage Systems (GIAHS) programme to protect and promote the world’s agricultural heritage systems.

Statement 1 is incorrect: Globally Important Agricultural Heritage Systems (GIAHS) was started by the FAO to safeguard and support the world’s agricultural heritage systems.

Statement 2 is correct: The initiative intends to safeguard the social, cultural, economic and environmental goods and services of these heritage systems.

Statement 3 is incorrect: There is no such provision. GI is accorded to products from a certain origin (e.g. Hyderabadi Biryani) and not to all traditional agricultural systems.

  • Three sites recognised as GIAHS in India:
    1. Kuttanad Below Sea Level Farming System of Kerala.
    2. Koraput Traditional  Agriculture of Odisha.
    3. Pampore Saffron Heritage of Kashmir.

NOTE: The central theme of the question is “Heritage System”. Statement 1 talks about “modern technology, training in modern farming” which is in contradiction to the given question, so this can be marked as incorrect. Statement 3 is also incorrect as FAO has no role in accordance with GI status. In India, GI status is governed by the Geographical Indications of Goods (Registration and Protection) Act, 1999 (GI Act) in India. Geographical Indications protection is granted through the TRIPS Agreement.

 

Question  14

What is/are unique about ‘Kharai camel’, a breed found in India?

  1. It is capable of swimming up to three kilometres in seawater.

  2. It survives by grazing on mangroves.

  3. It lives in the wild and cannot be domesticated.

Select the correct answer using the code given

(a) 1 and 2 only

(b) 3 only

(c) 1 and 3 only

(d) 1, 2 and 3

Ans: a

Sub-Theme: Species (Fauna)

Statement 1 is correct: Kharai camels have a special ability to swim in seawater and feed on saline plants and mangroves, which is how they get their name, Kharai. Recognized as a separate breed a few years ago by the National Bureau of Animal Genetic Resources (NBAGR), the Kharai camel is probably the only domesticated breed of camel that lives in dual ecosystems. There are two camel breeds in Kutch, a coastal region of Gujarat that is also a sizable desert territory. IUCN Red List: Endangered

Statement 2 is correct: Kharai camels are known to feed on mangroves on the island offshore. And to eat this salty marine food, they sometimes swim for hours.

Statement 3 is incorrect: Given the breed’s ability to survive both on land and sea, this particular statement is highly unlikely.

NOTE: Observe the extreme keyword ‘cannot’ in statement 3, it makes the statement a bit absurd from the other two statements. Hence, by eliminating this single statement you can easily reach the correct answer.

 

Question  15

With reference to ‘Red Sanders’, sometimes seen in the news, consider the following statements:

  1. It is a tree species found in a part of South India.

  2. It is one of the most important trees in the tropical rainforest areas of South India.

Which of the statements given above is/are correct?

(a) 1 only

(b) 2 only

(c) Both 1 and 2

(d) Neither 1 nor 2

Ans: a         

Sub-Theme: Species (Flora)

Statement 1 is correct: Red Sanders is an Indian endemic tree species, with a restricted geographical range in the Eastern Ghats. It is endemic to a distinct tract of forests in Andhra Pradesh. Red Sanders usually grow in the rocky, degraded and fallow lands with Red Soil and hot and dry climate. IUCN Red List: Endangered.

Statement 2 is incorrect: It occurs in the forest formation which is classified as “Southern Tropical Dry Deciduous Forests”.

Threats: Illicit felling for smuggling, forest fires, cattle grazing and other anthropogenic threats. The tree is known for their rich hue and therapeutic properties, are high in demand across Asia, particularly in China and Japan, for use in cosmetics and medicinal products as well as for making furniture, woodcraft and musical instruments.

 

Question  16

In which of the following regions of India are you most likely to come across the ‘Great Indian Hornbill’ in its natural habitat?

(a) Sand deserts of northwest India

(b) Higher Himalayas of Jammu and Kashmir

(c) Salt marshes of western Gujarat

(d) Western Ghats

Ans: d         

Sub-Theme: Species – Fauna

Great Indian Hornbill (Western Ghats, Nilgiris)

Option (d) is the correct answer: The great hornbill (Buceros bicornis) also known as the great Indian hornbill or great pied hornbill, is one of the larger members of the hornbill family. A large majority of their population is found in India with a significant proportion in the Western Ghats and the Nilgiris. They also occur in Myanmar, islands in the Mergui archipelago, southern China, Vietnam, Laos, Cambodia, Thailand, peninsular Malaysia and in Sumatra, Indonesia.

 

2015

 

Question  1

Which one of the following is the best description of the term ‘ecosystem’?

(a) A community of organisms interacting with one another

(b) That part of the Earth which is inhabited by living organisms

(c) A community of organisms together with the environment in which they live

(d) The flora and fauna of a geographical area

Ans: c

Sub-Theme: Ecosystem

Option (c) is the correct answer: Ecosystem can be defined as a community of organisms together with the environment in which they live.

Ecosystem:

  • An ecosystem can be defined as a functional unit of nature, where living organisms (producers, consumers, and decomposers) interact among themselves and also with the surrounding physical
  • An ecosystem can be of any size but usually encompasses specific and limited species. E.g. Aquatic Ecosystem.
  • In the ecosystem, biotic and abiotic components are linked together through nutrient cycles and energy flows.
  • Everything that lives in an ecosystem is dependent on the other species and elements that are also part of that ecological community.
  • A damaged or extinct component of an ecosystem has an effect on the entire system.

 

Question  2

What can be the impact of excessive/inappropriate use of nitrogenous fertilizers in agriculture?

1. Proliferation of nitrogen fixing microorganisms in soil can occur.

2. Increase in the acidity of soil can take place.

3. Leaching of nitrate to the groundwater can occur.

Select the correct answer using the codes given below.

(a) 1 and 3 only

(b) 2 only

(c) 2 and 3 only

(d) 1, 2 and 3

Ans: c

Sub-Theme: Soil Pollution

Impact of Excessive/inappropriate use of nitrogenous fertilizers:

  • The necessity for nitrogen-fixing bacteria decreases as soil nitrogen levels increase.
  • The soil becomes more acidic and nitrate leaches into the groundwater when nitrogenous fertilisers are used excessively or inappropriately.
  • Fertilizers in agriculture increase inputs of nutrients to soils affecting their pH value
  • The molecule of nitrogen change soil pH in the form of uncharged urea molecule, the cation ammonium and the anion nitrate
  • The conversion of Nitrogen from one form to the other involves the generation or consumption of acidity and the uptake of urea, ammonium or nitrate by plants will also affect the acidity of the soil. Due to the leaching process, these substances accumulate underground.

Statement 1 is incorrect: More nitrogen in the soil leads to less need for nitrogen-fixing bacteria.

Statement 2 and 3 are correct: Excessive/ inappropriate use of nitrogenous fertilizers increases the acidity of soil and the leaching of nitrate into the groundwater.

NOTE: Excessive or inappropriate use of anything will have a negative impact on the environment, there should be a balance in everything. Now, look at the extreme keyword in the first statement. ‘Proliferation’     which   means ‘rapid increase’, think logically, is it possible that the use of excessive nitrogenous fertilizer will proliferate the nitrogen-fixing microorganisms in soil? Having said that, it is also advisable to read this topic thoroughly as it is very important and high yielding both in terms of prelims and mains.

 

Question  3

Which one of the following is associated with the issue of control and phasing out of the use of ozone- depleting substances?

(a) Bretton Woods Conference

(b) Montreal Protocol

(c) Kyoto Protocol

(d) Nagoya Protocol

Ans: b

Sub-Theme: Convention for the Protection of the Ozone Layer

Montreal Protocol

  • It is a protocol to the Vienna Convention and it deals with the substances that deplete the Ozone Layer (ozone-depleting substance- ODS).
  • It was the first treaty to achieve universal
  • The protocol recognizes that all nations should not be treated equally.

Option (a) is incorrect: Bretton Woods Conference established the International Bank for Reconstruction and Development (IBRD) and the International Monetary Fund (IMF).

Option (b) is correct: Montreal Protocol is a protocol to the Vienna Convention and it deals with the substances that deplete the Ozone Layer (ozone-depleting substance-ODS).

Option (c) is incorrect: Kyoto Protocol deals with the reduction of Greenhouse gas.

Option (d) is incorrect: Nagoya Protocol is a supplementary agreement to the 1992 Convention on Biological Diversity (CBD) on “Access to Genetic Resources and the Fair and Equitable Sharing of Benefits Arising from their Utilization”.

 

 

Question  4

What is the Rio+20 Conference, often mentioned in the news?

(a) It is the United Nations Conference on Sustainable Development

(b) It is a Ministerial Meeting of the World Trade Organization

(c) It is a Conference of the Intergovernmental Panel on Climate Change.

(d) It is a Conference of the Member Countries of the Convention on Biological Diversity.

Ans: a

Sub-Theme: Major Environment Conventions, Conference, etc.

  • Rio+20 is the short name for the United Nations Conference on Sustainable Development which took place in Rio de Janeiro, Brazil in June 2012, twenty years after the landmark 1992 Earth Summit in
  • At the Rio+20 Conference, world leaders, along with thousands of participants from the private sector, NGOs and other groups, came together to shape how we can reduce poverty, advance social equity and ensure environmental protection on an ever more crowded planet.
  • The official discussions focussed on two main themes:
    • How to build a green economy to achieve sustainable development and lift people out of poverty; and
    • How to improve international coordination for sustainable development.

 

Question  5

The Genetic Engineering Appraisal Committee is constituted under the

(a) Food Safety and Standards Act, 2006

(b) Geographical Indications of Goods (Registration and Protection) Act, 1999

(c) Environment (Protection) Act, 1986

(d) Wildlife (Protection) Act, 1972

Ans: c

Sub-Theme: Genetically Modified Crops

Genetic Engineering Appraisal Committee

  • The Genetic Engineering Appraisal Committee was constituted under the Environment (Protection) Act, 1986.
  • The Genetic Engineering Appraisal Committee (GEAC) functions under the Ministry of Environment, Forest and Climate Change (MoEF&CC).
  • It is responsible for the appraisal of activities involving large-scale use of hazardous microorganisms and recombinants in research and industrial production from the environmental angle.
  • The committee is also responsible for the appraisal of proposals relating to the release of genetically engineered (GE) organisms and products into the environment including experimental field trials.
  • GEAC is chaired by the Special Secretary/ Additional Secretary of MoEF&CC and co-chaired by a representative from the Department of Biotechnology (DBT)

NOTE: There are recurrent and repeated questions on “Genetic Engineering” either in science-tech or environment. It is advisable to prepare this topic thoroughly and holistically.

 

 

Question 6

With reference to an organisation known as ‘BirdLife International’, which of the following statements is/ are correct?

  1. It is a Global Partnership of Conservation

  2. The concept of ‘biodiversity hotspots’ originated from this organisation.

  3. It identifies the sites known/referred to as ‘Important Bird and Biodiversity Areas’.

Select the correct answer using the code given below.

(a) 1 only

(b) 2 and 3 only

(c) 1 and 3 only

(d) 1, 2 and 3

Ans: c

Sub-Theme: Major Conservation Organizations

BirdLife International

  • BirdLife International is an international alliance of conservation groups that works to protect birds, their habitats, and the world’s biodiversity while collaborating with people to ensure the sustainable use of natural resources.
  • With more than 120 partner groups, it is the largest cooperation of conservation organisations in the world.
  • The IUCN’s designated Red List Authority for birds is BirdLife International.
  • It lists the locations that are designated as “Important Bird and Biodiversity Areas.”

Statement 1 is correct: BirdLife International is a global partnership of conservation organisations that strives to conserve birds, their habitats, and global biodiversity, working with people towards sustainability in the use of natural resources.

Statement 2 is incorrect: The idea of Biodiversity Hotspots was first put forward by Norman Myers in 1988.

Statement 3 is correct: It identifies the sites known/referred to as ‘Important Bird and Biodiversity Areas’.

 

Question 7

With reference to the International Union for Conservation of Nature and Natural Resource (IUCN) and the Convention of International Trade in Endangered Species of Wild Fauna and Flora (CITES), which of the following statements is/are correct?

1. IUCN is an organ of the United Nations and CITES is an international agreement between

2. IUCN runs thousands of field projects around the world to better manage natural environments.

3. CITES is legally binding on the States that have joined it, but this Conventional does not take the place of national laws.

Select the correct answer using the codes given below.

(a) 1 only

(b) 2 and 3 only

(c) 1 and 3 only

(d) 1, 2 and 3

Ans: b

Sub-Theme: Major Environmental Conventions

Statement 1 is incorrect: IUCN is an NGO, founded in October 1948 as the International Union for the Protection of Nature (or IUPN) following an international conference in Fontainebleau, France. Later the name was changed to International Union for Conservation of Nature and Natural Resources (IUCN). CITES on the other hand is an intergovernmental treaty, concluded under the aegis of the United Nations Environment Programme.

Statement 2 is correct: IUCN supports scientific research, manages field projects globally and brings governments, non-government organisations, United Nations agencies, companies and local communities together to develop and implement policy.

Statement 3 is correct: CITES is legally binding but does not take the place of national laws.

NOTE: Only knowing the names of UN specialised agencies is enough for solving this question. With this knowledge, we can eliminate option 1 and get the correct answer. Most of the time the examiner connects two different statements in which one is correct and the other is incorrect. It is advisable to ascertain the correctness of both connected statements and don’t rush after only reading a partial statement.

 

Question 8

Which of the following statements regarding ‘Green Climate Fund’ is/are correct?

1. It is intended to assist developing countries in adaptation and mitigation practices to counter climate change.

2. It is founded under the aegis of UNEP, OECD, Asian Development Bank and World Bank.

Select the correct answer using the codes given below.

(a) 1 only

(b) 2 only

(c) Both 1 and 2

(d) Neither 1 nor 2

Ans: a

Sub-Theme: Global Climate Finance Architecture

Green Climate Fund (GCF)

  • It is a UNFCCC financial tool.
  • Through the idea of country ownership, GCF initiatives are in line with the interests of developing nations.
  • It aims to contribute significantly to the worldwide community’s mitigation and adaptation efforts to combat climate change.

Statement 1 is correct: Green Climate Fund aims to make an ambitious contribution to attaining the mitigation and adaptation goals of the international community to counter climate change.

Statement 2 is incorrect: GCF was discussed in Cancun Summit 2010 of UNFCCC and finally adopted in Durban Summit 2011.

 

Question 9

Which of the following National Parks is unique in being a swamp with floating vegetation that supports a rich biodiversity?

(a) Bhitarkanika National Park

(b) Keibul Lamjao National Park

(c) Keoladeo Ghana National Park

(d) Sultanpur National Park

Ans: b          

Sub-Theme: National Parks/ Tiger Reserve/ Biosphere Reserve/Wildlife Sanctuary/Wetland

Option (b) is the correct answer: Keibul Lamjao National Park in the Bishnupur district of the Indian state of Manipur is the only floating park in existence and is a crucial component of Loktak Lake, which lies in North East India. Locally known as “Phumdi,” the national park is distinguished by floating decaying plant matter. It was established in 1966 as a wildlife sanctuary to protect the Eld’s deer’s natural habitat, which is in risk of extinction (Cervus eldi eldi). It was declared a national park in 1977. The park is unique in that it is “too shallow to be a lake, too deep to be marsh.”

Flora: The park offers a diverse array of aquatic, wetland, and terrestrial ecosystems and is mostly made up of damp, semi-evergreen forests.

Fauna found in the park include the brow- antlered deer (Cervus eldi eldi), which is its emblematic species, as well as the Asian Golden Cat, Marbled Cat, Malayan Bear, Hog Deer, Himalayan Black Bear, and others.

 

Question 10

Which one of the following National Parks has a climate that varies from tropical to subtropical, temperate and arctic?

(a) Khangchendzonga National Park

(b) Nanda Devi National Park

(c) Neora Valley National Park

(d) Namdapha National Park

Ans: d          

Sub-Theme: National Parks/ Tiger Reserve/ Biosphere Reserve/Wildlife Sanctuary/Wetland

Option (d) is the correct answer: The Namdapha National Park is located 27° N of the equator. It is located at the international border between India and Myanmar (Burma) in Changlang District, Arunachal Pradesh in northeast India. Geographically, Namdapha National Park is in the sub-tropical zone because it is 27° N of the equator, but because its altitude ranges from 200 m to 4571 m, its climate changes and it also experiences a lot of rain. There are different types of climates, such as tropical, subtropical, temperate, and arctic. For instance, the Namdapha National Park’s mountainous region has a mountain climate, whereas the low-lying plains and valleys experience a tropical climate. It was established as the 15th Tiger Reserve of India by the Government in 1983.

 

Question 11

With reference to ‘dugong’, a mammal found in India, which of the following statements is/are correct?

  1. It is a herbivorous marine animal.

  2. It is found along the entire coast of India.

  3. It is given legal protection under Schedule I of the Wildlife (Protection) Act, 1972.

Select the correct answer using the codes given below.

(a) 1 and 2

(b) 2 only

(c) 1 and 3

(d) 3 only

Ans: c         

Sub-Theme: Marine Species

Statement 1 is correct: Sea Cow or Dugong is one of the four surviving species in the Order Sirenia and it is the only existing species of herbivorous mammal that lives exclusively in the sea including in India.

Statement 2 is incorrect: They are found in over 30 countries and in India are seen in the Gulf of Mannar, Gulf of Kutch, Palk Bay, and the Andaman and Nicobar Islands.

Statement 3 is correct: It gives legal protection under Schedule I of the Wildlife (Protection) Act, 1972.

 

Question 12

Which one of the following is the national aquatic animal of India?

(a) Saltwater crocodile

(b) Olive ridley turtle

(c) Gangetic dolphin

(d) Gharial

Ans: c

Sub-Theme: Marine Species

Option (c) is the correct answer: Gangetic River Dolphin, also called ‘susu’ is the National Aquatic Animal of India.

 Gangetic Dolphin

  • Scientific Name: Platanista gangetica
  • The Ganges River Dolphin was officially discovered in 1801.
  • They are also called ‘susu’ is the National Aquatic Animal of India.

Habitat:

  • Live in the Ganges-Brahmaputra-Meghna and Karnaphuli-Sangu river systems of Nepal, India, and Bangladesh.
  • The Ganges river dolphin can only live in freshwater

Features:

  • Essentially blind.
  • They hunt by emitting ultrasonic sounds, which bounces off of fish and other prey, enabling them to “see” an image in their mind.

 

2014

 

Question 1

Which of the following are some important pollutants released by steel industry in India?

  1. Oxides of sulphur

  2. Oxide of nitrogen

  3. Carbon monoxide

  4. Carbon dioxide

Select the correct answer using the code given below.

(a) 1, 3 and 4 only

(b) 2 and 3 only

(c) 1 and 4 only

(d) 1, 2, 3 and 4

Ans: d

Sub-Theme: Industrial Pollution

Option (d) is the correct answer: All the above are major pollutants released by the steel industry in India.

The environmental impact of steel production:

  • Steel production has a number of impacts on the environment, including air emissions (CO, SOx, NOx, PM2), wastewater contaminants, hazardous wastes, and solid wastes.
  • The major environmental impacts of integrated steel mills are from coking and iron-making.

Emissions to Air:

  • Coke production is one of the major pollution sources from steel production.
  • Air emissions such as coke oven gas, naphthalene, ammonium compounds, crude light oil, sulfur and coke dust are released from coke ovens.

Emissions to water:

  • Water emissions come from the water used to cool coke after it has finished baking.
  • Quenching water becomes contaminated with coke breezes and other compounds.
  • While the volume of contaminated water can be great, quenching water is fairly easy to reuse. Most pollutants can be removed by filtration.

Waste:

  • Slag, the limestone and iron ore impurities collected at the top of the molten iron make up the largest portion of iron-making by-products.
  • Sulfur dioxide and hydrogen sulfide are volatized and captured in air emissions control equipment and the residual slag is sold to the construction industry.
  • While this is not a pollution prevention technique, solid waste does not reach

 

Question 2

There is some concern regarding the nanoparticles of some chemical elements that are used by the industry in the manufacture of various Why?

  1. They can accumulate in the environment and contaminate water and soil.

  2. They can enter the food chains.

  3. They can trigger the production of free radicals.

Select the correct answer using the code given below.

(a) 1 and 2 only

(b) 3 only

(c) 1 and 3 only

(d) 1, 2 and 3

Ans: d

Sub-Theme: Industrial Pollution

Use of Nanoparticles:

  • Nanoparticles are used in the manufacture of scratch proof eyeglasses, crack-resistant paints, anti-graffiti coatings for walls, transparent sunscreens, stain-repellent fabrics, and self-cleaning windows, and ceramic coatings for solar cells.
  • Additionally, stronger, lighter, cleaner, and more intelligent surfaces and systems can be created with nanoparticles.

Impact of Nanoparticles:

  • They concentrate in the environment and contaminate the land and water.
  • They enter into the food
  • They also trigger the production of free
  • Nanoparticles may cross cell
  • Nanoparticles can reach the blood and may reach other target sites such as the liver, heart, or blood cells and may cause damage to cells as they stay for a persistently long

Statement 1 is correct: A matter particle with a dimension ranging from 1 to one 100 nanometers (nm) is referred to as a nanoparticle. They can accumulate in the environment, contaminate water and soil.

Statement 2 is correct: They enter into the food chains.

Statement 3 is correct: They also trigger the production of free radicals.

 

Question 3

Brominated flame retardants are used in many household products like mattresses and upholstery. Why is there some concern about their use?

1. They are highly resistant to degradation in the environment.

2. They are able to accumulate in humans and animals.

Select the correct answer using the code given below.

(a) 1 only

(b) 2 only

(c) Both 1 and 2

(d) Neither 1 nor 2

Ans: (c)

Sub-Theme: General Environment

Brominated Flame Retardants (BFRs):

  • Brominated flame retardants (BFRs) are mixtures of man-made chemicals that are added to a wide variety of products, including for industrial use, to make them less flammable.
  • They are used commonly in plastics, textiles and electrical/electronic equipment.

Concerns:

  • They don’t degrade easily in fact they are highly resistant to degradation in the
  • Some brominated flame retardants were identified as persistent, bio accumulative, and toxic to both humans and the environment and were suspected of causing neurobehavioral effects and endocrine

Statement 1 is correct: They don’t degrade easily in fact they are highly resistant to degradation in the environment.

Statement 2 is correct: Some brominated flame retardants were identified as persistent, bioaccumulative, and toxic to both humans and the environment and were suspected of causing neurobehavioral effects and endocrine disruption.

 

Question 4

The scientific view is that the increase in global temperature should not exceed 2°C above the pre- industrial level. If the global temperature increases beyond 3°C above the pre-industrial level, what can be its possible impact/impacts on the world?

1. Terrestrial biosphere tends toward a net carbon source.

2. Widespread coral mortality will occur.

3. All the global wetlands will permanently disappear.

4. Cultivation of cereals will not be possible anywhere in the world.

Select the correct answer using the code given below:

(a) 1 only

(b) 1 and 2 only

(c) 2, 3 and 4 only

(d) 1, 2, 3 and 4

Ans: b

Sub-Theme: Effects of Global Warming

Statement 1 is correct: If the global temperature increases beyond 3°C above the pre-industrial level, the terrestrial biosphere tends toward a net carbon source. Taiga and temperate forests act as an important carbon sink; these forests would turn into a carbon source.

Statement 2 is correct: As temperatures rise, mass coral bleaching events occur. If the temperature keeps on rising, widespread coral mortality will occur.

Statement 3 is incorrect: The temperate regions can still survive if the temperature increases. The temperate zones are where the widest seasonal changes occur. The vegetation can survive there.

Statement 4 is incorrect: The weather should be warm and moist during the early stage of the growth of cereals. Therefore, the cultivation of cereals would still be possible if the temperature rises.

 

Question 5

What are the significances of a practical approach to sugarcane production known as ‘Sustainable Sugarcane Initiative’?

  1. Seed cost is very low in this compared to the conventional method of cultivation.

  2. Drip irrigation can be practised very effectively in this.

  3. There is no application of chemical/inorganic fertilizers at all in this.

  4. The scope for intercropping is more in this compared to the conventional method of cultivation.

Select the correct answer using the code given below.

(a) 1 and 3 only

(b) 1, 2 and 4 only

(c) 2, 3 and 4 only

(d) 1, 2, 3 and 4

Ans: b

Sub-Theme: Sustainable Agriculture

Sustainable Sugarcane Initiative:

The Sustainable Sugar Initiative (SSI) was launched jointly by International Crops Research Institute for the Semi-Arid Tropics (ICRISAT) and by ICRISAT and World Wide Fund for Nature (WWF).

Aims: Cultivating sugarcane mainly by changing the way the inputs and methods are used, such as less use of seeds ,less use of water, optimum utilization of fertilizers and land, aims at reducing the input cost.

Statement 1 is correct: It majorly aims at reduction of the input cost compared to the conventional method of cultivation. Hence, statement 1 is correct.

Statements 2 and 4 are correct: SSI provides ample scope for intercropping and the drip irrigation method.

Statement 3 is incorrect: There is an application of both Inorganic and organic fertilizers such as Nitrogen, Phosphorus, and Potassium (NPK) fertilizers.

NOTE: here, the topic in question is the “Sustainable” Sugarcane Initiative (SSI). Statement 2 talks about Drip-irrigation which is sustainable irrigation practice, that complies and is in conformity with SSI. Statement 1 talks about low input cost which is also in confirmation of sustainable practices. However, statement 3 endorses that there is no application of chemical/ inorganic fertilizers at all” which sounds extreme. So statement 3 can be eliminated to get the correct answer.

 

Question 6

Consider the following international agreements:

  1. The International Treaty on Plant Genetic Resources for Food and Agriculture.

  2. The United Nations Convention to Combat Desertification

  3. The World Heritage Convention

Which of the above has/have a bearing on biodiversity?

(a) 1 and 2 only

(b) 3 only

(c) 1 and 3 only

(d) 1, 2 and 3

Ans: d

Sub-Theme: International efforts to protect biodiversity/Laws and Legislations

  • All the three agreements have a bearing on biodiversity.
  • International Treaty on Plant Genetic Resources for Food and Agriculture
  • Adopted by the 31st session of the Conference of the Food and Agriculture Organization (FAO) of the United Nations on 3rd November 2001.
  • It is also known as the Seed Treaty. India is a signatory to the treaty.

Objectives:

  • Recognising Farmers’ Contribution
  • Access and Benefit Sharing
  • Sustainability

United Nations Convention to Combat Desertification (UNCCD):

  • Established in 1994.
  • It is the sole legally binding international agreement linking environment and development to sustainable land management.
  • Aim: Its 197 Parties aim, through partnerships, to implement the Convention and achieve the Sustainable Development Goals. The end goal is to protect land from overuse and drought, so it can continue to provide food, water and energy.
  • Only convention stemming from a direct recommendation of the Rio Conference’s Agenda 21.
  • It is called the Rio Convention along with its sister conventions:
    1. United Nations Framework Convention on Climate Change (UNFCCC)
    2. UN Convention on Biological Diversity (UNCBD)

World Heritage Convention:

  • Convention Concerning the Protection of the World Cultural and Natural Heritage by UNESCO
  • Primary mission: to identify and protect the world’s natural and cultural heritage considered to be of Outstanding Universal Value.
  • India is a signatory to the convention and has 40 world heritage sites, which include 32 cultural, 7 natural and one mixed property. It acknowledges how people interact with nature and the critical necessity to maintain their delicate equilibrium.

NOTE: Question on the same theme earlier has been asked in 2011 i.e provisions bearing on biodiversity. It is advisable to go through provisions of the Indian constitution bearing upon the environment and its conservation.

 

Question 7

Consider the following statements regarding ‘Earth Hour’

  1. It is an initiative of UNEP and UNESCO

  2. It is a movement in which the participants switch off the lights for one hour on a certain day every year.

  3. It is a movement to raise awareness about climate change and the need to save the planet.

Which of the statements given above is/are correct?

(a) 1 and 3 only

(b) 2 only

(c) 2 and 3 only

(d) 1, 2 and 3

Ans: c

Sub-Theme: UNEP/ Major Programmes of the UNEP

Earth Hour:

  • Earth Hour is a global movement for environmental conservation, started by The World Wide Fund for Nature (WWF) in 2007.
  • It is a yearly occurrence that happens on the last Saturday in March. In more than 180 nations and territories, millions of people take part in it by turning down their lights.
  • Aim: It aims to call attention to global warming, loss of biodiversity, and climate
  • As a global solidarity, people turn off their lights for one hour during Earth Hour.

Significance: The urgent need to address global climate change is highlighted in this hour. Additionally, it raises public awareness of environmental issues.

Statement 1 is incorrect: Earth Hour is a global movement for environmental conservation, started by the World Wide Fund for Nature (WWF) in 2007.

Statement 2 is correct: It is an annual event that takes place on the last Saturday of March. Millions of people in over 180 countries and territories participate in it by turning off their lights.

Statement 3 is correct: It showcases the need to take action on the pressing issue of global climate change. Further, it promotes awareness about environmental issues on a public platform.

 

Question 8

If a wetland of international importance is brought under the ‘Montreux Record’, what does it imply?

(a) Changes in ecological character have occurred, are occurring or are likely to occur in the wetland as a result of human interference.

(b) The country in which the wetland is located should enact a law to prohibit any human activity within five kilometres from the edge of the wetland.

(c) The survival of the wetland depends on the cultural practices and traditions of certain communities living in its vicinity and therefore the cultural diversity therein should not be destroyed.

(d) It is given the status of ‘World Heritage Site’

Ans: a

Sub-Theme: Wetland Conservation

Option (a) is the correct answer: ‘Montreux Record’ is a register of wetlands on the List of Wetlands of International Importance where ecological character changes have occurred, are occurring, or are anticipated to occur due to technological advancements, pollution, or other human influence.

Wetlands:

  • The convention uses a broad definition that encompasses all marsh, fen, peatland, and water areas, whether they are man-made or natural, permanent or temporary, with static or flowing fresh, brackish, or salt water, including marine areas whose depth at low tide is less than six metres.
  • World Wetland Day – 2nd February
  • Presently, India stands first in South Asia and third in Asia in terms of number of designated sites.

Ramsar Convention:

  • It is an international agreement between nations aimed at “the conservation and sustainable use of wetlands.”
  • The Convention on Wetlands is another name for it. It is named after the Iranian city of Ramsar.
  • India signed in Ramsar, Iran, in 1971
  • It is the only international treaty that addresses a specific ecosystem (wetland.)

Ramsar Convention is not a regulatory regime.

NOTE: This is a straightforward question from Montreux Record under Ramsar Convention. Also, these are high yielding topics and to attempt this type of question aspirants are requested to go through all the National Parks/Wildlife Sanctuary/ Biosphere Reserve/Wetlands including their locations (map), and geographical features, important flora and fauna if any.

 

Question 9

With reference to a conservation organisation called ‘Wetlands International’, which of the following statements is/are correct?

1. It is an intergovernmental organisation formed by the countries which are signatories to the Ramsar convention.

2. It works at the field level to develop and mobilise knowledge, and use the practical experience to advocate for better policies.

Select the correct answer using the code given below.

(a) 1 only

(b) 2 only

(c) Both 1 and 2

(d) Neither 1 nor 2

Ans: b

Sub-Theme: Wetland Conservations

Wetlands International:

  • It is a global organisation that works to sustain and restore wetlands and their resources for people and biodiversity.
  • It is an independent, not-for-profit, global organization, supported by government and NGO members from around the world.
  • Wetlands International’s work ranges from research and community-based field projects to advocacy and engagement with governments, corporate and international policy fora and
  • Wetlands International works through partnerships and is supported by contributions from an extensive specialist expert network and tens of thousands of volunteers.
  • It was founded in 1937 as the International Wildfowl Inquiry and the organization was focused on the protection of waterbirds. Later, the name became International Waterfowl & Wetlands Research Bureau (IWRB).
  • The scope became wider; besides waterbirds, the organization was also working on the protection of wetland areas.
  • Later, organizations with similar objectives emerged in Asia and the Americas: the Asian Wetland Bureau (AWB) (initiated as INTERWADER in 1983) and Wetlands for the Americas (WA) (initiated in 1989). In 1991, the three organizations started to work closely together.
  • In 1995, the working relationship developed into the global organization Wetlands International.

Statement 1 is incorrect: Wetlands International is an independent, not-for-profit, global organization, supported by government and NGO members from around the world.

Statement 2 is correct: Wetlands International does work at the field level to develop and mobilise knowledge and use the practical experience to advocate for better policies.

NOTE: These are high yielding topics and to attempt this type of question aspirants are requested to go through all the National Parks/Wildlife Sanctuary/Biosphere Reserve/ Wetlands including their locations (map), geographical features, important flora and fauna if any.

 

 

Question 10

Consider the following pairs:

Wetlands                        Confluence of Rivers

  1. Harike Wetlands: Confluence of Beas and Satluj/ Sutlej

  2. Keoladeo Ghana National Park: Confluence of Banas and Chambal

  3. Kolleru Lake: Confluence of Musi and Krishna

Which of the above pairs is/are correctly matched?

(a) 1 only

(b) 2 and 3 only

(c) 1 and 3 only

(d) 1, 2 and 3

Ans: a

Sub-Theme: Wetland Conservation/Map Based

Harike Wetland

  • Harike Wetland also is the largest wetland in northern India on the border of Tarn Taran Sahib district and Ferozepur district of Punjab.
  • The wetland and the lake were formed by constructing the headworks across the Sutlej River in 1953, located downstream of the confluence of the Beas and Sutlej rivers just south of Harike village.

Keoladeo National Park

  • The Keoladeo National Park or Keoladeo Ghana National Park is formerly known as the Bharatpur Bird Sanctuary is in the Bharatpur District of Rajasthan.
  • It is situated at the confluence of the Gambhira and the Banganga rivers.

Kolleru Lake

  • It is the largest freshwater lake in Andhra Pradesh, India
  • It is located between Krishna and Godavari deltas.
  • It spans two districts of Andhra Pradesh-Krishna and West Godavari.

Pair 1 is correct: The Harike Wetland is located downstream of the confluence of the Beas and Sutlej rivers just south of Harike village.

Pair 2 is incorrect: The Keoladeo National Park is situated at the confluence of the Gambhira and the Banganga rivers.

Pair 3 is incorrect: Kolleru Lake is located between Krishna and Godavari deltas.

 

Question 11

With reference to ‘Global Environment Facility’, which of the following statements is/are correct?

(a) It serves as a financial mechanism for ‘Convention on Biological Diversity’ and ‘United Nations Framework Convention on Climate Change’.

(b) It undertakes scientific research on environmental issues at global level.

(c) It is an agency under OECD to facilitate the transfer of technology and funds to underdeveloped countries with specific aims to protect their environment.

(d) Both (a) and (b)

Ans: a

Sub-Theme: Climate Change Organization/ Global Environmental Facility

Option (a) is the correct answer: The GEF serves as financial mechanism for the following Conventions:

  • Stockholm
  • UN-FCCC
  • UN-CBD
  • UN-CCD
  • Montreal Protocol
  • Minamata Convention
  • GEF

GLOBAL ENVIRONMENT FACILITY (GEF)

  • GEF was established on the eve of the 1992 Rio Earth Summit to help tackle environmental issues.
  • Located in: Washington C., United States
  • Parties:
  • The GEF is jointly operated by the United Nations Development Programme (UNDP), World Bank, & United Nations Environment Programme (UNEP).

 

Question 12

Consider the following statements:

1. Animal Welfare Board of India was established under the Environment (Protection) Act, 1986.

2. The National Tiger Conservation Authority is a statutory body.

3. The National Ganga River Basin Authority is chaired by the Prime Minister.

Which of the statements given above is/are correct?

(a) 1 only

(b) 2 and 3 only

(c) 2 only

(d) 1, 2 and 3

Ans: b

Sub-Theme: Environmental Organizations and Institutions in India.

Statement 1 is incorrect: Animal Welfare Board of India was established in 1962 under Section 4 of the Prevention of Cruelty to Animals Act, 1960.

Statement 2 is correct: National Tiger Conservation Authority (NTCA) is a statutory body under the Ministry of Environment, Forests and Climate Change.

Statement 3 is correct: The National Ganga Council (Earlier National Ganga River Basin Authority) is chaired by the Prime Minister. The National Ganga Council was established in accordance with the Environment (Protection) Act, 1986. The National Ganga River Basin Authority was replaced (NGRBA). It is in charge of overseeing the restoration of the River Ganga Basin, including Ganga and its tributaries, and the prevention of pollution.

Animal Welfare Board of India

  • It is a statutory body advisory in nature on Animal Welfare Laws and promotes animal welfare in the country.
  • Founded in 1962 in accordance with Section 4 of the 1960 Prevention of Cruelty to Animals Act.
  • Under leadership of Rukmini Devi Arundale, the Animal Welfare Board of India was established, who was a well-known Bharatnatyam dancer and humanitarian Rukmini Devi Arundale.
  • The 28 members of the Board each have a three-year term.
  • Grants are given to organisations that endeavour to ensure that the nation’s animal welfare rules are observed.

National Tiger Conservation Authority

  • National Tiger Conservation Authority (NTCA) is a statutory organisation created under the Ministry of Environment, Forests, and Climate Change.
  • The Tiger Task Force’s proposals led to its establishment in 2005.
  • It was established in accordance with the authorities and duties given to it by the Wildlife (Protection) Act of 1972, as modified, under the enabling provisions of that Act.

 

Question 13

With reference to Bombay Natural History Society (BNHS), consider the following statements:

  1. It is an autonomous organisation under the Ministry of Environment and Forests.

  2. It strives to conserve nature through action-based research, education and public awareness.

  3. It organises and conducts nature trails and camps for the general public.

Which of the statements given above is/are correct?

(a) 1 and 3 only

(b) 2 only

(c) 2 and 3 only

(d) 1, 2 and 3

Ans: c

Sub-Theme: Major Conservation Organizations

Statement 1 is incorrect: BNHS is one of the largest non-governmental organisations in India engaged in conservation and biodiversity research.

Statement 2 is correct: It supports many research efforts through grants and publishes the Journal of the Bombay Natural History Society. It has been classified as a “Scientific and Industrial Research Organization” by the Department of Science and Technology.

Statement 3 is correct: It organises and conducts nature trails and camps for the general public.

BNHS

  • BNHS is one of the biggest non- governmental groups working on biodiversity and conservation.
  • It was established on September 15, 1883, and has its headquarters in Mumbai’s Hornbill House.
  • It funds numerous research projects, and it also puts out the Bombay Natural History Society Journal.
  • For the general public, it plans and runs environmental hikes and camps.
  • It has been classified as a “Scientific and Industrial Research Organization” by the Department of Science and Technology.
  • The great hornbill serves as the BNHS’s
  • The Internet of Birds platform, created by IT consulting company Accenture and the Bombay Natural History Society, uses Artificial Intelligence (AI) tools like machine learning and computer vision to identify bird species found in India from digital photographs that are uploaded.

 

Question 14

With reference to ‘Eco-Sensitive Zones’, which of the following statements is/are correct?

  1. Eco-Sensitive Zones are the areas that are declared under the Wildlife (Protection) Act, 1972.

  2. The purpose of the declaration of Eco-Sensitive Zones is to prohibit all kinds of human activities in those zones except agriculture.

Select the correct answer using the code given below.

(a) 1 only

(b) 2 only

(c) Both 1 and 2

(d) Neither 1 nor 2

Ans: d

Sub-Theme: Eco-Sensitive Zones

Statement 1 is incorrect: ESZs are notified by MoEFCC, Government of India under Environment Protection Act 1986.

Statement 2 is incorrect: The basic aim is to regulate certain activities around National Parks and Wildlife Sanctuaries so as to minimise the negative impacts of such activities on the fragile ecosystem encompassing the protected areas.

Activities in ESZs:

 

 

 

 

Prohibited activities

Commercial mining, sawmills, industries that pollute the environment (air, water, soil, noise, etc.), the construction of large hydroelectric projects (HEP), the use of wood for commercial purposes, tourism-related activities like hot-air balloon flights over national parks, the discharge of effluents or any solid waste, or the manufacture of hazardous materials.

 

 

Regulated activities

Tree cutting, the construction of hotels and resorts, the exploitation of natural water for commercial purposes, the installation of electricity lines, a radical transformation of agriculture, such as the use of heavy machinery, pesticides, etc., and road widening.

 

Permitted activities

The ongoing horticultural or agricultural techniques, the use of renewable energy sources, organic farming, rainwater harvesting, and the implementation of green technology across all sectors.

 

Question 15

The most important strategy for the conservation of biodiversity together with traditional human life is the establishment of:

(a) Biosphere reserves

(b) Botanical gardens

(c) National parks

(d) Wildlife sanctuaries

Ans: a

Sub-Theme: National Parks/Tiger Reserve/ Biosphere Reserve/Wildlife Sanctuary/Wetland

Option (a) is the correct answer: Biosphere reserves are legally protected regions where people and the environment can coexist while honouring each other’s needs. These areas might be terrestrial, coastal, or a combination of both. They are recognised under UNESCO’s MAB programme and nominated and established by the relevant nations. Hence, the most important strategy for the conservation of biodiversity together with traditional human life is the establishment of Biosphere Reserve.

Zonation Of Biosphere Reserve:

Core Areas

The central portions of biosphere reserves are strictly protected zones, which are frequently publicly owned lands that are subject to legal protection, such as a previously established national park, wilderness area, or wildlife refuge. The  core  region,  however,  could  be privately held or be a part of non- governmental groups.

Buffer Zones

Facilities for education,  training, tourism, and recreation may be located nearby the core area(s). In many biosphere reserves, the buffer zone  is  viewed  as  a  region  where scientific study is conducted and human use is less intense than what might be found in the transition zone.

Transition Areas

The  most     remote area      where   a society  supports  socioculturally  and environmentally sound  economic  and human activity.

 

NOTE: This is again a high yielding topic w.r.t. Environment and Ecology. To solve this type of question, aspirants need to have a clear concept regarding various conservation efforts. Hence, it is advisable that aspirants should read and understand this topic thoroughly.

 

Question 16

With reference to Neem tree, consider the following statements:

1. Neem oil can be used as a pesticide to control the proliferation of some species of insects and mites.

2. Neem seeds are used in the manufacture of biofuels and hospital detergents.

3. Neem oil has applications in the pharmaceutical industry.

Which of the statements given above is/are correct?

(a) 1 and 2 only

(b) 3 only

(c) 1 and 3 only

(d) 1, 2 and 3

Ans: c          

Sub-Theme: Tree Species

  • Neem Tree
    • Scientific Name: Azadirachta indica
    • Commonly known as neem, nimtree or Indian lilac.
    • It is a tree in the mahogany family Meliaceae.
    • It is native to the Indian subcontinent and most of the countries in Africa.
    • Neem is typically grown in tropical and subtropical regions. Neem trees also grow on islands in southern Iran. Its fruits and seeds are the sources of neem oil.
  • Importance of Neem Tree:
    • Neem trees are cultivated and utilized as multipurpose trees. Nearly all parts of the neem tree are useful.
    • In many areas, neem is considered a weed and based on its antimicrobial resistance and other antifungal properties, they are also used in pharmaceutical and cosmetic industries.

Statement 1 is correct: Neem oil and neem barks are used as an insect repellent.

Statement 2 is incorrect: No such relevant and strong reference has been found.

Statement 3 is correct: Neem oil has multiple applications in the pharmaceutical industry.

 

Question 17. Other than poaching, what are the possible reasons for the decline in the population of Ganges River Dolphins?

1. Construction of dams and barrages on rivers

2. An increase in the population of crocodiles in rivers

3. Getting trapped in fishing nets accidentally

4. Use of synthetic fertilisers and other agricultural chemicals in crop-fields in the vicinity of rivers.

Select the correct answer using the code given below.

(a) 1 and 2 only

(b) 2 and 3 only

(c) 1, 3 and 4 only

(d) 1, 2, 3 and 4

Ans: c

Sub-Theme: Marine Species

Statement 1 is correct: Gangetic dolphins live in the Ganges-Brahmaputra-Meghna and Karnaphuli-Sangu river systems of Nepal, India, and Bangladesh. The Ganges river dolphin can only live in freshwater. They are essentially blind. They are also called ‘susu’. Construction of dams and other irrigation-related projects make them susceptible to inbreeding and more vulnerable to other threats because they cannot move to new areas.

Statement 2 is incorrect: An increase in the population of crocodiles in rivers is not a threat to Gangetic dolphins.

Statement 3 is correct: Dolphins die as a result of accidentally being caught in fishing nets, also known as bycatch.

Statement 4 is correct: Industrial, agricultural, and human pollution is another serious cause of habitat degradation.

 

Question 18

If you walk through countryside, you are likely to see some birds stalking alongside the cattle to seize the insects disturbed by their movement through grasses.

Which one of the following is/are such bird/birds?

  1. Painted Stork

  2. Common Myna

  3. Black-necked Crane

Select the correct answer using the code given below.

(a) 1 and 2

(b) 2 only

(c) 2 and 3

(d) 3 only

Ans: b

Sub-Theme: Species(Avifauna)

Common Myna

  • It is an opportunistic insect feeder that is disturbed by grazing cattle.
  • It is an omnivorous open woodland bird with a strong sense of territorial instinct.
  • It has adapted extremely well to the urban

Statement 1 is incorrect: Painted storks forage in flocks in shallow waters along rivers or lakes. IUCN Red List: Near Threatened

Statement 2 is correct: Common Myna is an opportunistic feeder on insects, disturbed by grazing cattle.

Statement 3 is incorrect: The habitat of Black- necked Crane is in the high altitude wetlands of the Tibetan plateau, Sichuan (China), and eastern Ladakh (India). Therefore, it is highly unlikely it can be seen in the countryside. IUCN Red List: Near Threatened.

 

2013

 

Question 1

With reference to the food chains in ecosystems, which of the following kinds of organism is/are known as decomposer organism/organisms?

  1. Virus

  2. Fungi

  3. Bacteria

Select the correct answer using the codes given below:

(a) 1 only

(b) 2 and 3 only

(c) 1 and 3 only

(d) 1, 2 and 3

Ans: b

Sub-Theme: Biotic Components

Option (b) is the correct answer: Fungi and Bacteria, are known as decomposers.

Decomposer:

  • An organism that primarily feeds on dead organisms or the waste from living
  • Detritivores: Some organisms perform a similar function as decomposers, and are sometimes called detritivores.
  • The difference lies in the way decomposers and detritivores break down organic material. Detritivores must digest organic material within their bodies in order to break it down and gain nutrients from it.
  • Decomposers do not need to digest organic material internally in order to break it
  • Scavengers: Scavengers are the first to arrive at a dead organism’s remains. It includes lions, jackals, wolves, raccoons, and opossums. Example: Bacteria, fungi.

 

Question 2

In the grasslands, trees do not replace the grasses as a part of an ecological succession because of: 

(a) Insects and fungi

(b) Limited sunlight and paucity of nutrients

(c) Water limits and fire

(d) None of the above

Ans: c

Sub-Theme: Ecological Succession

Option (c) is the correct answer: Water limits and Fire restrict trees to replace the grasses as a part of an ecological succession.

Grassland Ecosystem:

  • Grassland ecosystems are an early stage of succession in regions where the mature ecosystems are forests.
  • Grass ecosystems, on the other hand, are climax ecosystems in grassland regions when there is insufficient precipitation to support a forest.
  • Grass that dries out during the dry season ignites fires that decimate other plant species and their seeds.
  • In areas with little rainfall, where plants cannot thrive, grasslands develop.
  • A grassland environment transforms into a desert ecosystem as a result of constraining forces including water scarcity and forest
  • Therefore, the ecological succession of grassland is desert. It is, however, notable that the soil of the grassland ecosystem is rich in nutrients and organic matter and provides the largest biomass.


 

Question 3

Which one of the following is the correct sequence of ecosystems in the order of decreasing productivity?

(a) Oceans, lakes, grasslands, mangroves

(b) Mangroves, oceans, grasslands, lakes

(c) Mangroves, grasslands, lakes, oceans

(d) Oceans, mangroves, lakes, grasslands

Ans: c

Sub-Theme: Energy Flow

Option (c) is correct:

  • The average ocean productivity is about 50 grams carbon per square metre per year;
  • The average land productivity is 160 grams carbon per square metre per year;
  • Salt marshes and mangroves have one of the highest productivity of 3300-6000 grams of carbon per square metre per

Productivity:

  • Productivity is the rate of biomass
  • Productivity is measured in mass per unit volume per unit time.
  • The quantity of biomass or organic matter created by plants during photosynthesis per unit area during a certain time is referred to as primary production.
  • Gross primary productivity of an ecosystem is the rate of production of organic matter during photosynthesis.
    • A considerable amount of GPP is utilized by plants in respiration.
    • Gross primary productivity minus respiration losses (R), is the net primary productivity (NPP). GPP – R = NPP.
  • Secondary productivity is the rate at which consumers create new organic matter.

 

Question 4

With reference to food chains in ecosystems, consider the following statements:

  1. A food chain illustrates the order in which a chain of organisms feeds upon each other.

  2. Food chains are found within the populations of a species.

  3. A food chain illustrates the numbers of each organism which are eaten by others.

Which of the statements given above is/are correct?

(a) 1 only

(b) 1 and 2 only

(c) 1, 2 and 3

(d) None

Ans: a

Sub-Theme: Food Chain

Food Chain:

  • A food chain is the successive transfer of nutrients and energy from one creature to
  • A food chain shows a straightforward and isolated relationship and only one component of the food or energy flow.
  • g. Grasses → Grasshopper → Frog → Snake→ Hawk/Eagle.
  • A trophic level is the name given to each step in the food chain.
  • The place an organism holds in a food chain determines its trophic level.
  • Top carnivores are at the end of a food chain, which begins with producers.
  • Types of Food Chains: 1) Grazing food chain and 2) Detritus food chain
  • Grazing food chain: The consumers which start the food chain, utilising the plant or plant part as their food, constitute the grazing food chain.
  • Detritus food chain: This type of food chain starts from organic matter of dead and decaying animals and plant bodies from the grazing food Dead organic matter or detritus feeding organisms are called detritivores or decomposers.
  • Statement 1 is correct: A food chain is a linear flow of energy and nutrients from one organism to another.

Statement 2 is incorrect: Food chains are found within the populations of a species but not among all species (Man won’t eat man).

Statement 3 is incorrect: A food web illustrates the numbers of each organism which are eaten by others, not the food chain.

 

Question 5

Which one of the following terms describes not only the physical space occupied by an organism but also its functional role in the community of organisms?

(a) Ecotone

(b) Ecological niche

(c) Habitat

(d) Home range

Ans: b

Sub-Theme: Ecological community

Option (b) is the correct answer: Ecological niche describes not only the physical space occupied by an organism but also its functional role in the community as well.

Ecological Niche:

  • The term niche was first time used by Grinnel (1917) to explain microhabitats.
  • The physical space occupied by the organism, its functional role in the community i.e., trophic position, its position in the environmental gradients of temperature, moisture, pH, soil etc. and the conditions of
  • Therefore, Niche is the sum of all the activities and relationships of a species by which it uses the resources in its habitat for its survival and reproduction.
  • No two species have exactly the same Different types of niches are: Habitat niche, Food niche, Reproductive niche, Physical and Chemical niche.

Option (a) is incorrect: Ecotone is a zone of transition between two ecosystems such the mangrove forests represent an ecotone between marine and terrestrial ecosystems, etc.

Option (b) is correct: Ecological niche subsumes all of the interactions between a species and the biotic and abiotic environment, and thus represents a very basic and fundamental ecological concept.

Option (c) is incorrect: A habitat is a place where an organism makes its home also it meets all the environmental conditions an organism needs to survive.

Option (d) is incorrect: A home range is an area in which an animal lives and moves on a daily or periodic basis (a little bigger than habitat – home

→ office → home).

 

Question 6

On the planet earth, most of the freshwater exists as ice caps and Out of the remaining freshwater, the largest proportion:

(a) is bound in atmosphere as moisture and clouds

(b) is found in freshwater lakes and rivers

(c) exists as groundwater

(d) exists as soil moisture

Ans: c

Sub-Theme: Water Conservation

Option (c) is the correct answer: Ground has the largest portion of freshwater after freshwater that exists as ice caps and glaciers.

Water Resources:

  • About 71% of the earth’s surface is covered with water.
  • Out of the total water present on the planet earth, the majority is in the form of seas and oceans (more than 97%) and is termed as saline water.
  • While the remaining water is termed as
  • The decreasing order of proportion of freshwater in different forms: snow caps, icebergs and glaciers → groundwater → soil moistures → freshwater lakes → atmosphere→ rivers.

NOTE: This is a basic, easy and direct question from NCERT. Also, same question was repeated in Pre 2021.

 

Question 7

Which of the following can be found as pollutants in the drinking water in some parts of India?

  1. Arsenic

  2. Sorbitol

  3. Fluoride

  4. Formaldehyde

  5. Uranium

Select the correct answer using the codes given below.

(a) 1 and 3 only

(b) 2, 4 and 5 only

(c) 1, 3 and 5 only

(d) 1, 2, 3, 4 and 5

Ans: c

b-Theme: Water Pollution

Option (c) is the correct answer: Arsenic, Fluoride and Uranium are found as pollutants in the drinking water.

Water Pollution:

  • Water pollution is the contamination of water bodies, usually as a result of human activities.
  • Point pollution sources, where the source is a well-defined location.
  • Non-point pollution sources, which are spread over larger areas.
  • Causes: Agricultural sources, Sewage water, Industrial effluents, Thermal Pollution, Oil- spills, ships & other economic activities.
  • Effects: Decline on Dissolved oxygen, Increase in BOD, Death of Aquatic species, Eutrophication etc.

Options 1, 3 and 5 are correct: Arsenic, Fluoride, Iron, Uranium, Nitrate, Salinity are some major groundwater pollutants in India.

Option 2 is incorrect: Sorbitol is a sugar alcohol found in fruits and plants with diuretic, laxative and cathartic properties.

Option 4 is incorrect: Formaldehyde is used to preserve fish longer. The use of this chemical is banned in fresh floods by the Food Safety and Standards Authority of India.

 

Question 8

Due to improper/indiscriminate disposal of old and used computers or their parts, which of the following are released into the environment as e-waste?

  1. Beryllium

  2. Cadmium

  3. Chromium

  4. Heptachlor

  5. Mercury

  6. Lead

  7. Plutonium

Select the correct answer using the codes given below:

(a) 1, 3, 4, 6 and 7 only

(b) 1, 2, 3, 5 and 6 only

(c) 2, 4, 5 and 7 only

(d) 1, 2, 3, 4, 5, 6 and 7

 

Ans: b

Sub-Theme: Waste Management

E-WASTE:

  • E-waste is any electrical or electronic equipment that’s been discarded.
  • India’s first e-waste clinic – at Bhopal.
E-waste Sources Constituents
PCBs, glass panels, and Computer monitors Lead
Resistors & Semiconductors Cadmium
Relays and switches, & PCBs Mercury
Galvanised steel plates & decorator or hardener Chromium
Cabling, Computer & housing Plastics & PVC
Electronic equipments & circuit boards Brominated flame-

retardants

Front panels of CRTs Barium, Phosphorus & Heavy metals
Copper wires, PCB tracks Copper
Nickel Cadmium batteries Nickel
Lithium-ion battery Lithium
Motherboards Beryllium

Options 1,2,3,5 and 6 are correct: All elements in these options are the harmful constituents that are released into the environment due to improper disposal of e-waste.

Option 4 is incorrect: Heptachlor is used as pesticides and also comes under the 12 initial POPs under the Stockholm Convention.

Option 7 is incorrect: Plutonium is a radioactive material.

 

 

Question 9

Acid rain is caused by the pollution of environment by

(a) Carbon dioxide and nitrogen

(b) Carbon monoxide and carbon dioxide

(c) Ozone and carbon dioxide

(d) Nitrous oxide and sulphur dioxide

Ans: d

Sub-Theme: Air Pollution

Option (d) is the correct answer: Nitrous oxide and Sulphur dioxide cause Acid rain.

  • Normal rain has a pH of about 5.6 when pH falls below this; it is called Acid rain (pH 4.2 to 4.4).
  • Acid rain results when Sulphur Dioxide (SO2) & Nitrogen Oxides (NOx) react with water, oxygen & other chemicals to form Sulfuric & Nitric acids.
  • NOx + SO2 + Moisture + other chemicals →Acid Rain (HNO3 + H2SO4)
  • Effects: Harms microorganisms in the soil; inhibits the activity of nitrogen fixation bacteria; soil acidification; ocean acidification; affects the growth of plants; food chain; kills aquatic animals; corrosion of metals & weathering of stone buildings & statues.

Common Air Pollutants and their Sources

Pollutant Sources
 

Particulate matter (PM)

Vehicles, power plants, construction activities, oil refinery, railway yard, industries, etc.
Nitrogen dioxide (NO2) Emissions     from      combustion processes
Sulphur dioxide (SO2) Burning of fossil fuels, power plants, metals processing and smelting facilities, vehicles
 

Ozone (O3)

Results from photochemical reactions b/w NOx & VOCs in presence of sunlight.

 

Question 10

Photochemical smog is a resultant of the reaction among

(a) NO2, O3 and peroxyacetyl nitrate in the presence of sunlight

(b) CO, O2 and peroxyacetyl nitrate in the presence of sunlight

(c) CO, CO2 and NO2 at low temperature

(d) High concentration of NO2, O3 and CO in the evening

Ans: a

Sub-Theme: Air Pollution

Option (a) is the correct answer: NO2, O3 and peroxyacetyl nitrate react in the presence of sunlight and Photochemical Smog (summer smog) is formed.

  • Photochemical Smog (summer smog) forms when pollutants such as nitrogen oxides (primary pollutant) and organic compounds (primary pollutants) react together in the presence of SUNLIGHT. A gas called OZONE (Secondary pollutant) is formed.

 

Question 11

Improper handling and storage of cereal grains and oilseeds result in the production of toxins known as aflatoxins which are not generally destroyed by the normal cooking process. Aflatoxins are produced by:

(a) bacteria

(b) protozoa

(c) moulds

(d) viruses

Ans: c

Sub-Theme: General Science

Option (c) is the correct answer: Fungi (moulds) produce Aflatoxins.

  • Aflatoxins are poisonous substances produced by certain kinds of fungi (moulds) such as Aspergillus flavus and Aspergillus
  • These fungi (moulds) can be found naturally all over the world; they can contaminate food crops and pose a serious health threat to humans and livestock.
  • Children who are exposed to these toxins may experience stunted growth and have a higher risk of developing liver cancer.
  • High temperatures and humid climates or stressful conditions like drought make the conducive environment for these fungi to produce aflatoxin.
  • Aflatoxins also pose a significant economic burden, causing an estimated 25% or more of the world’s food crops to be destroyed

 

Question 12

With reference to the usefulness of the by products of the sugar industry, which of the following statements is/are correct?

  1. Bagasse can be used as biomass fuel for the generation of energy.

  2. Molasses can be used as one of the feedstocks for the production of synthetic chemical fertilizers.

  3. Molasses can be used for the production of ethanol.

Select the correct answer using the codes given below:

(a) 1 only

(b) 2 and 3 only

(c) 1 and 3 only

(d) 1, 2 and 3

Ans: c

Sub-Theme: Byproduct of Sugar Industry

Bagasse and Molasses

Introduction Uses
 

 

 

Bagasse

•   The remnant left after extracting the juice of the sugarcane or sorghum is known as bagasse. It is a dry pulpy fibrous material. •   It is used for the generation of heat energy and electricity. Ethanol produced from it is also used as a popular fuel in some countries.

•   It substitutes wood to produce pulp, paper, and board.

•   As a cattle feed when mixed with molasses and enzymes and then fermenting.

 

 

 

Molasses

•   It is the thick syrup that results from the refining of sugarcane or sugar beets into sugar.

•   It contains a good amount of Vitamin B6 and minerals like calcium, manganese, iron and magnesium.

•   Sweetening and flavouring foods.

•   In the production of fine commercial brown sugar.

•   In baking and cooking.

•   Producing ethanol and is the prime ingredient in the distillation of few alcoholic beverages.

•   As a cattle feed.

 

Statement 1 is correct: Bagasse can be used for the generation of heat energy and electricity.

Statement 2 is incorrect: Molasses is not used for the production of chemical fertilizers.

Statement 3 is correct: Molasses can be used for the production of ethanol and it is also used as the prime ingredient in the distillation of a few alcoholic beverages.

 

Question 13

Consider the following fauna and India:

  1. Gharial

  2. Leatherback turtle

  3. Swamp deer

Which of the above is/are endangered?

(a) 1 and 2 only

(b) 3 only

(c) 1, 2 and 3

(d) None

Ans: c         

Sub-Theme: Species

Option (c) is the correct answer: All the above- mentioned species are endangered. Gharials, sometimes called gavials, are a type of Asian crocodilian distinguished by their long, thin snouts.

 India has three species of Crocodilians namely:

  • Gharial (Gavialis gangeticus): IUCN Red List- Critically Endangered
  • Mugger crocodile (Crocodylus palustris):

IUCN- Vulnerable.

  • Saltwater  crocodile  (Crocodylus porosus): IUCN- Least Concern.
  • All the three are listed: Appendix I of CITES and Schedule I of the Wild Life (Protection) Act, 1972.
  • Exception: Saltwater Crocodile populations of Australia, Indonesia and Papua New Guinea are included in Appendix II of CITES.

 

Question 14

Consider the following:

  1. Star tortoise

  2. Monitor lizard

  3. Pygmy hog

  4. Spider monkey

Which of the above are naturally found in India?

(a) 1, 2 and 3 only

(b) 2 and 3 only

(c) 1 and 4 only

(d) 1, 2, 3 and 4

Ans: a

Sub-Theme: Species

Statements 1, 2 and 3 are correct: Star tortoise, Monitor lizard and Pygmy hog are naturally found in India.

Statement 4 is incorrect: Spider Monkey is found in the tropical forests of Central and South America. It is critically endangered as per the IUCN Red List.

 

Question 15

Consider the following animals:

  1. Sea cow

  2. Sea horse

  3. Sea lion

Which of the above is/are mammal/mammals?

(a) 1 only

(b) 1 and 3 only

(c) 2 and 3 only

(d) 1, 2 and 3

Ans: b

Sub-Theme: Species Diversity

Statements 1 and 3 are correct: Both the Sea Cow or Dugong and Sea Lion are mammals. Dugong also called ‘Sea Cow’ is the only existing species of herbivorous mammal that lives exclusively in the sea including in India. Dugongs are an important part of the marine ecosystem and their depletion will have effects all the way up the food chain. They are found in over 30 countries and in India are seen in the Gulf of Mannar, Gulf of Kutch, Palk Bay, and the Andaman and Nicobar Islands. IUCN Red List status: Vulnerable. Sea Lion belongs to the family Otariidae of class Mammalia. They are carnivorous aquatic mammals, hence the name Sea Lion.

Statement 2 is incorrect: Sea Horse is a bony fish (Osteichthyes). Seahorses are mainly found in shallow tropical and temperate saltwater throughout the world, from about 45°S to 45°N. They live in sheltered areas such as seagrass beds, estuaries, coral reefs, and mangroves. Four species are found in Pacific waters from North America to South America.

 

Question 16

Consider the following organisms:

  1. Agaricus

  2. Nostoc

  3. Spirogyra

Which of the above is/are used as biofertilizer/ biofertilizers?

(a) 1 and 2

(b) 2 only

(c) 2 and 3

(d) 3 only

Ans: b         

Sub-Theme: Species helps in agriculture

Statement 1 is incorrect: Bio-fertilizers are living or biologically active products or microbial inoculants of bacteria, algae and fungi (separately or in combination) which are able to enrich the soil with nitrogen, phosphorus, organic matter etc. Agaricus is a genus of mushrooms containing both edible and poisonous species, it is not a biofertilizer.

Statement 2 is correct: Nostoc fixes atmospheric nitrogen into ammonia, which may then be used or converted to a form suitable for plant growth. Thus demonstrating Nostoc’s potential as a sustainable biofertilizer.

Statement 3 is incorrect: Spirogyra is not generally used as a biofertilizer, instead if you use it; it would compete with the plants.

 

Question 17

In which of the following States is lion-tailed macaque found in its natural habitat?

  1. Tamil Nadu

  2. Kerala

  3. Karnataka

  4. Andhra Pradesh

Select the correct answer using the codes given below:

(a) 1, 2 and 3 only

(b) 2 only

(c) 1, 3 and 4 only

(d) 1, 2, 3 and 4

Ans: a          

Sub-Theme: Species

Option (a) is the correct answer: Lion-tailed Macaque is endemic to the Western Ghats in the states of Karnataka, Kerala and Tamil Nadu.

Although the species has a relatively wide range, its area of occupancy is small and severely fragmented. Primarily diurnal arboreal, it prefers the upper canopy of primary tropical evergreen rainforest. It can also be found in monsoon forests in hilly country and in disturbed forest.

 

2012

 

Question 1

The Millennium Ecosystem Assessment describes the following major categories of ecosystem services— provisioning, supporting, regulating, preserving and cultural.

Which one of the following is a supporting service?

(a) Production of food and water

(b) Control of climate and disease

(c) Nutrient cycling and crop pollination

(d) Maintenance of diversity

Ans: c

Sub-Theme:

Option (c) is the correct answer: Nutrient cycling and crop pollination are supporting services under Millennium Ecosystem Assessment.

Millennium Ecosystem Assessment (MA):

  • The Millennium Ecosystem Assessment (MA) was initiated in 2001 by the United
  • The objective of the MA was to evaluate how changing ecosystems might affect human well-being as well as the scientific foundation for any necessary action to improve the systems’ conservation and sustainable usage, as well as their contribution to human well-being.
  • Ecosystem services to the society in the form of –
    1. Provisioning services: food, raw materials, genetic resources, water, etc
    2. Regulating services: carbon sequestration and climate regulation
    3. Cultural services: tourism and religion
    4. Supporting services: that are necessary for the production of all other ecosystem services such as nutrient recycling and soil formation, etc.

 

Question 2

What would happen if phytoplankton of an ocean is completely destroyed for some reason?

1. The ocean as a carbon sink would be adversely affected.

2. The food chains in the ocean would be adversely affected.

3. The density of ocean water would drastically decrease.

Select the correct answer using the codes given below:

(a) 1 and 2 only

(b) 2 only

(c) 3 only

(d) 1, 2 and 3

Ans: a

Sub-Theme: Food Chain

Statement 1 is correct: Phytoplankton prefer iron and iron pulls carbon out of the atmosphere during photosynthesis, thus the complete destruction of phytoplankton would certainly affect the carbon sinking mechanism of the ocean.

Statement 2 is correct: In a balanced ecosystem, phytoplankton provide food for a wide range of sea creatures, thus Phytoplankton are considered the primary producers in the marine food chain, they are called the ‘grass of the sea’. Thus, the destruction of phytoplankton will certainly have an adverse effect on the marine food chain.

Statement 3 is incorrect: First of all the use of extreme words like ‘drastically’ in the statement is itself a red flag. Now coming to the main part of the statement, if the phytoplankton of an ocean is completely destroyed for some reason, then it may affect the marine food chain because when the phytoplankton disappeared, that affects the zooplankton, which then affects the small fish that ate the zooplankton, and the large fish that ate the small fish. So it’s like dominoes falling. Phytoplankton and macroalgae → zooplankton → small fish → large fish. Therefore, it may hardly affect the density of water.

 

Question 3

How does the National Biodiversity Authority (NBA) help in protecting Indian Agriculture?

1. NBA checks the biopiracy and protects the indigenous and traditional genetic resources.

2. NBA directly monitors and supervises the scientific research on genetic modification of crop

3. Application for Intellectual Property Rights related to genetic/biological resources cannot be made without the approval of NBA.

Which of the statements given above is/are correct?

(a) 1 only

(b) 2 and 3 only

(c) 1 and 3 only

(d) 1, 2 and 3

Ans: c

Sub-Theme: Conservation Effort National Biodiversity Authority(NBA):

  • Established in 2003 with HQ in Chennai under the ambit of MoEF&CC
  • As per the Biological Diversity Act of 2002, NBA is a statutory body.
  • The act was passed to put the Convention on Biological Diversity (CBD) into effect, which was signed by India in 1992.
  • Mandate: to perform regulatory & advisory functions for the GoI on issues of conservation, sustainable use of biological
  • NBA supports the creation of State Biodiversity Boards (SBBs).
  • Biodiversity Heritage Sites: It is notified by the State Governments in consultation with local bodies.

Statements 1 and 3 are correct:

  • Two Important Functions of the NBA, inter alia:
  • Advising the central government, regulating activities and issuing guidelines for access to biological resources and for fair and equitable benefit sharing in accordance with the Biological Diversity Act, 2002.
  • Taking necessary measures to oppose the grant of intellectual property rights in any country outside India on any biological resource obtained from India or knowledge associated with such biological resources derived from India illegally.

Statement 2 is incorrect: Genetic Engineering Appraisal Committee (GEAC) monitors and supervises the scientific research on genetic modification of crop plants.

 

Question 4

Consider the following agricultural practices:

  1. Contour bunding

  2. Relay cropping

  3. Zero tillage

In the context of global climate change, which of the above helps/help in carbon sequestration/storage in the soil?

(a) 1 and 2 only

(b) 3 only

(c) 1, 2 and 3

(d) None of them

Ans: b

Sub-Theme: Agriculture practices

Option (b) is the correct answer: Zero Tillage helps in carbon sequestration/storage in the soil. Zero Tillage:

  • It is the process where the crop seed will be sown through drillers without prior land preparation and distributed in the soil where previous crop stubbles are present.
  • Zero tillage not only reduces the cost of cultivation it also reduces the soil erosion, crop duration, irrigation requirement and weed effect which is better than tillage. It also helps in carbon sequestration in the soil.

Contour bundling:

  • Contour bundling involves the construction of banks along the contours.
  • Terracing and contour bunding divide the hill slope into numerous small slopes, check the flow of water, promote absorption of water by soil and save soil from erosion.
  • Retaining walls of terraces control the flow of water and help in reducing soil erosion.

Relay Cropping:

  • Relay Cropping is the process of growing one crop, then planting another crop (usually a cover crop) in the same field before harvesting the first.
  • This helps avoid competition between the main crop and the intercrop.
  • It also uses the field for a longer time since the cover crop usually continues to grow after the main crop is harvested.

 

Question 5

Consider the following statements:

Chlorofluorocarbons, known as ozone-depleting

substances, are used:

  1. In the production of plastic foams

  2. In the production of tubeless tyres.

  3. In cleaning certain electronic components.

  4. As pressurizing agents in aerosol cans.

Which of the statements given above is/are correct?

(a) 1, 2 and 3 only

(b) 4 only

(c) 1, 3 and 4 only

(d) 1, 2, 3 and 4

Ans: c

Sub-Theme: Ozone Depleting Substances Ozone Depleting Substances: Chlorofluorocarbons, Hydrochlorofluorocarbons, Carbon tetrachloride, Methyl chloroform, Methyl chloroform, Halons, Methyl bromide, etc.

Chlorofluorocarbons (CFCs):

  • Chlorofluorocarbons, also referred to as ozone depleting materials, are used to make plastic foams, clean electronic components, and pressurise aerosol
  • They are non-toxic, non-flammable chemicals containing atoms of carbon, chlorine and fluorine.
  • Uses Refrigeration, solvents, insulation foams, aero propellants, industrial and commercial uses.

Statements 1,3 and 4 are correct: Chlorofluoro- carbons, known as ozone-depleting substances, are used in the production of plastic foams, in cleaning electronic components, and as pressurising agents in aerosol cans.

Statement 2 is incorrect: Tubeless tyre is basically a clincher tyre inflated onto a rim with no inner tube. Instead of an inner tube holding the air pressure, an airtight chamber is created with a tubeless-specific tyre, developed with a special (commonly carbon) bead, and a compatible rim.

 

Question 6

The increasing amount of carbon dioxide in the air is slowly raising the temperature of the atmosphere, because it absorbs

(a) The water vapour of the air retains its

(b) The ultraviolet part of the solar

(c) All the solar

(d) The infrared part of the solar

Ans: d

Sub-Theme: Global Warming

Option (d) is the correct answer: The increasing amount of carbon dioxide in the air is slowly raising the temperature of the atmosphere because it absorbs the infrared part of the solar radiation.

Infrared Radiation:

  • It absorbs energy with a wavelength of 15 μm (micrometres).

Greenhouse Effect:

  • A naturally occurring phenomena that warms and covers the lower atmosphere of the planet, keeping it at a temperature where life may survive.
  • The earth’s cooling and warming are balanced by them.
  • Without this, the current 15 degrees Celsius average temperature of the earth’s surface would be -19 degrees Celsius.
  • Human-induced greenhouse gas emissions upset the natural balance & lead to increased warmth.

 

Question 7

Which of the following can be threats to the biodiversity of a geographical area?

  1. Global warming

  2. Fragmentation of habitat

  3. Invasion of alien species

  4. Promotion of vegetarianism

Select the correct answer using the codes given below:

(a) 1, 2 and 3 only

(b) 2 and 3 only

(c) 1 and 4 only

(d) 1, 2, 3 and 4

Ans: a

Sub-Theme: Biodiversity Loss

Option (a) is the correct answer: Global Warming, Habitat Loss/Fragmentation and Invasive Alien Species threats to the biodiversity of a geographical area.

  • Mammals and birds that require large territories as well as animals with migratory habitat are adversely affected, which results in population Habitat loss is caused by pollution. Large habitats are broken up into small fragments as a result of various human activities.
  • Invasive alien species are those that appear outside of their normal habitat.
  • Co-extinctions: When a species becomes extinct, the plant and animal species associated with it in an obligatory way also become extinct.
  • Overexploitation: Humans have traditionally relied on nature for food and shelter, but when “need” transforms into “greed,” this relationship breaks down. Overexploitation results from it.

Options 1,2 and 3 are correct: Global Warming, Fragmentation of Habitat, Invasion of Alien Species, Co- extinctions are the potential threats to biodiversity.

Option 4 is incorrect: Promotion of vegetarianism can’t be a cause for the loss or threat of biodiversity.

 

Question 8

The National Green Tribunal Act, 2010 was enacted in consonance with which of the following provisions of the Constitution of India?

1. Right to healthy environment, construed as a part of Right to life under Article 21.

2. Provision of grants for raising the level of administration in the Scheduled Areas for the welfare of Scheduled Tribes under Article 275(1).

3. Powers and functions of Gram Sabha as mentioned under Article 243(a).

Select the correct answer using the codes given below:

(a) 1 only

(b) 2 and 3 only

(c) 1 and 3 only

(d) 1, 2 and 3

Ans: b

Option (b) is the correct answer: National- Green Tribunal (NGT) Act, 2010 was enacted for ensuring a healthy environment, construed as a part of Right to life under Article 21.

National-Green Tribunal (NGT) Act, 2010

  • Objective: For efficient & quick resolution of environmental disputes (within 6 months of appeal); To lessen the burden of litigation in the higher courts.
  • The Act establishes NGT & it has jurisdiction over all civil cases involving substantial questions relating to the environment.
  • Provides for Enforcement of legal environmental rights, relief & compensation for damages caused.
  • Tribunal is guided by principles of natural justice & its order is executable as a decree of a civil court.
  • Although the NGT’s orders are final, they may be contested in the SC within 90 days.
  • NGT has four regional benches in Pune, Bhopal, Chennai, and Kolkata in addition to its principal bench in New Additionally, a system for circuit benches is present.
  • The chairperson of the NGT is a retired judge of The Supreme Court.

Statement 1 is correct: The National Green Tribunal Act, 2010, draws inspiration from India’s constitutional provision of (Constitution of India/ Part III) Article 21 Protection of life and personal liberty, which assures the citizens of India the right to a healthy environment.

Statements 2 and 3 are incorrect: These statements have no relation to NGT.

 

Question 9

Consider the following protected areas:

  1. Bandipur

  2. Bhitarkanika

  3. Manas

  4. Sunderbans

Which of the above are declared Tiger Reserves?

(a) 1 and 2 only

(b) 1, 3 and 4 only

(c) 2, 3 and 4 only

(d) 1, 2, 3 and 4

Ans: b

Sub-Theme: National Parks/ Tiger Reserve/ Biosphere Reserve/Wildlife Sanctuary/Wetland

Option (b) is correct: List of Tiger Reserve

State Tiger Reserves
Arunachal Pradesh Namdapha; Kamlang Tiger Reserve; Pakke
Assam Manas; Nameri; Orang Tiger Reserve; Kaziranga
Andhra Pradesh Nagarjunasagar Srisailam
Bihar Valmiki
 

Chattisgarh

Udanti-Sitanadi;                Achanakmar; Indravati; Guru Ghasidas National Park and Tamor Pingla Wildlife Sanctuary (latest)
Jharkhand Palamau
 

Karnataka

Bandipur; Bhadra; Dandeli-Anshi; Nagarhole; Biligiri Ranganatha Temple
Kerala Periyar; Parambikulam
 

Madhya Pradesh

Kanha; Pench (contiguous with Maharashtra); Bandhavgarh; Panna; Satpura; Sanjay-Dubri
 

Maharashtra

Pench (contiguous with Madhya Pradesh); Melghat; Tadoba-Andhari; Sahyadri; Nawegaon-Nagzira; Bor
Mizoram Dampa
Odisha Simlipal; Satkosia
 

Rajasthan

Ranthambore; Sariska; Mukundra Hills; Ramgarh Vishdhari Wildlife Sanctuary
 

Tamil Nadu

Kalakad-Mundanthurai; Annamalai Mudumakai;              Sathyamangalam; Srivilliputhur Megamalai
Telangana Kawal; Amrabad
 

Uttar Pradesh

Dudhwa; Pilibhit; Amangarh (Buffer of Corbett TR)
Uttarakhand Corbett; Rajaji
West Bengal Sundarban; Buxa

 

Question 10

In which one among the following categories of protected areas in India are local people not allowed to collect and use the biomass?

(a) Biosphere Reserves

(b) National Parks

(c) Wetlands declared under Ramsar Convention

(d) Wildlife Sanctuaries

Ans: b

Sub-Theme: National Parks/ Tiger Reserve/ Biosphere Reserve/Wildlife Sanctuary/Wetland

Option (a) is incorrect: A biosphere reserve is an area of land or water that is protected by law in order to support the conservation of ecosystems, as well as the sustainability of mankind’s impact on the environment.

Option (b) is correct: No human activity is permitted inside the national park except for the ones permitted by the Chief Wildlife Warden of the state.

Option (c) is incorrect: It is an international treaty for “the conservation and sustainable use of wetlands”.

Option (d) is incorrect: The difference between a Sanctuary and a National Park mainly lies in the vesting of rights of people living inside. Unlike National Parks, certain rights can be allowed in Sanctuary.

 

Question 11

Consider the following kinds of organisms

  1. Bat

  2. Bee

  3. Bird

Which of the above is/are pollinating agent/agents?

(a) 1 and 2 only

(b) 2 only

(c) 1 and 3 only

(d) 1, 2 and 3

Ans: d         

Sub-Theme: Species/Pollination

Option (d) is the correct answer: When a pollen grain moves from the anther (male part) of a flower to the stigma (female part), pollination happens and it is the first step in a process that produces seeds, fruits, and the next generation of plants. Pollinators are Vectors that move pollen within the flower and from flower to flower are called pollinators.

  • There are two categories of pollinators:
    • Invertebrate pollinators: Include bees, moths, flies, wasps, beetles and butterflies.
    • Vertebrate     pollinators:  Include monkeys, rodents, lemurs, tree squirrels, bats and birds.

Pollinators (Bat, Bee, Birds)

  • Vectors that move pollen within the flower and from flower to flower are called pollinators.
  • They visit flowers to drink nectar or feed off of pollen and transport pollen grains as they move from spot to spot.

 

Question 12

The Government of India encourages the cultivation of ‘sea buckthorn’. What is the importance of this plant:

  1. It helps in controlling soil erosion and in preventing desertification.

  2. It is a rich source of biodiesel.

  3. It has nutritional value and is well-adapted to live in cold areas of high altitudes.

  4. Its timber is of great commercial

Which of the statements given above is/are correct?

(a) 1 only

(b) 2, 3 and 4 only

(c) 1 and 3 only

(d) 1, 2, 3 and 4

Ans: c

Sub-Theme: Species

Option (c) is the correct answer: Seabuckthorn (soil binding, rich in vitamins)

  • It’s a shrub which produces an orange- yellow coloured edible berry.
  • It can be found in India above treeline in the Himalayan region, usually in arid places like the cold deserts of Ladakh and Spiti.
  • It grows naturally in Lahaul, Spiti, and some regions of Kinnaur in Himachal Pradesh where it is known as chharma.

Statements 1 and 3 are correct: Seabuckthorn is a soil-binding plant which prevents soil-erosion, it also has commercial value, as it is used in making juices, jams, nutritional capsules etc.

Statement 2 is incorrect: Though seabuckthorn is an important source of fuelwood and fodder, it is not a rich source of biodiesel.

Statement 4 is incorrect: Seabuckthorn has commercial value, as it is used in making juices, jams, nutritional capsules etc.

 

Question 13

Which one of the following groups of animals belongs to the category of endangered species?

(a) Great Indian Bustard, Musk Deer, Red Panda and Asiatic Wild Ass

(b) Kashmir Stag, Cheetal, Blue Bull and Great Indian Bustard

(c) Snow Leopard, Swamp Deer, Rhesus Monkey and Saras (Crane)

(d) Lion-tailed Macaque, Blue Bull, Hanuman Langur and Cheetal

Ans: a

Sub-Theme: Threatened Species

Threatened Species: Great Indian Bustard, Musk Deer, Red Panda and Asiatic Wild Ass (Currently Vulnerable, but as the question is from 2012, we will consider it as Endangered) all are Endangered Species.

Option (a) is the correct answer: Great Indian Bustard, Musk Deer, Red Panda and Asiatic Wild Ass (Currently Vulnerable, but as the question is from 2012, we will consider it as Endangered) all are Endangered Species. Important species and their Conservation status are:

Species Particulars
 

 

 

 

 

 

 

Great Indian Bustard

•   State bird of Rajasthan

•   Flagship grassland species, representing the health of the grassland ecology.

•   States: Rajasthan and Gujarat. Small populations occur in Maharashtra, Karnataka and Andhra Pradesh.

•   Conservation Status:

•   IUCN Status: Critically Endangered

•   CITES: Appendix 1

•   Convention        on        Migratory Species (CMS): Appendix I

•   Wildlife (Protection) Act, 1972:

Schedule 1

 

 

Musk Deer

•   Habitat: J&K, Uttarakhand. Himachal Pradesh, Sikkim and Arunachal Pradesh.

•   Conservation Status:

•   IUCN Status: Endangered

 

 

 

 

 

 

Red Panda

•   State animal of Sikkim.

•   Red  panda  is  endemic  to the temperate forests of the Himalayas.

•   Habitat: Sikkim and Assam, northern Arunachal Pradesh.

•   Threats: habitat loss and fragmentation, poaching, and inbreeding depression.

•   Conservation Status:

•   IUCN Status: Endangered

•   Wildlife (Protection) Act, 1972:

Schedule 1

•   Scientific Name: Equus hemionus
•   Also called: Asiatic Wild Ass, Asian Wild Ass, Indian onager or, Ghudkhur and Khur (Gujarati language)
 

Asiatic Wild Ass

•   A subspecies of the onager native to Southern Asia.

•   Habitat: Western India, southern Pakistan (i.e. provinces of Sindh and Baluchistan), Afghanistan, and south-eastern Iran.

•   Conservation Status:
•   IUCN: currently listed as Near Threatened (Earlier it was listed under Endangered Category)

 

Question 14

What is the difference between the antelopes Oryx and Chiru?

(a) Oryx is adapted to live in hot and arid areas whereas Chiru is adapted to live in steppes and semi-desert areas of cold high mountains.

(b) Oryx is poached for its antlers whereas Chiru is poached for its musk.

(c) Oryx exists in western India only whereas Chiru exists in north-east India only.

(d) None of the statements (a), (b) and (c) given above is correct.

Ans: a

Sub-Theme: Species

Option (a) is the correct answer: Chiru, also Known as Tibetan antelope. Endemic to the Tibetan Plateau, the Tibetan antelope inhabits open alpine and cold steppe environments between 3,250 and 5,500 m (10,660 and 18,040 ft) elevation.

IUCN: Near Threatened.

Oryx is a genus consisting of four large antelope species called oryxes.

Oryx species prefer near-desert conditions and can survive without water for long periods.

IUCN: In 2011 it was down-listed from the Endangered category to Vulnerable. Hence, Oryx species prefer near-desert conditions and can survive without water for long periods and Chiru or Tibetan antelope inhabit open alpine and cold steppe environments.

 

Question 15

Consider the following:

  1. Black-necked crane

  2. Cheetah

  3. Flying squirrel

  4. Snow leopard

Which of the above are naturally found in India?

(a) 1, 2 and 3 only

(b) 1, 3 and 4 only

(c) 2 and 4 only

(d) 1, 2, 3 and 4

Ans: b 

Sub-Theme: Species

Statement 1 is correct: Black-necked crane is a state bird of Ladakh, therefore it ought to be naturally found in India at high altitude.

Statement 2 is incorrect: The Cheetahs are believed to have disappeared from the country when Maharaja Ramanuj Pratap Singh Deo of Koriya hunted and shot the last three recorded Asiatic cheetahs in India in 1947.

Statement 3 is correct: Various species of Flying squirrel are naturally found in India.

Statement 4 is correct: In India, Snow Leopard’s geographical range encompasses through Western Himalayas: Jammu and Kashmir, Himachal Pradesh. Eastern Himalayas: Uttarakhand and Sikkim and Arunachal Pradesh. The Snow Leopard capital of the world is Hemis, Ladakh.

 

2011

 

Question 1

In the context of ecosystem productivity, marine upwelling zones are important as they increase marine productivity by bringing the:

  1. Decomposer microorganisms to the surface.

  2. Nutrients to the surface.

  3. Bottom-dwelling organisms to the surface.

Which of the statements given above is/are correct?

(a) 1 and 2

(b) 2 only

(c) 2 and 3

(d) 3 only

Ans: b

Sub-theme: Ecosystem productivity/Energy Flow

Option (b) is the correct answer: Marine upwelling brings nutrients to the surface.

Upwelling:

  • It is a process in which deep, cold water of the ocean rises toward the surface.
  • Winds blowing across the ocean surface pushes the water away. Water then rises up from beneath the surface to replace the water that was pushed away. This process is known as “upwelling.”
  • Upwelling is usually observed at the equator where surface waters diverge, in the vicinity of Antarctica where the east and west winds drift, and along coastlines where the wind blows in a way that the Ekman Transport pushes water away from the coast.
  • The water that upwelling causes to rise to the surface is usually cooler and nutrient-
  • Surface waters are “fertilised” by these nutrients, which means that they frequently have high biological As a result, good fishing spots are usually found where upwelling is frequent
  • Statement 1 and 3 are incorrect: At the upwelling zone no such movement of organisms can be observed, they would remain at their position.
  • Statement 2 is correct: Deep ocean water during upwelling is usually extremely cold and nutrient- rich. We frequently find very productive waters in upwelling locations as a result.

 

Question 2

If a tropical rainforest is removed, it does not regenerate quickly as compared to a tropical deciduous forest. This is because

(a) The soil of rainforest is deficient in nutrients

(b) Propagules of the trees in a rainforest have poor viability

(c) The rain forest species are slow-growing

(d) Exotic species invade the fertile soil of rainforest

Ans: a

Sub-Theme: Types of ecosystem/Forest/ Biomes

Option (a) is correct: The high volume of rainfall in tropical rainforests leaches out most of the nutrients from the soil and makes these soils virtually useless and nutrient deficient for agricultural purposes.

Option (b) is incorrect: Propagule is a vegetative structure that can become detached from a plant and give rise to a new plant, e.g. a bud, sucker, or spore. Seed bearing plants are more significant than Propagules in rainforests.

Option (c) is incorrect: In the rainforest, the plant species generally compete with each other for sunlight and while doing so they grow so fast that they rapidly consume the nutrients from the decomposed leaf litter. As a result, most of the nutrients are contained in the trees and other plants rather than in the soil.

Option (d) is incorrect: Though the exotic invasive species are a threat to rainforests (E.g. Most plantation crops like rubber, palm etc.) but it is also true that the rainforest soil is heavily leached, nutrient deficient, and thus less fertile.

 

Question 3

The “Red Data Books” published by the International Union for Conservation of Nature and Natural Resource (IUCN) contain lists of

  1. Endemic plant and animal species present in the biodiversity hotspots.

  2. Threatened plant and animal species.

  3. Protected sites for conservation of nature and natural resources in various countries.

Select the correct answer using the codes given below:

(a) 1 and 3

(b) 2 only

(c) 2 and 3

(d) 3 only

Ans: b

Sub-Theme: Biodiversity Conservation

Option (b) is the correct answer: Red Data Books contain lists of Threatened plant and animal species.

Statements 1 and 3 are incorrect: The IUCN Red List of “threatened species” is the world’s most comprehensive inventory of the global conservation status of plant and animal species, which is a grouping of three categories: Critically Endangered, Endangered, and Vulnerable.

Statement 2 is correct: The IUCN Red List Categories define the extinction risk of species assessed. Nine categories extend from NE (Not Evaluated) to EX (Extinct). Among these nine categories, Critically Endangered (CR), Endangered (EN) and Vulnerable (VU) species are considered to be threatened with extinction.

 

Question 4

Which one of the following is not a site for the in-situ method of conservation of flora?

(a) Biosphere Reserve

(b) Botanical Garden

(c) National Park

(d) Wildlife Sanctuary

Ans: b

Sub-Theme: Modes of Conservation

Option (b) is the correct answer: Botanical Garden

Ex situ Conservation In situ Conservation
•   Ex situ conservation is the preservation of biodiversity away from their natural habitats. Here, zoological or botanical parks are places where animals and plants are raised or grown. •   In situ conservation refers  to the

preservation of plants and animals in their natural environments.

•   Another method of ex situ conservation is the reintroduction of an animal or plant into its former environment. For instance, the extinct Gangetic gharial has been brought back into the rivers of Uttar Pradesh, Madhya Pradesh, and Rajasthan.

•   Important locations for ex situ conservation include seed banks, botanical, horticultural, and recreational gardens.

•   This includes the establishment of:

1.  National      parks and sanctuaries

2.  Biosphere reserves

3.  Nature reserves

4.  Reserved        and protected forests

5.  Preservation plots

6.  Reserved forests

 

Question 5

Three of the following criteria have contributed to the recognition of Western Ghats-Sri Lanka and Indo- Burma regions as hotspots of biodiversity:

  1. Species richness

  2. Vegetation density

  3. Endemism

  4. Ethno-botanical importance

  5. Threat perception

  6. Adaptation of flora and fauna to warm and humid conditions

Which three of the above are correct criteria in this context?

(a) 1, 2 and 6

(b) 2, 4 and 6

(c) 1, 3 and 5

(d) 3, 4 and 6

Ans: c

Sub-Theme: Biodiversity Hotspot

Biodiversity Hotspot:

  • Biodiversity hotspots are regions with high species richness and a high degree of
  • In 1988, the British biologist Norman Myers coined the term “biodiversity hotspot” as a biogeographic region characterized both by exceptional levels of plant endemism and by serious levels of habitat loss.
  • In 1989, Conservation International (CI) adopted Myers’ hotspots as its institutional model and provided the following qualifications for a hotspot:
    1. Species endemism: It includes at least 1,500 vascular plant species that are unique to the planet (endemic species).
    2. Degree of Threat: Primary native vegetation has lost at least 70% of its original coverage.

Statement 1 and 3 are correct: Species richness and endemism is one of the major criteria which must contain at least 1,500 species of vascular plants (> 0.5% of the world’s total) as endemics.

Statement 5 is correct: ‘Degree of Threat’ is also a major criteria for the recognition of biological hotspots. It has to have lost at least 70% of its original habitat. (It must have 30% or less of its original natural vegetation). In other words, it must be threatened.

Statement 2, 4 and 6 are incorrect: Are not the criteria for the recognition of Biodiversity Hotspots.

 

Question 6

Consider the following statements:

1. Biodiversity is normally greater in the lower latitudes as compared to the higher latitudes.

2. Along the mountain gradients, biodiversity is normally greater in the lower altitudes as compared to the higher altitudes.

Which of the statements given above is/are correct?

(a) 1 only

(b) 2 only

(c) Both 1 and 2

(d) Neither 1 nor 2

Ans: c

Sub-Theme: Biomes

Patterns of Biodiversity:

  • Latitudinal gradients: The diversity of plants and animals varies widely over the world.
  • The diversity of species declines as we move from the equator to the pole.
  • Than temperate or polar regions, the tropics are home to more species.

Statement 1 is correct: Diversity of plants and animals is not uniform throughout the world. Species diversity decreases as we move away from the equator towards the pole.

Statement 2 is correct: Elevational diversity gradient (EDG) is an ecological pattern where biodiversity changes with elevation. The EDG states that species richness tends to increase as elevation increases, up to a certain point, creating a “diversity bulge” at middle elevations, after which it decreases with altitude. Therefore, it is true that along the mountain gradients, biodiversity is normally greater in the lower altitudes as compared to the higher altitudes.

 

Question 7

Biodiversity forms the basis for human existence in the following ways:

  1. Soil formation

  2. Prevention of soil erosion

  3. Recycling of waste

  4. Pollination of crops

Select the correct answer using the codes given below:

(a) 1, 2 and 3 only

(b) 2, 3 and 4 only

(c) 1 and 4 only

(d) 1, 2, 3 and 4

Ans: d

Sub-Theme: Services provided by Biodiversity

Services provided by Biodiversity: Biodiversity provides a number of natural services for human beings:

 

 

 

Ecosystem services

•   Protection of water resources

•   Soils formation and protection

•   Nutrient storage and recycling

•   Pollution breakdown and absorption

•   Contribution to climate stability

•   Maintenance of ecosystems

•   Recovery from unpredictable events

 

 

 

Biological services

•   Food

•   Medicinal resources and pharmaceutical drugs

•   Wood products

•   Ornamental plants

•   Breeding stocks

•   Diversity in genes, species and ecosystems

 

Social services

•   Research, education and monitoring

•   Recreation and tourism

•   Cultural values

 

Question 8

The Himalayan Range is very rich in species Which one among the following is the most appropriate reason for this phenomenon?

(a) It has high rainfall that supports luxuriant vegetative growth

(b) It is a confluence of different biogeographical zones

(c) Exotic and invasive species have not been introduced in this region

(d) It has less human interference

Ans: b

Sub-Theme: Biomes

Option (b) is the correct answer: Being a confluence of different biogeographical zones Himalayan Range is very rich in species diversity.

Biogeographic Zones:

  • The categorization of India into biogeographic regions is known as biogeographic classification.
  • The study of the geographic and geological distribution of biological species, animals, and ecosystems is known as biogeography.

There are ten biogeographic zones in India:

  1. Trans-Himalayan
  2. Himalayan zone
  3. Desert
  4. Semi Arid
  5. Western ghat
  6. Deccan plateau
  7. Gangetic plain
  8. North east
  9. Coastal zone
  10. Islands

 

Question 9

With reference to micro-irrigation, which of the following statements is/are correct?

  1. Fertilizer/nutrient loss can be reduced.

  2. It is the only means of irrigation in dryland farming.

  3. In some areas of farming, the receding of groundwater tables can be checked.

Select the correct answer using the codes given below:

(a) 1 only

(b) 2 and 3 only

(c) 1 and 3 only

(d) 1, 2 and 3

Ans: c

Sub-Theme: Agriculture and Environment

Micro-irrigation:

  • Micro-irrigation is the slow distribution of water to the soil using surface drip and micro-sprinkler systems in the form of discrete or continuous drips, tiny streams, or miniature spray.

Benefits of micro irrigation systems:

  • Helps in water-saving and checking further depletion
  • Increased irrigation efficiency
  • Higher yields
  • Less water loss
  • Energy efficient
  • Lower consumption of fertilizers
  • Weed and disease reduction
  • Cost savings
  • Precision farming

Statement 1 is correct: An efficient drip irrigation system reduces the consumption of fertiliser through fertigation.

Statement 2 is incorrect: Dryland farming is defined as the production of crops without irrigation in regions that receive rainfall of less than 500mm annually.

Statement 3 is correct: Micro-irrigation helps in water-saving and checking further groundwater depletion.

 

Question 10

There is a concern over the increase in harmful algal blooms in the seawaters of India. What could be the causative factors for this phenomenon?

  1. Discharge of nutrients from the estuaries.

  2. Run-off from the land during the monsoon.

  3. Upwelling in the seas.

Select the correct answer from the codes given below:

(a) 1 only

(b) 1 and 2 only

(c) 2 and 3 only

(d) 1, 2 and 3

Ans: d

Sub-Theme: Water Pollution

Algal Bloom:

  • A rapid rise in the population of algae in an aquatic system is known as an algal bloom, marine bloom, or water bloom.
  • Algal blooms can appear in both marine and freshwater habitats.
  • “Harmful algal blooms (HAB)” are blooms that can affect wildlife or the environment.
  • Harmful Algal Bloom can lead to fish die- offs, cities cutting off water to residents, or states having to close fisheries.

Responsible Factors for ALGAL BLOOM

•   Nutrients
•   Temperature
•   Light
•   Stable (Calm Waters) Conditions
•   Turbidity
•   Aquaculture Operations

Statement 1 is correct: Nutrients promote and support the growth of algae and Cyanobacteria. The Eutrophication (nutrient enrichment) of waterways is considered as a major factor behind the increase in harmful algal blooms.

Statement 2 is correct: External sources include runoff and soil erosion from fertilized agricultural areas, erosion from river banks, river beds, land clearing (deforestation), and sewage effluent.

Statement 3 is correct: Water that rises to the surface as a result of upwelling is typically colder and is rich in nutrients, which again leads to algal blooms.

 

 

Question 11

Human activities in the recent past have caused the increased concentration of carbon dioxide in the atmosphere, but a lot of it does not remain in the lower atmosphere because of

  1. Its escape into the outer stratosphere.

  2. The photosynthesis by phytoplankton in the oceans.

  3. The trapping of air in the polar ice caps.

Which of the statements given above is/are correct?

(a) 1 and 2

(b) 2 only

(c) 2 and 3

(d) 3 only

Ans: b

Sub-Theme: Carbon Mitigation Strategies

Carbon Sequestration:

  • To slow down global warming, carbon sequestration involves the capture and storage of CO2. It is captured from the air, industries or power stations and stored permanently underground.
  • This promotes Long-term reserve of CO2 or forms of carbon to control temperatures.
  • Lowering of the amount of GHGs present in the air due to combustion of fossil fuels.

3 main steps to Carbon Capture and Storage (CCS):

  1. Trapping & separating CO2 from other gases
  2. Transporting captured CO2 to storage location
  3. Storing CO2 far from the atmosphere, either in the deep ocean or underground.

Statement 1 is incorrect: Some amount of CO2 does escape but not much CO2 is denser than Nitrogen, Oxygen and Argon (the main components of the atmosphere) and thus this tend to stay more in the lower atmosphere, however, this does not stop some of it moving to the upper atmosphere by the process of diffusion.

Statement 2 is correct: The photosynthesis by phytoplankton in the oceans is natural carbon sequestration.

Statement 3 is incorrect: The trapping of CO2 by polar ice is not known.

 

Question 12

Consider the following:

  1. Carbon dioxide

  2. Oxides of Nitrogen

  3. Oxides of Sulphur

Which of the above is/are the emission/emissions from coal combustion at thermal power plants?

(a) 1 only

(b) 2 and 3 only

(c) 1 and 3 only

(d) 1, 2 and 3

Ans: d

Sub-Theme: Major air pollutants and their sources/Green House Gases

Option (d) is the correct answer: Emissions of all the above-mentioned gases i.e. Carbon dioxide, Oxides of Nitrogen and Oxides of Sulphur take place from coal combustion at thermal power plants.

Emission of Gases from Coal Combustion

Gas Sources and Causes
Carbon dioxide (CO2) Burning of fossil fuels, deforestation
 

Chlorofluorocarbons

(CFCs)

Refrigeration, solvents, insulation foams, aero propellants, industrial and commercial uses
 

Methane (CH4)

Growing paddy, excreta of cattle and other livestock, termites, burning of fossil fuel, wood, landfills, wetlands, fertilizer factories.
Nitrogen oxides (N2O) Burning of fossil fuels, fertilizers; burning of wood and crop residue.
Carbon Monoxide (CO) Iron ore smelting, burning of fossil fuels, burning e-waste.

 

Question 13

The formation of ozone hole in the Antarctic region has been a cause of concern. What could be the reason for the formation of this hole?

(a) Presence of prominent tropospheric turbulence; and inflow of chlorofluorocarbons

(b) Presence of prominent polar front and stratospheric clouds; and inflow of chlorofluorocarbons.

(c) Absence of polar front and stratospheric clouds; and inflow of methane and chlorofluorocarbons.

(d) Increased temperature at polar region due to global warming

Ans: b

Sub-Theme: Ozone Depletion

Option (b) is the correct answer: Formation of Polar front, Polar Stratospheric Clouds lead to formation of ozone hole in the Antarctic region.

  • The formation of the ozone hole in the Antarctic has been an annual occurrence (in the months of September, October and November, due to a set of special meteorological and chemical conditions that arise at the South Pole, such as the presence of strong polar front, polar stratospheric clouds and increasing inflow of human- made chemicals like CFCs, etc.

Antarctic Ozone Hole:

  • The Antarctic “ozone hole” was discovered by British Antarctic Survey scientists Farman, Gardiner and Shanklin in 1985.

Causes:

  • Due to an increase in halocarbons (a compound in which the hydrogen of a hydrocarbon is replaced by halogens like chlorine, bromine, iodine etc.) in the atmosphere.
  • Presence of Polar Front which is a large area of low pressure and cold air that surrounds both of Earth’s poles.
  • The Cl-catalysed ozone depletion is enhanced in the presence of polar stratospheric clouds (PSCs) convert “reservoir” compounds into reactive free radicals (Cl and ClO), which in turn deplete ozone. In this way, PSCs accelerate ozone depletion.

 

Question 14

Regarding “carbon credits”, which one of the following statements is not correct?

(a) The carbon credit system was ratified in conjunction with the Kyoto Protocol

(b) Carbon credits are awarded to countries or groups that have reduced greenhouse gases below their emission quota

(c) The goal of the carbon credit system is to limit the increase of carbon dioxide emission

(d) Carbon credits are traded at a price fixed from time to time of the United Nation Environment Programme

Ans: d

Sub-Theme: Carbon Mitigation Strategies

Carbon Credit:

  • A carbon credit is a permit that allows the company the right to emit 1 ton of CO2 or its equivalent.
  • One credit permits the emission of a mass equal to one ton of carbon dioxide
  • Options (a), (b) and (c) are correct: The carbon credit system was ratified in conjunction with the Kyoto Protocol under UNFCCC to encourage groups or countries to reduce carbon emissions.
  • Option (d) is incorrect: Carbon credit prices are traded on an exchange and their prices are never fixed.

 

Question 15

With reference to India, consider the following Central Acts:

  1. Import and Export (Control) Act, 1947

  2. Mining and Mineral development (Regulation) Act, 1957

  3. Customs Act, 1962

  4. Indian Forest Act, 1927

Which of the above Acts have relevance to/bearing on the biodiversity conservation in the country?

(a) 1 and 3 only

(b) 2, 3 and 4 only

(c) 1, 2, 3 and 4

(d) None of the above Acts.

Ans: c

Sub-Theme: National efforts to protect biodiversity/Laws and Legislations

 

 

The Indian Forest Act, 1927

•   In order to increase accountability for forest protection, this act lays out the steps that must be taken when designating a region as a Reserved Forest, Protected Forest, or Village Forest.
Import and Export (Control) Act,

1947

•   Import and export of GMOs or exotic species are prohibited whereas certain medicinal plants are subjected to high customs duties to regulate their trade.
 

Mining and Mineral Development (Regulation) Act 1957

•   Mining activities, whether occurring within or near Protected Areas, cause a range of environmental; consequences that can be severe and irreversible. Thus mining is restricted in reserved areas.
 

Customs Act, 1962

•   Import and export of GMOs or exotic species are prohibited whereas certain medicinal plants are subjected to high customs duties to regulate their trade.

 Final Result – CIVIL SERVICES EXAMINATION, 2023.   Udaan-Prelims Wallah ( Static ) booklets 2024 released both in english and hindi : Download from Here!     Download UPSC Mains 2023 Question Papers PDF  Free Initiative links -1) Download Prahaar 3.0 for Mains Current Affairs PDF both in English and Hindi 2) Daily Main Answer Writing  , 3) Daily Current Affairs , Editorial Analysis and quiz ,  4) PDF Downloads  UPSC Prelims 2023 Trend Analysis cut-off and answer key

THE MOST
LEARNING PLATFORM

Learn From India's Best Faculty

      

 Final Result – CIVIL SERVICES EXAMINATION, 2023.   Udaan-Prelims Wallah ( Static ) booklets 2024 released both in english and hindi : Download from Here!     Download UPSC Mains 2023 Question Papers PDF  Free Initiative links -1) Download Prahaar 3.0 for Mains Current Affairs PDF both in English and Hindi 2) Daily Main Answer Writing  , 3) Daily Current Affairs , Editorial Analysis and quiz ,  4) PDF Downloads  UPSC Prelims 2023 Trend Analysis cut-off and answer key

Quick Revise Now !
AVAILABLE FOR DOWNLOAD SOON
UDAAN PRELIMS WALLAH
Comprehensive coverage with a concise format
Integration of PYQ within the booklet
Designed as per recent trends of Prelims questions
हिंदी में भी उपलब्ध
Quick Revise Now !
UDAAN PRELIMS WALLAH
Comprehensive coverage with a concise format
Integration of PYQ within the booklet
Designed as per recent trends of Prelims questions
हिंदी में भी उपलब्ध

<div class="new-fform">







    </div>

    Subscribe our Newsletter
    Sign up now for our exclusive newsletter and be the first to know about our latest Initiatives, Quality Content, and much more.
    *Promise! We won't spam you.
    Yes! I want to Subscribe.